SlideShare a Scribd company logo
1 of 80
ĐẠI HỌC QUỐC GIA HÀ NỘI
TRƯỜNG ĐẠI HỌC KHOA HỌC TỰ NHIÊN
- - - - - - - - - - - - - - - - - - - - - - -
NGUYỄN TÀI TUỆ
MỘT SỐ LỚP BẤT ĐẲNG THỨC
VÀ BÀI TOÁN CỰC TRỊ VỚI ĐA THỨC
ĐỐI XỨNG BA BIẾN
Chuyên ngành: PHƯƠNG PHÁP TOÁN SƠ CẤP
Mã số: 60.46.01.13
LUẬN VĂN THẠC SỸ KHOA HỌC
NGƯỜI HƯỚNG DẪN KHOA HỌC:
GS.TSKH. NGUYỄN VĂN MẬU
Hà Nội – Năm 2014
Mục lục
MỞ ĐẦU 3
1 Một số kiến thức bổ trợ 5
1.1 Đa thức đối xứng ba biến . . . . . . . . . . . . . . . . . . . . . . . 5
1.2 Tính chất cơ bản của bất đẳng thức . . . . . . . . . . . . . . . . . 6
1.3 Bất đẳng thức thường dùng . . . . . . . . . . . . . . . . . . . . . . 6
1.3.1 Bất đẳng thức AM-GM . . . . . . . . . . . . . . . . . . . . 6
1.3.2 Bất đẳng thức Cauchy - Schwarz . . . . . . . . . . . . . . . 7
1.3.3 Bất đẳng thức Karamata . . . . . . . . . . . . . . . . . . . 7
2 Bất đẳng thức có tổng các biến không đổi 9
2.1 Bất đẳng thức có tổng các biến không đổi với hàm phân thức hữu tỉ 9
2.1.1 Sử dụng bất đẳng thức AM-GM . . . . . . . . . . . . . . . 9
2.1.2 Sử dụng bất đẳng thức Cauchy-Schwarz . . . . . . . . . . . 15
2.1.3 Sử dụng các tính chất của hàm số . . . . . . . . . . . . . . 21
2.1.4 Bài toán liên quan . . . . . . . . . . . . . . . . . . . . . . . 31
2.2 Bất đẳng thức có tổng các biến không đổi với hàm vô tỉ . . . . . . 33
2.2.1 Sử dụng bất đẳng thức AM-GM . . . . . . . . . . . . . . . 33
2.2.2 Sử dụng bất đẳng thức Cauchy-Schwarz . . . . . . . . . . . 36
2.2.3 Sử dụng các tính chất của hàm số . . . . . . . . . . . . . . 41
2.2.4 Bài toán liên quan . . . . . . . . . . . . . . . . . . . . . . . 43
3 Bất đẳng thức có tích các biến không đổi 45
3.1 Bất đẳng thức có tích các biến không đổi với hàm phân thức hữu tỉ 45
3.1.1 Sử dụng bất đẳng thức AM-GM . . . . . . . . . . . . . . . 45
3.1.2 Sử dụng bất đẳng thức Cauchy-Schwarz . . . . . . . . . . . 50
3.1.3 Sử dụng các tính chất của hàm số . . . . . . . . . . . . . . 53
3.1.4 Bài toán liên quan . . . . . . . . . . . . . . . . . . . . . . . 55
3.2 Bất đẳng thức có tích các biến không đổi với hàm vô tỉ . . . . . . 56
3.2.1 Sử dụng bất đẳng thức AM-GM . . . . . . . . . . . . . . . 56
3.2.2 Sử dụng bất đẳng thức Cauchy-Schwarz . . . . . . . . . . . 59
3.2.3 Sử dụng các tính chất của hàm số . . . . . . . . . . . . . . 60
3.2.4 Bài toán liên quan . . . . . . . . . . . . . . . . . . . . . . . 62
1
MỤC LỤC
4 Một số lớp bài toán cực trị với đa thức đối xứng ba biến 63
4.1 Sử dụng bất đẳng thức AM-GM . . . . . . . . . . . . . . . . . . . 63
4.2 Sử dụng bất đẳng thức Cauchy-Schwarz . . . . . . . . . . . . . . . 68
4.3 Sử dụng các tính chất của hàm số . . . . . . . . . . . . . . . . . . . 73
4.4 Bài toán liên quan . . . . . . . . . . . . . . . . . . . . . . . . . . . . 77
KẾT LUẬN 78
TÀI LIỆU THAM KHẢO 79
2
MỞ ĐẦU
Bất đẳng thức là một nội dung cổ điển và quan trọng của Toán học. Ngay
từ đầu, sự ra đời và phát triển của bất đẳng thức đã đặt dấu ấn quan trọng,
chúng có sức hút mạnh mẽ đối với những người yêu toán, không chỉ ở vẻ đẹp
hình thức mà cả những bí ẩn nó mang đến luôn thôi thúc người làm toán phải
tìm tòi, sáng tạo. Bất đẳng thức còn có nhiều ứng dụng trong các môn khoa
học khác và trong thực tế. Ngày nay, bất đẳng thức vẫn luôn chiếm một vai trò
quan trọng và vẫn thường xuất hiện trong các kì thi quốc gia, quốc tế, Olympic.
Là một giáo viên THPT, tôi muốn nghiên cứu sâu hơn về bất đẳng thức nhằm
nâng cao chuyên môn phục vụ cho quá trình giảng dạy và bồi dưỡng học sinh
giỏi, vậy nên tôi đã chọn bất đẳng thức làm luận văn thạc sĩ của mình.
Bất đẳng thức vô cùng rộng lớn, trong thời gian ngắn, tôi chỉ có thể nghiên
cứu lĩnh vực nhỏ trong đó. Dưới sự hướng dẫn của GS. TSKH Nguyễn Văn Mậu,
tác giả đã hoàn thành luận văn với để tài
"Một số lớp bất đẳng thức và bài toán cực trị với đa thức đối xứng
ba biến."
Luận văn được chia làm bốn chương:
• Chương 1: Một số kiến thức bổ trợ.
• Chương 2: Bất đẳng thức với tổng không đổi.
• Chương 3: Bất đẳng thức có tích không đổi.
• Chương 4: Một số lớp bài toán cực trị với đa thức đối xứng ba biến.
Mặc dù có nhiều cố gắng, song do thời gian và trình độ còn hạn chế nên luận
văn khó tránh khỏi những thiếu sót. Vì vậy tác giả rất mong nhận được sự góp
ý của các thầy cô và các bạn để luận văn được hoàn thiện hơn.
Qua luận văn này, tác giả xin bày tỏ lòng biết ơn sâu sắc đến GS.TSKH
Nguyễn Văn Mậu, người Thầy đã truyền cho tác giả có niềm say mê nghiên cứu
3
MỞ ĐẦU
toán học. Thầy đã tận tình hướng dẫn, giúp đỡ tác giả trong suốt quá trình học
tập và hoàn thiện luận văn này.
Tác giả xin chân thành cảm ơn Ban giám hiệu, Phòng Đào tạo Sau đại học,
Khoa Toán- Cơ - Tin, các thầy cô đã tạo điều kiện thuận lợi cho em hoàn thành
bản luận văn này.
Em xin chân thành cảm ơn!
Hà Nội, ngày 01 tháng 12 năm 2014
Tác giả
4
Chương 1
Một số kiến thức bổ trợ
1.1 Đa thức đối xứng ba biến
1.1.1 Các khái niệm cơ bản
Định nghĩa 1.1. Một đơn thức ϕ(x, y, z) của các biến x, y, z được hiểu là hàm
số có dạng
ϕ(x, y, z) = aklmxk
yl
zm
,
trong đó k, l, m ∈ N được gọi là bậc của biến x, y, z, số aklm ∈ R∗ = R{0} được
gọi là hệ số của đơn thức, còn số k +l+m được gọi là bậc của đơn thức ϕ(x, y, z).
Định nghĩa 1.2. Một hàm số P(x, y, z) của các biến x, y, z được gọi là một đa
thức nếu nó có thể được biểu diễn ở dạng tổng hữu hạn các đơn thức
P(x, y, z) =
k,l,m∈N
k+l+m=n
aklmxk
yl
zm
, n ∈ N.
Bậc lớn nhất của các đơn thức trong đa thức được gọi là bậc của đa thức.
Định nghĩa 1.3. Đa thức P(x, y, z) được gọi là đối xứng, nếu nó không thay
đổi với mọi hoán vị của x, y, z, nghĩa là
P(x, y, z) = P(y, x, z) = P(z, y, x) = P(x, z, y).
Định nghĩa 1.4. Đa thức f(x, y, z) được gọi là thuần nhất bậc m, nếu
f(tx, ty, tz) = tm
f(x, y, z), t = 0
Định nghĩa 1.5. Các đa thức
σ1 = x + y + z, σ2 = xy + yz + zx, σ3 = xyz,
được gọi là đa thức đối xứng cơ sở của các biến x, y, z.
1.1.2 Tổng lũy thừa
5
Chương 1. Một số kiến thức bổ trợ
Định nghĩa 1.6. Các đa thức sk = xk + yk + zk, (k = 0, 1, ...), được gọi là tổng
lũy thừa bậc k của các biến x, y, z.
Định lý 1.1 ( Công thức Newton). Với mọi k ∈ Z, ta có hệ thức
sk = σ1sk−1 − σ2sk−2 + σ3sk−3.
Định lý 1.2. Một tổng lũy thừa sk = xk + yk + zk đều có thể biểu diễn được
dưới dạng một đa thức bậc n theo các biến σ1, σ2, σ3.
Định lý 1.3 (Công thức Waring). Tổng lũy thừa sk được biểu diễn qua cá đa
thức đối xứng cở sở theo công thức
sk
k
=
0≤l,m,n
l+2m+3n=k
(−1)k−l−m−n(l + m + n − 1)!
l!m!n!
σl
1σm
2 σn
3 .
1.2 Tính chất cơ bản của bất đẳng thức
1. a > b ⇔ a + c > b + c.
2. a > b, b > c thì a > c.
3. a > b thì
ca > cb khi c > 0
ca < cb khi c < 0.
4. a > b, c > d thì a + c > b + d.
5. a > b > 0, c > d > 0 thì ac > bd.
6. Với n nguyên dương, ta có
a < b ⇔ a2n+1
< b2n+1
0 < a < b ⇒ a2n
< b2n
.
1.3 Bất đẳng thức thường dùng
1.3.1 Bất đẳng thức AM-GM
Định lý 1.4. Giả sử a1, a2, . . . , an là các số thực không âm, khi đó ta luôn có
a1 + a2 + · · · + an
n
≥ n
√
a1a2 . . . an.
Đẳng thức xảy ra khi và chỉ khi a1 = a2 = · · · = an.
6
Chương 1. Một số kiến thức bổ trợ
Hệ quả 1.1. Với mọi số thực dương a1, a2, . . . , an ta có
1
a1
+
1
a2
+ · · · +
1
an
(a1 + a2 + · · · + an) ≥ n2
.
Đẳng thức xảy ra khi và chỉ khi a1 = a2 = · · · = an.
Hệ quả 1.2. Với mọi số thực a, b, c, ta luôn có
1. a2 + b2 + c2 ≥ ab + bc + ca
2. a2 + b2 + c2 ≥
(a + b + c)2
3
3. (a + b + c)2 ≥ 3(ab + bc + ca)
4. a2b2 + b2c2 + c2a2 ≥ abc(a + b + c)
5. (ab + bc + ca)2 ≥ 3abc(a + b + c).
1.3.2 Bất đẳng thức Cauchy - Schwarz
Định lý 1.5. Nếu a1, a2, . . . , an, b1, b2, . . . , bn là các số thực tùy ý thì
(a1b1 + a2b2 + · · · + anbn)2
≤ a2
1 + a2
2 + · · · + a2
n b2
1 + b2
2 + · · · + b2
n . ( )
Đẳng thức xảy ra khi và chỉ khi
a1
b1
=
a2
b2
= · · · =
an
bn
( ở đây ta sử dụng quy ước
nếu mẫu bằng 0 thì tử cũng bằng 0).
Nhận xét 1.1. Theo bất đẳng thức ( ), chọn ai =
xi
√
yi
và bi =
√
yi với xi, yi ∈
R, yi > 0. Ta thu được bất đẳng thức Cauchy-Schwarz dạng phân thức ( hay còn
gọi là bất đẳng thức Cauchy-Schwarz dạng Engel).
Hệ quả 1.3. Nếu x1, x2, . . . , xn là các số thực và y1, y2, . . . , yn là các số thực
dương thì
x2
1
y1
+
x2
2
y2
+ · · · +
x2
n
yn
≥
(x1 + x2 + . . . xn)2
y1 + y2 + · · · + yn
.
Đẳng thức xảy ra khi và chỉ khi
x1
y1
=
x2
y2
= · · · =
xn
yn
.
1.3.3 Bất đẳng thức Karamata
Định lý 1.6. Cho hai dãy số {xk, yk ∈ I(a, b), k = 1, 2, . . . , n}, thỏa mãn điều kiện
x1 ≥ x2 ≥ · · · ≥ xn, y1 ≥ y2 ≥ · · · ≥ yn
7
Chương 1. Một số kiến thức bổ trợ
và 


x1 ≥ y1
x1 + x2 ≥ y1 + y2
........
x1 + x2 + · · · + xn−1 ≥ y1 + y2 + · · · + yn−1
x1 + x2 + · · · + xn = y1 + y2 + · · · + yn
Khi đó, ứng với hàm số lồi f(x)(f (x) ≥ 0) trên I(a, b), ta đều có
f(x1) + f(x2) + · · · + f(xn) ≥ f(y1) + f(y2) + · · · + f(yn).
Đẳng thức xảy ra khi và chỉ khi xi = yi, i = 1, 2, . . . n.
Ta cũng phát biểu tương tự đối với hàm số lõm bằng cách đổi chiều dấu bất
đẳng thức.
Bổ đề 1.1. Cho hàm số y = f(x) liên tục và có đạo hàm cấp 2 trên I(a; b).
a. Nếu f (x) ≥ 0, ∀x ∈ I(a; b) thì f(x) ≥ f (x0)(x − x0) + f(x0), ∀x0 ∈ I(a; b).
b. Nếu f (x) ≤ 0, ∀x ∈ I(a; b) thì f(x) ≤ f (x0)(x − x0) + f(x0), ∀x0 ∈ I(a; b).
Đẳng thức trong hai bất đẳng thức trên xảy ra khi và chỉ khi x = x0.
8
Chương 2
Bất đẳng thức có tổng các biến
không đổi
2.1 Bất đẳng thức có tổng các biến không đổi với hàm phân
thức hữu tỉ
2.1.1 Sử dụng bất đẳng thức AM-GM
Đối với bất đẳng thức P(x, y, z) ≥ 0 (≤ 0), Trong đó P(x, y, z) là đa thức hoặc
phân thức hữu tỉ và có tổng x + y + z không đổi, thì khi đó sử dụng các kĩ thuật
của bất đẳng thức AM − GM như dự đoán dấu bằng xảy ra, AM − GM ngược
dấu, đặt ẩn phụ, ... tỏ ra rất hiệu quả.
Bài toán 2.1. Với a, b, c là các số thực dương thỏa mãn a + b + c = 3. Chứng
minh rằng
a2
b + 2
+
b2
c + 2
+
c2
a + 2
≥ 1.
Chứng minh.
Áp dụng bất đẳng thức AM-GM, ta có
a2
b + 2
+
b + 2
9
≥
2a
3
b2
c + 2
+
c + 2
9
≥
2b
3
c2
a + 2
+
a + 2
9
≥
2c
3
.
Cộng các bất đẳng thức cùng chiều ta được
a2
b + 2
+
b2
c + 2
+
c2
a + 2
≥
5
9
(a + b + c) −
2
3
= 1.
Đẳng thức xảy ra khi và chỉ khi a = b = c = 1.
9
Chương 2. Bất đẳng thức có tổng các biến không đổi
Bài toán 2.2. Với a, b, c là các số thực dương thỏa mãn a + b + c = 3. Chứng
minh rằng
2(ab + bc + ca) +
1
ab
+
1
bc
+
1
ca
≥ 9.
Chứng minh.
Cộng cả hai vế của bất đẳng thức cần chứng minh với a2 + b2 + c2, ta được
a2
+ b2
+ c2
+ 2(ab + bc + ca) +
1
ab
+
1
bc
+
1
ca
≥ 9 + a2
+ b2
+ c2
⇔(a + b + c)2
+
1
ab
+
1
bc
+
1
ca
≥ 9 + a2
+ b2
+ c2
⇔
1
ab
+
1
bc
+
1
ca
≥ a2
+ b2
+ c2
⇔
a + b + c
abc
≥ a2
+ b2
+ c2
⇔
3
abc
≥ a2
+ b2
+ c2
⇔abc(a2
+ b2
+ c2
) ≤ 3
⇔abc(a + b + c)(a2
+ b2
+ c2
) ≤ 9.
Ta có
abc(a + b + c)(a2
+ b2
+ c2
) ≤
(ab + bc + ca)2
3
(a2
+ b2
+ c2
).
Do đó ta chứng minh
(ab + bc + ca)2
3
(a2
+ b2
+ c2
) ≤ 9 ⇔ (ab + bc + ca)2
(a2
+ b2
+ c2
) ≤ 27.
Áp dụng bất đẳng thức AM-GM, ta có
(ab + bc + ca)2
(a2
+ b2
+ c2
) ≤
(ab + bc + ca) + (ab + bc + ca) + a2 + b2 + c2
3
3
=
(a + b + c)2
3
3
= 27.
Vậy bất đẳng thức được chứng minh.
Đẳng thức xảy ra khi và chỉ khi a = b = c = 1.
Bài toán 2.3. Với các số thực không âm a, b, c thỏa mãn a + b + c = 3. Chứng
minh rằng
2 a2
b + b2
c + c2
a + 3 a2
+ b2
+ c2
+ 4abc ≥ 19.
Chứng minh.
10
Chương 2. Bất đẳng thức có tổng các biến không đổi
Ta có
19 = 3 (a + b + c)2
− 8 = 3 a2
+ b2
+ c2
+ 6 (ab + bc + ca) − 8.
Khi đó bất đẳng thức cần chứng minh trở thành
2 a2
b + b2
c + c2
a + 4abc ≥ 6 (ab + bc + ca) − 8
⇔a2
b + b2
c + c2
a + 2abc + 4 ≥ 3 (ab + bc + ca) .
Mà ta lại có
3 (ab + bc + ca) = (a + b + c) (ab + bc + ca)
= a2
b + b2
c + c2
a + ab2
+ bc2
+ ca2
+ 3abc.
Do đó bất đẳng thức cần chứng minh tương đương với
4 ≥ ab2
+ bc2
+ ca2
+ abc.
Không mất tính tổng quát giả sử a ≥ b ≥ c ≥ 0
⇒ (b − a) (b − c) ≤ 0
⇔b2
− bc − ab + ac ≤ 0
⇔ab2
+ a2
c ≤ a2
b + abc.
Như vậy
ab2
+ bc2
+ ca2
+ abc ≤ bc2
+ abc + a2
b + abc
= bc2
+ 2abc + a2
b
= b (a + c)2
= 4b
a + c
2
a + c
2
≤ 4


b +
a + c
2
+
a + c
2
3


3
= 4.
Vậy bất đẳng thức được chứng minh.
Đẳng thức xảy ra khi và chỉ khi a = b = c = 1.
11
Chương 2. Bất đẳng thức có tổng các biến không đổi
Bài toán 2.4 (Bulgaria TST 2003). Với ba số thực dương a, b, c thỏa mãn điều
kiện a + b + c = 3. Chứng minh rằng
a
1 + b2
+
b
1 + c2
+
c
1 + a2
≥
3
2
.
Chứng minh.
Áp dụng bất đẳng thức AM-GM, ta có
a
1 + b2
= a −
ab2
1 + b2
≥ a −
ab2
2b
= a −
ab
2
.
Tương tự, ta có
b
1 + c2
≥ b −
bc
2
c
a2 + 1
≥ c −
ac
2
.
Cộng vế với vế, ta có
a
1 + b2
+
b
1 + c2
+
c
1 + a2
≥ a + b + c −
1
2
(ab + bc + ca) = 3 −
ab + bc + ca
2
.
Mặt khác, ta có
ab + bc + ca ≤
(a + b + c)2
3
= 3
do đó
a
1 + b2
+
b
1 + c2
+
c
1 + a2
≥ 3 −
ab + bc + ca
2
≥
3
2
.
Đẳng thức xảy ra khi và chỉ khi a = b = c = 1.
Bài toán 2.5 (Turkey TST 2007). Với a, b, c là các số thực dương thỏa mãn
a + b + c = 1. Chứng minh rằng
1
ab + 2c2 + 2c
+
1
bc + 2a2 + 2a
+
1
ca + 2b2 + 2b
≥
1
ab + bc + ca
.
Chứng minh.
Áp dụng bất đẳng thức AM-GM, ta có
12
Chương 2. Bất đẳng thức có tổng các biến không đổi
ab + 2c2
+ 2c = ab + 2c2
+ 2c(a + b + c)
= ab + 4c2
+ 2ac + 2bc
= (b + 2c)(a + 2c)
=
a(b + 2c)b(a + 2c)
ab
=
(ab + 2ac)(ab + 2bc)
ab
≤
[ab + 2ac + ab + 2bc]2
4ab
=
(ab + bc + ca)2
ab
từ đó suy ra
1
ab + 2c2 + 2c
≥
ab
(ab + bc + ca)2
.
Tương tự, ta có
1
bc + 2a2 + 2a
≥
bc
(ab + bc + ca)2
1
ca + 2b2 + 2b
≥
ca
(ab + bc + ca)2
.
Cộng các bất đẳng thức cùng chiều, ta có
1
ab + 2c2 + 2c
+
1
bc + 2a2 + 2a
+
1
ca + 2b2 + 2b
≥
1
ab + bc + ca
.
Đẳng thức xảy ra khi và chỉ khi a = b = c =
1
3
.
Bài toán 2.6. Với các số thực dương a, b, c thỏa mãn a + b + c = 3. Chứng minh
rằng
a2 + bc
b + ca
+
b2 + ca
c + ab
+
c2 + ab
a + bc
≥ 3.
Chứng minh.
Bất đẳng thức cần chứng minh tương đương với
a2 + bc
3b + 3ca
+
b2 + ca
3c + 3ab
+
c2 + ab
3a + 3bc
≥ 1
⇔
a2 + bc
b(a + b + c) + 3ca
+
b2 + ca
c(a + b + c) + 3ab
+
c2 + ab
a(a + b + c) + 3bc
≥ 1.
Mặt khác
3b + 3ac = b(a + b + c) + 3ac ≤ b(a + b + c) + ac + a2
+ c2
= a2
+ b2
+ c2
+ ab + bc + ca
13
Chương 2. Bất đẳng thức có tổng các biến không đổi
do đó
a2 + bc
3b + 3ac
≥
a2 + bc
a2 + b2 + c2 + ab + bc + ca
.
Tương tự, ta có
b2 + ca
3c + 3ab
≥
b2 + ca
a2 + b2 + c2 + ab + bc + ca
c2 + ab
3a + 3bc
≥
c2 + ab
a2 + b2 + c2 + ab + bc + ca
.
Cộng các bất đẳng thức cùng chiều, ta có
a2 + bc
3b + 3ca
+
b2 + ca
3c + 3ab
+
c2 + ab
3a + 3bc
≥ 1.
Bất đẳng thức được chứng minh.
Đẳng thức xảy ra khi và chỉ khi a = b = c = 1.
14
Chương 2. Bất đẳng thức có tổng các biến không đổi
2.1.2 Sử dụng bất đẳng thức Cauchy-Schwarz
Nếu yêu cầu của bất đẳng thức là chứng minh P(x, y, z) ≥ 0(≤ 0) với P(x, y, z)
có dạng tổng các bình phương hoặc phân thức với tử số của mỗi phân thức có
dạng bình phương như vậy ta có thể nghĩ đến việc áp dụng bất đẳng thức
Cauchy- Schwarz. Hoặc sử dụng giả thiết x + y + z không đổi ta có thể biến đổi
để đưa bất đẳng thức về dạng trên để sử dụng Cauchy-Schwarz.
Bài toán 2.7. Với các số thực dương a, b, c thỏa mãn điều kiện a + b + c = 1.
Chứng minh rằng
a
1 + b − a
+
b
1 + c − b
+
c
1 + a − c
≥ 1.
Chứng minh.
Ta luôn có
a
1 + b − a
+
b
1 + c − b
+
c
1 + a − c
=
a
2b + c
+
b
2c + a
+
c
2a + b
=
a2
2ab + ac
+
b2
2bc + ab
+
c2
2ac + bc
.
Áp dụng bất đẳng thức Cauchy-Schwarz, ta có
a2
2ab + ac
+
b2
2bc + ab
+
c2
2ac + bc
≥
(a + b + c)2
3 (ab + bc + ca)
.
Theo hệ quả của bất đẳng thức AM-GM, ta có
(a + b + c)2
≥ 3(ab + bc + ca)
do đó
a2
2ab + ac
+
b2
2bc + ab
+
c2
2ac + bc
≥
(a + b + c)2
3 (ab + bc + ca)
≥ 1.
Đẳng thức xảy ra khi và chỉ khi a = b = c = 1.
Bài toán 2.8. Với các số thực dương a, b, c sao cho a + b + c = 1. Chứng minh
1
a2 + b2 + c2
+
1
ab
+
1
bc
+
1
ca
≥ 30.
Chứng minh.
Áp dụng bất đẳng thức Cauchy-Schwarz cho hai bộ số
1
√
a2 + b2 + c2
,
1
√
ab
,
1
√
bc
,
1
√
cd
15
Chương 2. Bất đẳng thức có tổng các biến không đổi
và
a2 + b2 + c2, 3
√
ab, 3
√
bc, 3
√
ca
ta có
1
a2 + b2 + c2
+
1
ab
+
1
bc
+
1
ca
a2
+ b2
+ c2
+ 9ab + 9bc + 9ca ≥ (1 + 3 + 3 + 3)2
⇔
1
a2 + b2 + c2
+
1
ab
+
1
bc
+
1
ca
(a + b + c)2
+ 7(ab + bc + ca) ≥ 100
⇔
1
a2 + b2 + c2
+
1
ab
+
1
bc
+
1
ca
≥
100
(a + b + c)2 + 7(ab + bc + ca)
.
Theo hệ quả của bất đẳng thức AM − GM, ta có
(a + b + c)2
3
≥ ab + bc + ca
nên
1
a2 + b2 + c2
+
1
ab
+
1
bc
+
1
ca
≥
100
1 + 7
(a + b + c)2
3
= 30.
Đẳng thức xảy ra khi và chỉ khi a = b = c =
1
3
.
Bài toán 2.9 (Iran MO TST 2009). Với các số thực dương a, b, c thỏa mãn điều
kiện a + b + c = 3. Chứng minh rằng
1
a2 + b2 + 2
+
1
b2 + c2 + 2
+
1
c2 + a2 + 2
≤
3
4
.
Chứng minh.
Bất đẳng thức cần chứng minh
⇔
1
2
−
1
a2 + b2 + 2
+
1
2
−
1
b2 + c2 + 2
+
1
2
−
1
c2 + a2 + 2
≥
3
2
−
3
4
⇔
a2 + b2
a2 + b2 + 2
+
b2 + c2
b2 + c2 + 2
+
c2 + a2
c2 + a2 + 2
≥
3
2
⇔
(a + b)2 + (a − b)2
a2 + b2 + 2
+
(b + c)2 + (b − c)2
b2 + c2 + 2
+
(c + a)2 + (c − a)2
c2 + a2 + 2
≥ 3
⇔
(a + b)2
a2 + b2 + 2
+
(b + c)2
b2 + c2 + 2
+
(c + a)2
c2 + a2 + 2
+
(a − b)2
a2 + b2 + 2
+
(b − c)2
b2 + c2 + 2
+
(c − a)2
c2 + a2 + 2
≥ 3.
16
Chương 2. Bất đẳng thức có tổng các biến không đổi
Theo bất đẳng thức Cauchy-Schwarz, ta có
(a + b)2
a2 + b2 + 2
+
(b + c)2
b2 + c2 + 2
+
(c + a)2
c2 + a2 + 2
≥
[2(a + b + c)]2
2a2 + 2b2 + 2c2 + 6
(1)
và
(b − a)2
a2 + b2 + 2
+
(b − c)2
b2 + c2 + 2
+
(c − a)2
c2 + a2 + 2
≥
(2b − 2a)2
2a2 + 2b2 + 2c2 + 6
. (2)
Bất đẳng thức được chứng minh nếu
4 (a + b + c)2
+ (2b − 2a)2
2a2 + 2b2 + 2c2
≥ 3
⇔2(a + b + c)2
+ 2(b − a) ≥ 3(a2
+ b2
+ c2
+ 3)
⇔ a2
+ b2
+ c2 2
+ 2 (b − a)2
≥ a2
+ b2
+ c2
⇔bc − ab + ca − c2
≥ 0
⇔(c − a)(b − c) ≥ 0. (3)
Có thể nhận thấy bất đẳng thức (3) không phải luôn đúng nhưng ta có thể
ép nó đúng. Thật vậy sử dụng các đánh giá tương tự (3), ta có
(a − b)2
a2 + b2 + 2
+
(c − b)2
b2 + c2 + 2
+
(c − a)2
c2 + a2 + 2
≥
(2c − 2b)2
2a2 + 2b2 + 2c2 + 6
(a − b)2
a2 + b2 + 2
+
(b − c)2
b2 + c2 + 2
+
(a − c)2
c2 + a2 + 2
≥
(2a − 2c)2
2a2 + 2b2 + 2c2 + 6
và cùng lần lượt đưa bài toán về chứng minh
(a − b)(c − a) ≥ 0 (4)
(b − c)(a − b) ≥ 0. (5)
Như vậy nếu trong các bất đẳng thức (3),(4) và (5) có một bất đẳng thức
đúng thì bài toán sẽ được chứng minh song. Ta thấy rằng
[(c − a)(b − c)] [(a − b)(c − a)] [(b − c)(a − b)] = (a − b)2
(b − c)2
(c − a)2
≥ 0
nên ít nhất một trong ba số (c-a)(b-c), (a-b)(c-a),(b-c)(a-b) sẽ có một số không
âm. Tức là phải có ít nhất một bất đẳng thức đúng hay bất đẳng thức được
chứng minh.
Đẳng thức xảy ra khi và chỉ khi a = b = c = 1.
17
Chương 2. Bất đẳng thức có tổng các biến không đổi
Nhận xét 2.1. Có nhiều công cụ hỗ trợ ta thực hiện phương pháp dồn biến,
dưới đây ta xem xét ứng dụng yếu tố "ít nhất" và bất đẳng thức Cauchy-Schwarz
trong việc giảm biến số của bất đẳng thức. Cụ thể có thể đưa bất đẳng thức ba
biến về bất đẳng thức một biến để chứng minh. Ý tưởng của kĩ thuật như sau:
Với bốn số thực a, b, c, k ta có
[(a − k)(b − k)] [(b − k)(c − k)] [(c − k)(a − k)] = (a − k)2
(b − k)2
(c − k)2
≥ 0.
Do đó trong ba số (a − k)(b − k), (b − k)(c − k), (c − k)(a − k) sẽ có "ít nhất"
một số không âm. Giả sử (a − k)(b − k) ≥ 0 thế thì
a2
+ b2
= k2
+ (a + b − k)2
− 2(a − k)(b − k) ≤ k2
+ (a + b − k)2
.
Như vậy để chứng minh bất đẳng thức có giả thiết dạng a + b + c = s và đẳng
thức xảy ra khi 2 biến đằng một giá trị nào đó ta có thể sử dụng đánh giá trên
để làm giảm biến số của bất đẳng thức ban đầu. Cụ thể chọn k = m ( đảm bảo
dấu bằng) và ta có
a2
+ b2
≤ m2
+ (a + b − m)2
= m2
+ (s − c − m)2
.
Nghĩa là có thể sử dụng đánh giá này vào bài toán thì ta sẽ chỉ còn phải
chứng minh bất đẳng thức của một biến c.
Bài toán 2.10. Với a, b, c là các số thực thỏa mãn điều kiện a+b+c = 3. Chứng
minh rằng
a
7a2 + 11
+
b
7b2 + 11
+
c
7c2 + 11
≤
1
6
.
Chứng minh.
Bất đẳng thức cần chứng minh
⇔1 −
14a
7a2 + 11
+ 1 −
14b
7b2 + 11
+ 1 −
14c
7c2 + 11
≥ 3 −
14
6
⇔
7(a − 1)2 + 4
7a2 + 11
+
7(b − 1)2 + 4
7b2 + 11
+
7(c − 1)2 + 4
7c2 + 11
≥
2
3
⇔4
1
7a2 + 11
+
1
7b2 + 11
+
1
7c2 + 11
+ 7
(a − 1)2
7a2 + 11
+
(b − 1)2
7b2 + 11
+
(c − 1)2
7c2 + 11
≥
2
3
.
Áp dụng bất đẳng thức Cauchy -Schwarz, ta có
1
7a2 + 11
+
1
7b2 + 11
+
1
7c2 + 11
≥
(1 + 1 + 1)2
7(a2 + b2 + c2) + 33
=
9
7(a2 + b2 + c2) + 33
=
3(a + b + c)2
21(a2 + b2 + c2) + 11(a + b + c)2
18
Chương 2. Bất đẳng thức có tổng các biến không đổi
và cũng theo bất đẳng thức Cauchy-Schwarz, ta có
(1 − a)2
7a2 + 11
+
(b − 1)2
7b2 + 11
+
(c − 1)2
7c2 + 11
≥
(1 − a + b − 1 + c − 1)2
7(a+b2 + c2) + 33
=
(b + c − a − 1)2
7(a2 + b2 + c2) + 11.3
=
3 b + c − a −
a + b + c
3
2
21(a2 + b2 + c2) + 11.(a + b + c)2
=
4(b + c − 2a)2
3 [21(a2 + b2 + c2) + 11.(a + b + c)2]
.
Bất đẳng thức được chứng minh nếu
4.3(a + b + c)2
21(a2 + b2 + c2) + 11(a + b + c)2
+
7.4(b + c − 2a)2
3 [21(a2 + b2 + c2) + 11.(a + b + c)2]
≥
2
3
⇔2.9(a + b + c)2
+ 7.2(b + c − 2a)2
≥ 21(a2
+ b2
+ c2
) + 11(a + b + c)2
⇔42 [a(a − c) − b(a − c)] ≥ 0
⇔42(a − c)(a − b) ≥ 0. (1)
Rõ ràng (1 ) không phải lúc nào cũng đúng, nhưng từ đó vẫn đi đến điều
phải chứng minh bằng cách đánh giá tương tự
(a − 1)2
7a2 + 11
+
(1 − b)2
7b2 + 11
+
(c − 1)2
7c2 + 11
≥
4(a + c − 2b)2
3 [21(a2 + b2 + c2) + 11.(a + b + c)2]
(a − 1)2
7a2 + 11
+
(b − 1)2
7b2 + 11
+
(1 − c)2
7c2 + 11
≥
4(a + b − 2c)2
3 [21(a2 + b2 + c2) + 11.(a + b + c)2]
và ta lần lượt đưa bài toán về xét tính đúng sai của
42(b − a)(b − c) ≥ 0 (2)
42(c − a)(c − b) ≥ 0 (3)
mà ta lại có
(a − b)(a − c) + (b − a)(b − c) + (c − a)(c − b) = a2
+ b2
+ c2
− ab − bc − ca ≥ 0.
Do đó trong (1),(2),(3) có ít nhất một bất đẳng thức đúng. Từ đó suy ra điều
phải chứng minh.
Đẳng thức xảy ra khi và chỉ khi a = b = c = 1.
19
Chương 2. Bất đẳng thức có tổng các biến không đổi
Bài toán 2.11. Chứng minh rằng với các số thực dương a, b, c tùy ý ta đều có
(b + c − a)2
a2 + (b + c)2
+
(c + a − b)2
b2 + (c + a)2
+
(a + b − c)2
c2 + (a + b)2
≥
3
5
.
Chứng minh.
Do bất đẳng thức đã cho thuần nhất với biến a, b, c nên ta hoàn toàn có thể
chuẩn hóa cho a + b + c = 1. Khi đó bất đẳng thức đã cho được viết dưới dạng
(1 − 2a)2
2a2 − 2a + 1
+
(1 − 2b)2
2b2 − 2b + 1
+
(1 − 2c)2
2c2 − 2c + 1
≥
3
5
.
Ta có
a −
1
3
b −
1
3
b −
1
3
c −
1
3
c −
1
3
a −
1
3
= a −
1
3
2
b −
1
3
2
c −
1
3
2
≥ 0.
Không mất tính tổng quát, giả sử a −
1
3
b −
1
3
≥ 0.
Ta có
a2
+ b2
≤
1
9
+ (a + b −
1
3
)2
=
1
9
+
2
3
− c
2
.
Áp dụng bất đẳng thức Cauchy-Schwarz, ta có
(1 − 2a)2
2a2 − 2a + 1
+
(1 − 2b)2
2b2 − 2b + 1
≥
[2 − 2(a + b)]2
2(a2 + b2) − 2(a + b) + 2
≥
[2 − 2(a + b)]2
2
1
9
+ (
2
3
− c)2 − 2 (1 − c) + 2
=
2c2
1
9
+
2
3
− c
2
+ c
=
18c2
9c2 − 3c + 5
.
Bất đẳng thức được chứng minh nếu ta chứng minh được
(1 − 2c)2
2c2 − 2c + 1
+
18c2
9c2 − 3c + 5
≥ 35
⇔(3c − 1)2
(17c2
− 8c + 5) ≥ 0 luôn đúng .
Đẳng thức xảy ra khi và chỉ khi a = b = c.
20
Chương 2. Bất đẳng thức có tổng các biến không đổi
2.1.3 Sử dụng các tính chất của hàm số
Trong các bài toán chứng minh bất đẳng thức dạng P(x, y, z) ≥ 0(≤ 0), dựa
vào tổng không đổi và dựa vào dự đoán dấu đẳng thức xảy ra khi nào. Ta có
thể đưa bài toán về dạng hàm một biến và sử dụng các tính chất của hàm số để
khảo sát. Hoặc sử dụng bất đẳng thức Karamata và bổ đề cơ bản.
Bài toán 2.12. Với x, y, z > 0 thỏa mãn x + y + z = 1. Chứng minh rằng
1
xz
+
1
yz
≥ 16.
Chứng minh.
Yêu cầu bài toán ⇔
1
z
1
x
+
1
y
≥ 16.
Theo bất đẳng thức AM-GM, ta có
1
x
+
1
y
≥
4
x + y
=
4
1 − z
.
Ta đi chứng minh
1
z
.
4
1 − z
≥ 16, z ∈ (0; 1).
Xét hàm số f(z) =
1
z
.
4
1 − z
− 16 =
4
−z2 + z
− 16, với z ∈ (0; 1)
f (z) =
−4(−2z + 1)
(−z2 + z)2
.
f (z) = 0 ⇔ z =
1
2
.
Ta lập bảng biến thiên
z 0 1
2 1
f (z) − 0 +
f(z)
+∞
0
+∞
Từ bảng biến thiên, ta có f(z) ≥ 0, với ∀z ∈ (0; 1).
Đẳng thức xảy ra khi và chỉ khi x = y =
1
4
, z =
1
2
.
Bài toán 2.13 (Tiệp Khắc MO ,1984). Với x, y, z ≥ 0 thỏa mãn x + y + z = 1.
Chứng minh rằng
xy + yz + zx − 2xyz ≤
7
27
.
21
Chương 2. Bất đẳng thức có tổng các biến không đổi
Chứng minh.
Đặt xy + yz + zx − 2xyz = P.
Giả sử
x = min{x, y, z}.
Từ giả thiết x, y, z ≥ 0, x + y + z = 1 ⇒ 0 ≤ x ≤
1
3
.
Theo bất đẳng thức AM-GM, ta có
1 − x = y + z ≥ 2
√
yz ⇒
(1 − x)2
4
≥ yz.
Do đó
P = xy + yz + zx − 2xyz
= (xy + zx) + (yz − 2xyz)
= x(y + z) + yz(1 − 2x)
≤ x(1 − x) +
(1 − x)2
4
(1 − 2x)
=
−2x3 + x2 + 1
4
= f(x).
Xét hàm số f(x) =
−2x3 + x2 + 1
4
với x ∈ 0;
1
3
,
f (x) =
−6x2 + 2x
4
⇒f (x) = 0 ⇔
x = 0
x =
1
3
.
Ta có bảng biến thiên
x 0 1
3
f (x) 0 + 0
f(a) 1
4
7
27
Từ bảng biên thiên ta thấy với ∀x ∈ 0;
1
3
, ta có
P ≤ f(x) ≤
7
27
.
Đẳng thức xảy ra khi và chỉ khi x = y = z =
1
3
.
22
Chương 2. Bất đẳng thức có tổng các biến không đổi
Bài toán 2.14. Với x, y, z ≥ 0 thỏa mãn x + y + z = 1. Chứng minh rằng
xy + yz + zx ≤
1
4
+
9
4
xyz.
Chứng minh.
Giả sử z = min{x, y, z} ⇒ 0 ≤ z ≤
1
3
.
Có x + y + z = 1 ⇒ 1 − z = x + y.
Theo bất đẳng thức AM-GM, ta có
(x + y)2
4
≥ xy.
Bất đẳng thức cần chứng minh tương đương với xy + yz + zx −
9
4
xyz ≤
1
4
.
Ta có
xy + yz + zx −
9
4
xyz = xy + z(x + y) −
9
4
xyz
= xy(1 −
9
4
z) + z(x + y)
≤
(x + y)2
4
(1 −
9
4
z) + z(1 − z)
=
(1 − z)2
4
(1 −
9z
4
) + z − z2
=
1
16
(−9z3
+ 6z2
− z + 4).
Xét hàm f(z) = −9z3 + 6z2 − z + 4, với z ∈ 1;
1
3
.
Ta có f (z) = −27z2 + 12z − 1 ⇒ f (z) = 0 ⇔


z =
1
3
z =
1
9
.
Ta lập bảng biến thiên
z 0 1
9
1
3
f (z) | − 0 + 0
f(z)
4
320
81
4
Từ bảng biến thiên ta thấy f(z) ≤ 4, ∀z ∈ 0;
1
3
.
Vậy xy + yz + zx ≤
1
4
+
9
4
xyz.
Đẳng thức xảy ra khi và chỉ khi


z = 0, x = y =
1
2
x = y = z =
1
3
.
23
Chương 2. Bất đẳng thức có tổng các biến không đổi
Bài toán 2.15. Với x, y, z > 0 và x + y + z = 1. Chứng minh rằng
2(x3
+ y3
+ z3
) + 3(x2
+ y2
+ z2
) + 12xyz ≥
5
3
.
Chứng minh.
Giả sử x = min{x, y, z} ⇒ x ∈ 0;
1
3
.
Từ giả thiết ta có y + z = 1 − x và theo bất đẳng thức AM-GM, ta có
yz ≤
(y + z)2
4
.
Đặt 2(x3 + y3 + z3) + 3(x2 + y2 + z2) + 12xyz = P.
Khi đó, ta có
P = 2 x3
+ (y + z)3
− 3yz(y + z) + 3 x2
+ (y + z)2
− 2yz + 12xyz
= 2x3
+ 2(1 − x)3
− 6yz(1 − x) + 3x2
+ 3(1 − x)2
− 6yz + 12xyz
= 12x2
− 12x + 5 − 6yz(2 − 3x)
≥ 12x2
− 12x + 5 − 6
(y + z)2
4
(2 − 3x)
=
9
2
x3
−
3
2
x + 2.
Đặt
9
2
x3 −
3
2
x + 2 = f(x) với x ∈ 0;
1
3
.
Ta có f (x) =
27
2
x2 −
3
2
⇒ f (x) = 0 ⇔ x =
1
3
.
Ta có bảng biến thiên
x 0 1
3
f (x) | − 0
f(x)
2
5
3
Từ bảng biến thiên ta suy ra
P ≥ f(x) ≥
5
3
với x ∈ 0;
1
3
.
Đẳng thức xảy ra khi và chỉ khi x = y = z =
1
3
.
Bài toán 2.16. Với x, y, z ∈ [0; 2] thỏa mãn điều kiện x + y + z = 3. Chứng minh
rằng
x3
+ y3
+ z3
≤ 9.
24
Chương 2. Bất đẳng thức có tổng các biến không đổi
Chứng minh.
Giả sử 2 ≥ x ≥ y ≥ z ≥ 0 . Khi đó



2 ≥ x
2 + 1 ≥ x + y
2 + 1 + 0 = x + y + z
Xét hàm số f(t) = t3, với t ∈ [0; 2]
f (t) = 3t2
f (t) = 6t ≥ 0, với t ∈ [0; 2].
Do đó với x, y, z ∈ [0; 2], theo bất đẳng thức Karamata ta có
f(2) + f(1) + f(0) ≥ f(x) + f(y) + f(z)
⇔x3
+ y3
+ z3
≤ 9.
Đẳng thức xảy ra khi và chỉ khi x = 2; y = 1; z = 0 và các hoán vị của chúng.
Bài toán 2.17. Cho x, y, z ∈ [1; 3] , x + y + z = 6. Chứng minh rằng
x2
+ y2
+ z2
≤ 14.
Chứng minh.
Giả sử 3 ≥ x ≥ y ≥ z ≥ −1 . Khi đó



3 ≥ x
3 + 2 ≥ x + y
3 + 2 + 1 = x + y + z
Xét hàm số f(t) = t2, với t ∈ [1; 3]
f (t) = 2t
f (t) = 2 ≥ 0, với t ∈ [1; 3].
Do đó với x, y, z ∈ [1; 3], theo bất đẳng thức Karamata ta có
f(3) + f(2) + f(1) ≥ f(x) + f(y) + f(z)
⇔x2
+ y2
+ z2
≤ 14.
Đẳng thức xảy ra khi và chỉ khi x = 3; y = 2; z = 1 và các hoán vị của chúng.
25
Chương 2. Bất đẳng thức có tổng các biến không đổi
Bài toán 2.18. Với x, y, z là các số thực không âm và thỏa mãn x + y + z = 1.
Chứng minh rằng
x
x + 1
+
y
y + 1
+
z
z + 1
≤
3
4
.
Chứng minh.
Xét hàm số f(t) =
t
t + 1
, t ∈ [0; 1] .
Ta có f (t) =
1
(t + 1)2
⇒ f (t) =
−2
(t + 1)3
< 0, ∀x ∈ [0; 1] .
Áp dụng bổ đề 1.1,ta có
f(x) =
x
1 + x
≤ f
1
3
x −
1
3
+ f
1
3
f(y) =
y
1 + y
≤ f
1
3
y −
1
3
+ f
1
3
f(z) =
z
1 + z
≤ f
1
3
z −
1
3
+ f
1
3
.
Cộng các bất đẳng thức cùng chiều, ta có
x
x + 1
+
y
y + 1
+
z
z + 1
≤ 3f
1
3
(x + y + z − 1) + 3f
1
3
=
3
4
.
Đẳng thức xảy ra khi và chỉ khi x = y = z =
1
3
.
Bài toán 2.19. Với a, b, c là các số thực dương thỏa mãn a + b + c = 1. Chứng
minh rằng
a
a + bc
+
b
1 + ac
+
c
1 + ab
≥
9
10
.
Chứng minh.
Ta có
a
1 + bc
≥
a
1 +
(b + c)2
4
=
4a
4 + (1 − a)2
=
4a
a2 − 2a + 5
.
Tương tự, ta có
b
1 + ac
≥
4b
b2 − 2b + 5
c
1 + ab
≥
4c
c2 − 2c + 5
.
Cộng các bất đẳng thức cùng chiều, ta có
a
a + bc
+
b
1 + ac
+
c
1 + ab
≥
4a
a2 − 2a + 5
+
4b
b2 − 2b + 5
+
4c
c2 − 2c + 5
.
26
Chương 2. Bất đẳng thức có tổng các biến không đổi
Xét hàm số f(x) =
4x
x2 − 2x + 5
, với x ∈ (0; 1) .
Ta có f (x) =
−4x2 + 20
(x2 − 2x + 5)2
Ta đi chứng minh với ∀ ∈ (0; 1), ta luôn có
f(x) ≥ f (
1
3
) x −
1
3
+ f
1
3
⇔
4x
x2 − 2x + 5
≥
99
100
x −
1
3
+
3
10
⇔
−99x3 + 201x2 − 101x + 15
100(x2 − 2x + 5)
≥ 0
⇔
x −
1
3
2
(15 − 11x)
x2 − 2x + 5
≥ 0 luôn đúng ∀x ∈ (0; 1) .
Áp dụng bất đẳng thức trên với a, b, c ∈ (0; 1), ta có
f(a) ≥ f
1
3
a −
1
3
+ f
1
3
f(b) ≥ f
1
3
b −
1
3
+ f
1
3
f(c) ≥ f
1
3
c −
1
3
+ f
1
3
.
Cộng các bất đẳng thức cùng chiều, ta có
a
a + bc
+
b
1 + ac
+
c
1 + ab
≥
4a
a2 − 2a + 5
+
4b
b2 − 2b + 5
+
4c
c2 − 2c + 5
≥ f
1
3
(a + b + c − 1) + 3f
1
3
=
9
10
.
Đẳng thức xảy ra khi và chỉ khi a = b = c =
1
3
.
Bài toán 2.20 (Olympic 30-4-06). Với a, b, c là các số thực dương . Chứng minh
a(b + c)
a2 + (b + c)2
+
b(c + a)
b2 + (c + a)2
+
c(c + b)
c2 + (a + b)2)
≤
6
5
.
Chứng minh.
Bất đẳng thức cần chứng minh là một bất đẳng thức thuần nhất với ba biến
a, b, c Không mất tính tổng quát giả sử a + b + c = 1. Khi đó bất đẳng thức cần
chứng minh trở thành
a(1 − a)
a2 + (1 − a)2
+
b(1 − b)
b2 + (1 − b)2
+
c(1 − c)
c2 + (1 − c)2
≤
6
5
⇔f(a) + f(b) + f(c) ≤
6
5
27
Chương 2. Bất đẳng thức có tổng các biến không đổi
với f(x) =
x(1 − x)
x2 + (1 − x)2
=
−x2 + x
2x2 − 2x + 1
, x ∈ (0; 1),
f (x) =
−2x + 1
(2x2 − 2x + 1)2
⇒ f (
1
3
) =
27
25
.
Ta đi chứng minh với ∀x ∈ (0; 1), ta có
f(x) ≤ f (
1
3
)(x −
1
3
) + f(
1
3
)
⇔
−x2 + x
2x2 − 2x + 1
≤
27
25
(x −
1
3
) +
2
5
⇔
−54x3 + 27x2 − 1
25(2x2 − 2x + 1)
≤ 0
⇔
−54(x +
1
6
)(x −
1
3
)2
25(2x2 − 2x + 1)
≤ 0 luôn đúng ∀x ∈ (0; 1) .
Do đó với a, b, c ∈ (0; 1), ta có
f(a) + f(b) + f(c) ≤ f (
1
3
)(a + b + c − 3
1
3
) + 3f(
1
3
) =
6
5
.
Đẳng thức xảy ra khi và chỉ khi a = b = c =
1
3
.
Bài toán 2.21 (USA MO -2003). Với các số thực dương a, b, c. Chứng minh
rằng
(2a + b + c)2
2a2 + (b + c)2
+
(2b + a + c)2
2b2 + (c + a)2
+
(2c + a + b)2
2c2 + (a + b)2
≤ 8.
Chứng minh.
Giả sử a + b + c = 1. Khi đó bất đẳng thức cần chứng minh trở thành
(a + 1)2
2a2 + (1 − a)2
+
(b + 1)2
2b2 + (1 − b)2
+
(c + 1)2
2c2 + (1 − c)2
≤ 8
⇔
a2 + 2a + 1
3a2 − 2a + 1
+
b2 + 2b + 1
3b2 − 2b + 1
+
c2 + 2c + 1
3c2 − 2c + 1
≤ 8
hay
f(a) + f(b) + f(c) ≤ 8
với f(x) =
x2 + 2x + 1
3x2 − 2x + 1
, x ∈ (0; 1)
f (x) =
−8x2 − 4x + 4
(3x2 − 2x + 1)2
.
28
Chương 2. Bất đẳng thức có tổng các biến không đổi
Ta chứng minh với ∀x ∈ (0; 1), ta có
f(x) ≤ f (
1
3
)(x −
1
3
) + f(
1
3
)
⇔
x2 + 2x + 1
3x2 − 2x + 1
≤ 4(x −
1
3
) +
8
3
⇔
x2 + 2x + 1
3x2 − 2x + 1
−
12x + 4
3
≤ 0
⇔
−36x3 + 15x2 + 2x − 1
3(3x2 − 2x + 1)
≤ 0
⇔
−36(x +
1
4
)(x −
1
3
)2
3(3x2 − 2x + 1)
≤ 0, luôn đúng ∀x ∈ (0; 1) .
Từ đó, ta có
f(a) ≤ f (
1
3
)(a −
1
3
) + f(
1
3
)
f(b) ≤ f (
1
3
)(b −
1
3
) + f(
1
3
)
f(c) ≤ f (
1
3
)(c −
1
3
) + f(
1
3
).
Cộng các bất đẳng thức cùng chiều ta có
f(a) + f(b) + f(c) ≤ f (
1
3
)(a + b + c − 1) + 3f(
1
3
) = 8.
Đẳng thức xảy ra khi và chỉ khi a = b = c =
1
3
.
Bài toán 2.22 (Olympic Balkan 1996 và Olympic 30-4 -1999). Với các số thực
a, b, c thỏa mãn a + b + c = 1. Chứng minh rằng
a
1 + a2
+
b
1 + b2
+
c
1 + c2
≤
9
10
.
Chứng minh.
Đặt f(x) =
x
1 + x2
. Khi đó bất đẳng thức đã cho trở thành
f(a) + f(b) + f(c) ≤
9
10
.
Ta có f (x) =
1 − x2
(1 + x2)2
⇒ f (x) = 0 ⇔
x = −1
x = 1
29
Chương 2. Bất đẳng thức có tổng các biến không đổi
Ta có bảng biến thiên ( đưa thêm một số giá trị như x = −3; x =
−1
3
; x = 2
và giá trị của hàm số tại đó để so sánh)
x −∞ −3 −1
−1
3
1 2 +∞
f (x) − 0 + 0 +
f(x)
0 −3
10
−1
2
−3
10
1
2 2
5 0
Ta xét các trường hợp sau:
+ Trường hợp 1. Trong ba số a, b, c có một số thuộc (−∞; −3].
Giả sử a ∈ (−∞; −3] ⇒ b + c ≥ 4, giả sử b ≥ 2.
Khi đó từ bảng biến thiên, ta có
f(a) + f(b) + f(c) < 0 +
1
2
+
2
5
=
9
10
+ Trường hợp 2. Có một số, giả sử a ∈ −3;
−1
3
. Khi đó có
f(a) + f(b) + f(c) ≤
−3
10
+
1
2
+
1
2
=
7
10
<
9
10
.
+ Trường hợp 3. Cả ba số a, b, c ∈
−1
3
; +∞ . Khi đó tiếp tuyến của hàm số
y = f(x) tại x =
1
3
có phương trình
y = f
1
3
x −
1
3
+ f
1
3
hay
y =
18
25
x +
3
50
.
Ta chứng minh
f(x) ≤
18
25
x +
3
50
⇔
x
1 + x2
−
18
25
x −
3
50
≤ 0
⇔
−36x3 − 3x2 + 14x − 3
50(1 + x2)
≤ 0
⇔
−36(x +
3
4
(x −
1
3
)2)
50(1 + x2)
≤ 0 luôn đúng với ∀x ≥
−1
3
.
Do đó f(a) + f(b) + f(c) ≤
18
25
(a + b + c) + 3
3
50
=
9
10
.
Đẳng thức xảy ra khi và chỉ khi a = b = c =
1
3
.
30
Chương 2. Bất đẳng thức có tổng các biến không đổi
2.1.4 Bài toán liên quan
Bài toán 2.23. Với các số dương x, y, z thỏa mãn điều kiện x+y +z = 1. Chứng
minh
3
xy + yz + zx
+
2
x2 + y2 + z2
≥ 14.
Bài toán 2.24. Với a, b, c > 0 thỏa mãn a + b + c = 3. Chứng minh rằng
a + 1
b2 + 1
+
b + 1
c2 + 1
+
c + 1
a2 + 1
≥ 3.
Bài toán 2.25. Với a, b, c > 0 và thỏa mãn a + b + c = 3. Chứng minh rằng
a
b3 + ab
+
b
c3 + ca
+
c
a3 + ca
≥
3
2
.
Bài toán 2.26. Với a, b, c là các số thực dương thỏa mãn a + b + c = 3. Chứng
minh bất đẳng thức sau
ab + bc + ca +
1
abc
≥ abc + 3.
Bài toán 2.27. Với a, b, c là các số thực dương thỏa mãn a + b + c = 3. Chứng
minh rằng
a3
b(2c + a)
+
b3
c(2a + b)
+
c3
a(2b + c)
≥ 1.
Bài toán 2.28. Với a, b, c ≥ 0 thỏa mãn a + b + c = 1. Chứng minh
a2
+ b2
+ c2
+ 3abc ≥
4
9
.
Bài toán 2.29. Với a, b, c là độ dài ba cạnh của tam giác. Chứng minh rằng
1
a
+
1
b
+
1
c
+
9
a + b + c
≥ 4
1
a + b
+
1
b + c
+
1
c + a
.
Bài toán 2.30. Với a, b, c là các số thực dương . Chứng minh rằng
a
(b + c)2
+
b
(c + a)2
+
c
(a + b)2
≥
9
4(a + b + c)
.
Bài toán 2.31 (Japan Mathematical Olympiad Finals 1997). Với a, b, c là các
số thực dương . Chứng minh rằng
(b + c − a)2
(b + c)2 + a2
+
(c + a − b)2
(c + a)2 + b2
+
(a + b − c)2
(a + b)2 + c2
≥
3
5
.
Bài toán 2.32 (China MO 2005). Với a, b, c không âm thỏa mãn a + b + c = 1.
Chứng minh rằng
10(a3
+ b3
+ c3
) − 9(a5
+ b5
+ c5
) ≥ 1.
31
Chương 2. Bất đẳng thức có tổng các biến không đổi
Bài toán 2.33 (China Northern MO 2006). Với các số thực a, b, c thỏa mãn
a + b + c = 3. Chứng minh rằng
a2 + 9
2a2 + (b + c)2
+
b2 + 9
2b2 + (a + c)2
+
c2 + 9
2c2 + (a + b)2
≤ 5.
Bài toán 2.34. Với a, b, c là các số thực thỏa mãn điều kiện a+b+c = 3. Chứng
minh rằng
18a − 35
a2 − 4a + 6
+
18b − 35
b2 − 4b + 6
+
18c − 35
c2 − 4c + 6
≥ −17.
Bài toán 2.35. Với a, b, c là các số thực thỏa mãn a + b + c = 1. Chứng minh
rằng
a2
6a2 − 4a + 1
+
b2
6b2 − 4b + 1
+
c2
6c2 − 4c + 1
≤ 1.
Bài toán 2.36. Với a, b, c là các số thực thỏa mãn điều kiện a+b+c = 1. Chứng
minh rằng
a2
3a2 − 2a + 3
+
b2
3b2 − 2b + 3
+
c2
3c2 − 2c + 3
≥
1
8
.
32
Chương 2. Bất đẳng thức có tổng các biến không đổi
2.2 Bất đẳng thức có tổng các biến không đổi với hàm vô tỉ
2.2.1 Sử dụng bất đẳng thức AM-GM
Trong phần này ta đi chứng minh các bất đẳng thức với hàm vô tỉ với tổng
không đổi bằng bất đẳng thức AM-GM. Điều quan trọng là ta vận dụng bất
đẳng thức AM-GM khéo léo để phá được dấu căn và tận dụng tổng không đổi.
Bài toán 2.37. Với x, y, z là các số thực không âm thỏa mãn x + y + z = 2.
Chứng minh
x3y + y3z + z3x + xy3 + yz3 + zx3 ≤ 2.
Chứng minh.
Trước hết ta chứng minh bất đẳng thức
√
a +
√
b ≤ 2(a + b) (∗) với a, b ≥ 0.
Thật vậy, bình phương hai vế của bất đẳng thức (*), ta có
a + 2
√
ab + b ≤ 2(a + b) ⇔ a + b ≥ 2
√
ab luôn đúng theo AM-GM .
Áp dụng bất đẳng thức (* ) ta có
x3y + y3z + z3x+ xy3 + yz3 + zx3 ≤ 2[xy(x2 + y2) + yz(y2 + z2) + zx(z2 + x2)].
Yêu cầu của bài toán được thỏa mãn nếu chứng minh được
xy(x2
+ y2
) + yz(y2
+ z2
) + zx(z2
+ x2
) ≤ 2.
Ta có
xy(x2
+ y2
) + yz(y2
+ z2
) + zx(z2
+ x2
)
≤xy(x2
+ y2
+ z2
) + yz(x2
+ y2
+ z2
) + zx(x2
+ y2
+ z2
)
=(xy + yz + zx)(x2
+ y2
+ z2
).
Đến đây ta cần chứng minh
(xy + yz + zx)(x2
+ y2
+ z2
) ≤ 2.
Áp dụng bất đẳng thức AM-GM, ta có
(xy + yz + zx)(x2
+ y2
+ z2
) =
1
2
2(xy + yz + zx)(x2
+ y2
+ z2
)
≤
1
2
(2xy + 2yz + 2zx + x2 + y2 + z2)2
4
=
1
8
(x + y + z)4
= 2.
Đẳng thức xảy ra khi và chỉ khi x = y = 1, z = 0 hoặc các hoán vị của chúng.
33
Chương 2. Bất đẳng thức có tổng các biến không đổi
Bài toán 2.38. Với a, b, c là các số thực dương thỏa mãn a + b + c = 3. Chứng
minh rằng
a(b + 3c) + b(c + 3a) + c(a + 3b) ≤ 6.
Chứng minh. Áp dụng bất đẳng thức AM-GM, ta có
a(b + 3c) =
1
2
4a(b + 3c) ≤
1
2
4a + b + 3c
2
.
Tương tự
b(c + 3a) ≤
1
2
4b + c + 3a
2
c(a + 3b) ≤
1
2
4c + a + 3b
2
.
Cộng các bất đẳng thức cùng chiều, ta được
a(b + 3c) + b(c + 3a) + c(a + 3b) ≤
1
2
8(a + b + c)
2
= 6.
Đẳng thức xảy ra khi và chỉ khi a = b = c = 1.
Bài toán 2.39. [Russian MO 2002]Với a, b, c > 0 thỏa mãn a + b + c = 3. Chứng
minh rằng
√
a +
√
b +
√
c ≥ ab + bc + ca.
Chứng minh.
Bất đẳng thức cần chứng minh tương đương với
a2
+ b2
+ c2
+ 2(
√
a +
√
b +
√
c) ≥ 2(ab + bc + ca) + a2
+ b2
+ c2
⇔a2
+ b2
+ c2
+ 2(
√
a +
√
b +
√
c) ≥ (a + b + c)2
= 9.
Áp dụng bất đẳng thức AM-GM, ta có
a2
+
√
a +
√
a ≥ 3a
b2
+
√
b +
√
b ≥ 3b
c2
+
√
c +
√
c ≥ 3c.
Cộng các bất đẳng thức cùng chiều, ta có
a2
+ b2
+ c2
+ 2
√
a + 2
√
b + 2
√
c ≥ 3(a + b + c) = 9.
Đẳng thức xảy ra khi và chỉ khi a = b = c = 1.
34
Chương 2. Bất đẳng thức có tổng các biến không đổi
Bài toán 2.40 (Mexico MO 2007). Nếu a, b, c là các số thực dương thỏa mãn
a + b + c = 1. Chứng minh rằng
√
a + bc +
√
b + ca +
√
c + ab ≤ 2.
Chứng minh.
Sử dụng điều kiện a + b + c = 1, ta có
a + bc = a(a + b + c) + bc = (a + b)(a + c).
Theo bất đẳng thức AM-GM, ta có
√
a + bc = (a + b)(a + c) ≤
2a + b + c
2
.
Tương tự
√
b + ac ≤
2b + a + c
2
√
c + ab ≤
2c + a + b
2
.
Cộng các bất đẳng thức cùng chiều, ta có
√
a + bc +
√
b + ca +
√
c + ab ≤
4a + 4b + 4c
2
= 2(a + b + c) = 2.
Đẳng thức xảy ra khi và chỉ khi a = b = c =
1
3
.
Bài toán 2.41. Với các số thực dương thỏa mãn a+b+c = 3. Chứng minh rằng
ab
√
c2 + 3
+
bc
√
a2 + 3
+
ca
√
b2 + 3
≤
3
2
.
Chứng minh.
Ta luôn có
(a + b + c)2
3
≥ ab + bc + ca
⇔ 3 ≥ ab + bc + ca.
Suy ra
ab
√
c2 + 3
≤
ab
√
c2 + ab + bc + ca
=
ab
(a + b) (a + c)
.
Từ đó áp dụng bất đẳng thức AM-GM, ta có
ab
(b + c) (c + a)
≤
1
2
ab
b + c
+
ab
c + a
.
35
Chương 2. Bất đẳng thức có tổng các biến không đổi
Chứng minh tương tự
bc
√
a2 + 3
≤
bc
(a + c) (a + b)
≤
1
2
bc
c + a
+
bc
a + b
ca
√
b2 + 3
≤
ca
(b + a) (b + c)
≤
1
2
ca
b + a
+
ca
b + c
.
Cộng các bất đẳng thức cùng chiều, ta có
ab
√
c2 + 3
+
bc
√
a2 + 3
+
ca
√
b2 + 3
≤
1
2
ab
b + c
+
ca
b + c
+
ab
c + a
+
bc
c + a
+
bc
a + b
+
ca
a + b
=
a + b + c
3
=
3
2
.
Đẳng thức xảy ra khi và chỉ khi a = b = c = 1.
2.2.2 Sử dụng bất đẳng thức Cauchy-Schwarz
Bài toán 2.42 (Bosnia and Hercegovina MO 2005). Với các số thực dương a, b, c
thỏa mãn a + b + c = 1. Chứng minh rằng
a
√
b + b
√
c + c
√
a ≤
1
√
3
.
Chứng minh.
Sử dụng bất đẳng thức quen thuộc
(a + b + c)2
≥ 3(ab + bc + ca)
và theo giả thiết
a + b + c = 1.
Khi đó ta có
ab + bc + ca ≤
1
3
.
Áp dụng bất đẳng thức Cauchy- Schwarz, ta có
a
√
b + b
√
c + c
√
a
2
=
√
a
√
ab +
√
b
√
bc +
√
c
√
ca
2
≤ (a + b + c) (ab + bc + ca)
≤
1
3
.
36
Chương 2. Bất đẳng thức có tổng các biến không đổi
Vậy bất đẳng thức được chứng minh.
Đẳng thức xảy ra khi và chỉ khi a = b = c =
1
3
.
Bài toán 2.43. Với a, b, c là các số thực dương và a + b + c = 1. Chứng minh
rằng
√
a + bc +
√
b + ca +
√
c + ab ≥
√
ab +
√
bc +
√
ca + 1.
Chứng minh.
Sử dụng bất đẳng thức Cauchy -Schwarz, ta có
√
a + bc +
√
b + ca +
√
c + ab
= a(a + b + c) + bc + b(a + b + c) + ca + c(a + b + c) + ab
= (a + c)(a + b) + (b + a)(b + c) + (c + b)(c + a)
≥a +
√
bc + b +
√
ac + c +
√
ab
=1 +
√
ab +
√
bc +
√
ca.
Vậy bất đẳng thức được chứng minh.
Đẳng thức xảy ra khi và chỉ khi a = b = c =
1
3
.
Nhận xét 2.2. Bài toán trên được xây dựng từ bất đẳng thức APMO 2002 như
sau
Với x, y, z > 0 thỏa mãn
1
x
+
1
y
+
1
z
= 1. Chứng minh rằng
√
x + yz +
√
y + zx +
√
z + xy ≥
√
xyz +
√
x +
√
y +
√
z.
Bài toán 2.44. Với các số thực dương a, b, c thỏa mãn a+b+c = 1. Chứng minh
rằng
ab(1 − c) + bc(1 − a) + ca(1 − b) ≤
2
3
.
Chứng minh.
Theo hệ quả của bất đẳng thức AM-GM, ta có
(a + b + c)2
≥ 3(ab + bc + ca).
37
Chương 2. Bất đẳng thức có tổng các biến không đổi
Áp dụng bất đẳng thức Cauchy- Schwarz, ta có
ab(1 − c) + bc(1 − a) + ca(1 − b)
2
≤ (ab + bc + ca) (1 − c + 1 − a + 1 − b)
= 2(ab + bc + ca)
=
2
3
3(ab + bc + ca)
≤
2
3
(a + b + c)2
=
2
3
.
Do đó
ab(1 − c) + bc(1 − a) + ca(1 − b) ≤
2
3
.
Đẳng thức xảy ra khi và chỉ khi a = b = c =
1
3
.
Bài toán 2.45 (China MO 2008). Với các số thực dương a, b, c thỏa mãn a+b+c =
1. Chứng minh rằng
a +
(b − c)2
4
+
√
b +
√
c ≤
√
3
Chứng minh.
Đặt
√
bc = x suy ra
√
b +
√
c = b + c + 2
√
bc = 1 − a + 2
√
bc =
√
1 − a + 2x.
Và ta có
a +
(b − c)2
4
= a +
(b + c)2 − 4bc
4
= a +
(1 − a)2 − 4x2
4
=
1
2
(1 + a)2 − 4x2.
Khi đó bất đẳng thức ban đầu trở thành
(1 + a)2 − 4x2 + 2
√
1 − a + 2x ≤ 2
√
3.
Áp dụng bất đẳng thức Cauchy-Schwarz, ta có
(1 + a)2 − 4x2 + 2 (1 − a) + 2x ≤ 3 [(1 + a)2 − 4x2 + 2(1 − a + 2x)].
Vậy ta chỉ cần chứng minh
(1 + a)2
− 4x2
+ 2(1 − a + 2x) ≤ 4
⇔a2
− (2x − 1)2
≤ 0
⇔(a − 2x + 1)(a + 2x − 1) ≤ 0
⇔(1 − 2x − a)(1 + a − 2x) ≥ 0.
38
Chương 2. Bất đẳng thức có tổng các biến không đổi
Bất đẳng thức này hiển nhiên đúng vì
1 + a − 2x ≥ 1 − 2x − a = 1 − 2
√
bc − a ≥ 1 − (b + c) − a = 0.
Bất đẳng thức đã cho được chứng minh.
Đẳng thức xảy ra khi và chỉ khi a = b = c =
1
3
.
Bài toán 2.46 (China MO 2006). Với các số thực dương a, b, c thỏa mãn a+b+c =
1. Chứng minh rằng
ab
√
ab + bc
+
bc
√
bc + ca
+
ca
√
ca + ab
≤
√
2
2
.
Chứng minh.
Áp dụng bất đẳng thức Cauchy- Schwarz, ta có
ab
√
ab + bc
+
bc
√
bc + ca
+
ca
√
ca + ab
2
=
√
ab
√
ab + bc + ca
a(ab + bc + ca)
√
a + c
+
√
bc
√
ab + bc + ca
b(ab + bc + ca)
√
b + a
+
√
ca
√
ab + bc + ca
c(ab + bc + ca)
√
c + b
2
≤
ab + bc + ca
ab + bc + ca
a(ab + bc + ca)
a + c
+
b(ab + bc + ca)
b + a
+
c(ab + bc + ca)
c + b
= (ab + bc + ca)
a
a + c
+
b
b + a
+
c
c + b
.
Bất đẳng thức được chứng minh nếu ta chứng minh được
(ab + bc + ca)
a
a + c
+
b
b + a
+
c
c + b
≤
1
2
⇔2 (ab + bc + ca)
a
a + c
+
b
b + a
+
c
c + b
≤ (a + b + c)2
⇔2
a2b
a + c
+
ab2
b + a
+
abc
c + b
+
abc
a + c
+
b2c
b + a
+
bc2
c + b
+
ca2
a + c
+
abc
a + b
+
c2a
c + b
≤ (a + b + c)2
⇔2 ab + bc + ca +
ab2
a + b
+
bc2
b + c
+
ca2
a + c
≤ (a + b + c)2
⇔2
ab2
a + b
+
bc2
b + c
+
ca2
a + c
≤ a2
+ b2
+ c2
⇔2b2
−
2ab2
a + b
+ 2c2
−
2bc2
b + c
+ 2a2
−
2ca2
c + a
≥ a2
+ b2
+ c2
⇔
2b3
a + b
+
2c3
b + c
+
2a3
c + a
≥ a2
+ b2
+ c2
.
39
Chương 2. Bất đẳng thức có tổng các biến không đổi
Mặt khác theo bất đẳng thức Cauchy -Schwarz và bất đẳng thức AM-GM,
ta có
2
b3
a + b
+
c3
b + c
+
a3
c + a
≥
2(b2 + c2 + a2)2
b(a + b) + c(b + c) + a(c + a)
=
2(a2 + b2 + c2)2
a2 + b2 + c2 + ab + bc + ca
≥
(a2 + b2 + c2)2
a2 + b2 + c2 +
a2 + b2
2
+
b2 + c2
2
+
c2 + a2
2
=a2
+ b2
+ c2
.
Bất đẳng thức được chứng minh.
Đẳng thức xảy ra khi và chỉ khi a = b = c =
1
3
.
Bài toán 2.47. Với các số thực dương a, b, c thỏa mãn điều kiện a + b + c = 1.
Chứng minh rằng
a2
9a + 1
+
b2
9b + 1
+
c2
9c + 1
≤
1
12 3(ab + bc + ca)
.
Chứng minh.
Ta có
a2
9a + 1
=
a
9
−
a
9(9a + 1)
b2
9b + 1
=
b
9
−
b
9(9b + 1)
c2
9c + 1
=
c
9
−
c
9(9c + 1)
.
Khi đó, bất đẳng thức cần chứng minh trở thành
a
9a + 1
+
b
9b + 1
+
c
9c + 1
+
3
4 3(ab + bc + ca)
≥ 1.
Áp dụng bất đẳng thức Cauchy- Schwarz, ta có
a
9a + 1
+
b
9b + 1
+
c
9c + 1
≥
(a + b + c)2
a(9a + 1) + b(9b + 1) + c(9c + 1)
=
1
9(a2 + b2 + c2) + 1
.
40
Chương 2. Bất đẳng thức có tổng các biến không đổi
Theo bất đẳng thức AM-GM, ta có
3
4 3(ab + bc + ca)
≥
3
2 [1 + 3(ab + bc + ca)]
.
Do đó, bất đẳng thức được chứng minh nếu ta chứng minh được
1
9(a2 + b2 + c2) + 1
+
3
6(ab + bc + ca) + 2
≥ 1.
Áp dụng bất đẳng thức Cauchy- Schwarz, ta có
1
9(a2 + b2 + c2) + 1
+
3
6(ab + bc + ca) + 2
≥
(1 + 3)2
9(a2 + b2 + c2) + 18(ab + bc + ca) + 7
=
16
9(a + b + c)2 + 7
= 1.
Bất đẳng thức được chứng minh.
Đẳng thức xảy ra khi và chỉ khi a = b = c =
1
3
.
2.2.3 Sử dụng các tính chất của hàm số
Bài toán 2.48. Với các số thực dương a, b, c thỏa mãn a+b+c = 1. Chứng minh
rằng
a
√
a2 + 1
+
b
√
b2 + 1
+
c
√
c2 + 1
≤
3
√
10
.
Chứng minh.
Xét hàm số f(t) =
t
√
t2 + 1
, t ∈ (0; 1)
f (t) =
1
(t2 + 1)3
, f (t) = −
3t
(t2 + 1)5
< 0, ∀t ∈ (0; 1).
Theo bổ đề 1.1, ta có
f(a) ≤ f (
1
3
)(a −
1
3
) + f(
1
3
)
f(b) ≤ f (
1
3
)(b −
1
3
) + f(
1
3
)
f(c) ≤ f (
1
3
)(c −
1
3
) + f(
1
3
).
Cộng các bất đẳng thức cùng chiều, ta có
f(a) + f(b) + f(c) ≤ f (
1
3
)(a + b + c − 1) + 3f(
1
3
) =
3
√
10
.
Đẳng thức xảy ra khi và chỉ khi a = b = c =
1
3
.
41
Chương 2. Bất đẳng thức có tổng các biến không đổi
Bài toán 2.49. Với a, b, c là các số thực dương thỏa mãn a + b + c = 3. Chứng
minh rằng
1
1 + 8
√
a
+
1
1 + 8
√
b
+
1
1 + 8
√
b
≥ 1.
Chứng minh.
Xét hàm số f(t) =
1
1 + 8
√
t
, t ∈ (0; 3).
Ta có
f (t) =
−2
t(1 + 8
√
t)3
f (x) =
20
√
t + 1
(8
√
t + 1)5t3
> 0, ∀t ∈ (0; 3).
Với a, b, c ∈ (0; 3), theo bổ đề 1.1, ta có
f(a) ≥ f (1)(a − 1) + f(1)
f(b) ≥ f (1)(b − 1) + f(1)
f(c) ≥ f (1)(c − 1) + f(1).
Cộng các bất đẳng thức cùng chiều, ta có
f(a) + f(b) + f(c) ≥ f (1)(a + b + c − 3) + 3f(1) = 1.
Đẳng thức xay ra khi và chỉ khi a = b = c = 1.
Nhận xét 2.3. Bài toán có thể cho dưới hình thức sau:
Với các số thực dương a, b, c thỏa mãn a2 + b2 + c2 = 3. Chứng minh rằng
1
√
1 + 8a
+
1
√
1 + 8b
+
1
√
1 + 8c
≥ 1.
Bài toán 2.50 (Đề đại học khối A năm 2003). Với các số thực dương x, y, z
thỏa mãn x + y + z = 1. Chứng minh rằng
x2 +
1
x2
+ y2 +
1
y2
+ z2 +
1
z2
≥
√
82.
Chứng minh.
42
Chương 2. Bất đẳng thức có tổng các biến không đổi
Xét hàm số f(t) = t2 +
1
t2
, t ∈ (0; 1),
f (t) =
(t2 +
1
t2
)
2 t2 +
1
t2
=
t4 − 1
√
t8 + t4
f (t) =
4t3
√
t8 + t4 − (t4 − 1)
8t7 + 4t3
2
√
t8 + t4
t8 + t4
=
6t4 + 2
(t5 + t)
√
t8 + t4
> 0, ∀t ∈ (0; 1).
Áp dụng bổ đề 1.1, ta có
f(x) ≥ f (
1
3
)(x −
1
3
) + f(
1
3
)
f(y) ≥ f (
1
3
)(y −
1
3
) + f(
1
3
)
f(z) ≥ f (
1
3
)(z −
1
3
) + f(
1
3
).
Cộng các bất đẳng thức cùng chiều, ta có
x2 +
1
x2
+ y2 +
1
y2
+ z2 +
1
z2
≥ f (
1
3
)(x + y + z − 1) + 3f(
1
3
)
= 3f(
1
3
) =
√
82.
Đẳng thức xảy ra khi và chỉ khi x = y = z =
1
3
.
2.2.4 Bài toán liên quan
Bài toán 2.51. Với a, b, c không âm thỏa mãn a + b + c = 1. Chứng minh rằng
4
√
ab3 +
4
√
bc3 +
4
√
ca3 ≤ 1.
Bài toán 2.52. Với a, b, c > 0 thỏa mãn a + b + c = 3. Chứng minh rằng
a + b
c + ab
+
b + c
a + bc
+
c + a
b + ac
≥ 3.
43
Chương 2. Bất đẳng thức có tổng các biến không đổi
Bài toán 2.53. Với a, b, c > 0 thỏa mãn a + b + c = 3. Chứng minh rằng với
∀k > 0 thì
(b + c)
k bc + 1
a2 + 1
+ (c + a) k ca + 1
b2 + 1
+ (a + b)
k ab + 1
c2 + 1
≥ 6.
Bài toán 2.54 (Ukraine MO 2001). Với các số thực dương a, b, c, x, y, z thỏa
mãn x + y + z = 1. Chứng minh rằng
ax + by + cz + 2 (xy + yz + zx)(ab + bc + ca) ≤ a + b + c.
44
Chương 3
Bất đẳng thức có tích các biến
không đổi
Khi bất đẳng thức có dạng P(x, y, z) ≥ 0 hoặc(≤ 0) với tích xyz không đổi,
thì trong các kĩ thuật chứng minh bất đẳng thức bằng các sử dụng AM-GM,
bất đẳng thức Cauchy-Schwarz, các tính chất của hàm số thường dùng, ta có
thể sử dụng phép thế để đưa bất đẳng thức cần chứng minh về bất đẳng thức
đơn giản hơn.
Cho các số thực dương a, b, c. Nếu abc = 1 thì có thể đặt a =
1
x
, b =
1
y
, c =
1
z
hoặc a =
x
y
, b =
y
z
, c =
z
x
hoặc a =
yz
x2
, b =
xz
y2
, c =
xy
z2
, . . .
3.1 Bất đẳng thức có tích các biến không đổi với hàm phân
thức hữu tỉ
3.1.1 Sử dụng bất đẳng thức AM-GM
Bài toán 3.1. Với các số dương x, y, z thỏa mãn điều kiện xyz = 1. Chứng minh
rằng với ∀n ∈ N∗, ta luôn có
xn
y + yn
z + zn
x ≥ xy + yz + zx.
Chứng minh.
Áp dụng bất đẳng thức AM-GM cho (n+1) số, ta có
xn
y + xn
y + yn
z + yz + yz + ... + yz
n−2
≥ (n + 1) n+1
(xny)2ynz(yz)n−2
= (n + 1) n+1
x2ny2nzn−1
= (n + 1) n+1
(xy)2n
1
(xy)n−1
= (n + 1)xy.
45
Chương 3. Bất đẳng thức có tích các biến không đổi
Tương tự, ta có
yn
z + yn
z + zn
x + zx + zx + ... + zx
n−2
≥ (n + 1) n+1
(ynz)2znx(zx)n−2
= (n + 1)yz
zn
x + zn
x + xn
y + xy + xy + ... + xy
n−2
≥ (n + 1) n+1
(znx)2xny(xy)n−2
= (n + 1)zx.
Cộng các bất đẳng thức cùng chiều trên, ta có
3(xn
y + yn
z + zn
x) + (n − 2)(xy + yz + zx) ≥ (n + 1)(xy + yz + zx)
⇔ xn
y + yn
z + zn
x ≥ xy + yz + zx.
Đẳng thức xảy ta khi và chỉ khi x = y = z = 1.
Bài toán 3.2. Với các số thực dương x, y, z có xyz = 1. Chứng minh rằng
x4y
x2 + 1
+
y4z
y2 + 1
+
z4x
z2 + 1
≥
3
2
.
Chứng minh.
Sử dụng bất đẳng thức AM-GM, ta có
x4y
x2 + 1
= x2
y −
x2y
x2 + 1
≥ x2
y −
x2y
2x
= x2
y −
xy
2
.
Chứng minh tương tự, ta có
y4z
y2 + 1
≥ y2
z −
yz
2
z4x
z2 + 1
≥ z2
x −
zx
2
.
Cộng các bất đẳng thức cùng chiều chúng ta có
x4y
x2 + 1
+
y4z
y2 + 1
+
z4x
z2 + 1
≥ x2
y + y2
z + z2
x −
1
2
(xy + yz + zx) .
46
Chương 3. Bất đẳng thức có tích các biến không đổi
Bất đẳng thức được chứng minh nếu ta chứng minh được
x2
y + y2
z + z2
x ≥
3
2
+
xy + yz + zx
2
.
Theo bất đẳng thức AM-GM, ta có
x2y + y2z + z2x
2
≥
3 3
x3y3z3
2
=
3
2
.
Vậy bất đẳng thức được chứng minh hoàn tất nếu ta chứng minh được
x2y + y2z + z2x
2
≥ xy + yz + zx.
Thật vậy sử dụng bất đẳng thức AM-GM, ta có
x2
y + x2
y + y2
z ≥ 3 3
x2y.x2y.y2z = 3 3
x4y4
1
xy
= 3xy
y2
z + y2
z + z2
x ≥ 3 3
y2z.y2z.z2x = 3 3
y4z4
1
yz
= 3yz
z2
z + z2
x + x2
y ≥ 3 3
z2x.z2x.x2y = 3
3
x4z4
1
xz
= 3zx.
Cộng các bất đẳng thức trên và rút gọn ta có điều phải chứng minh.
Đẳng thức xảy ra khi và chỉ khi x = y = z = 1.
Bài toán 3.3 (IMO 1995). Với a, b, c là các số thực dương và abc = 1. Chứng
minh rằng
1
a3(b + c)
+
1
b3(c + a)
+
1
c3(a + b)
≥
3
2
.
Chứng minh.
Theo bất đẳng thức AM-GM, ta có
1
a3(b + c)
+
b + c
4bc
≥ 2
b + c
4a3bc(b + c)
=
1
a
1
b3(a + c)
+
a + c
4ac
≥ 2
a + c
4b3ac(a + c)
=
1
b
1
c3(a + b)
+
a + b
4ab
≥ 2
a + b
4c3ab(a + b)
=
1
c
.
Cộng các bất đẳng thức cùng chiều, ta có
1
a3(b + c)
+
1
b3(c + a)
+
1
c3(a + b)
+
1
4
b + c
bc
+
a + c
ac
+
a + b
ab
≥
1
a
+
1
b
+
1
c
⇔
1
a3(b + c)
+
1
b3(c + a)
+
1
c3(a + b)
≥
1
2
1
a
+
1
b
+
1
c
≥
3
2
3 1
abc
=
3
2
.
Đẳng thức xảy ra khi và chỉ khi a = b = c = 1.
47
Chương 3. Bất đẳng thức có tích các biến không đổi
Bài toán 3.4 (Romania MO 1997 ). Với các số thực dương x, y, z thỏa mãn
xyz = 1. Chứng minh rằng
x9 + y9
x6 + x3y3 + y6
+
y9 + z9
y6 + y3z3 + z6
+
z9 + x9
z6 + z3x3 + x6
≥ 2.
Chứng minh.
Đặt a = x3, b = y3, c = z3. Khi đó bất đẳng thức cần chứng minh trở thành
a3 + b3
a2 + ab + b2
+
b3 + c3
b2 + bc + c2
+
c3 + a3
c2 + ca + a2
≥ 2.
Mặt khác, với ∀u, v > 0, ta có
u3 + v3
u2 + uv + v2
−
u + v
3
=
2(u + v)(u − v)2
3(u2 + uv + v2)
≥ 0 luôn đúng .
Do đó
a3 + b3
a2 + ab + b2
+
b3 + c3
b2 + bc + c2
+
c3 + a3
c2 + ca + a2
≥
a + b
3
+
b + c
3
+
c + a
3
≥
2
3
3
3
√
abc = 2.
Đẳng thức xảy ra khi và chỉ khi a = b = c hay x = y = z = 1.
Bài toán 3.5 (Korea MO 1999). Với các số thực dương a, b, c thỏa mãn abc = 1.
Chứng minh rằng
1
a + b4 + c4
+
1
a4 + b + c4
+
1
a4 + b4 + c
≤ 1.
Chứng minh.
Áp dụng bất đẳng thức AM-GM, ta có
b4
+ c4
=
3b4 + c4
4
+
b4 + 3c4
4
≥
4
4
√
b4b4b4c4
4
+
4
4
√
b4c4c4c4
4
= bc(b2
+ c2
)
≥
b2 + c2
a
.
Do đó
1
a + b4 + c4
≤
1
a +
b2 + c2
a
=
a
a2 + b2 + c2
.
Chứng minh tương tự, ta có
1
a4 + b + c4
≤
b
a2 + b2 + c2
48
Chương 3. Bất đẳng thức có tích các biến không đổi
1
a4 + b4 + c
≤
c
a2 + b2 + c2
.
Cộng các bất đẳng thức cùng chiều, ta có
1
a + b4 + c4
+
1
a4 + b + c4
+
1
a4 + b4 + c
≤
a + b + c
a2 + b2 + c2
≤
a + b + c
(a + b + c)2
3
=
3
a + b + c
≤
3
3 3
√
abc
≤ 1.
Đẳng thức xảy ra khi và chỉ khi a = b = c = 1.
Bài toán 3.6 (Hongkong 2000). Với các số thực dương a, b, c thỏa mãn điều
kiện abc = 1. Chứng minh rằng
1 + ab2
c3
+
1 + bc2
a3
+
1 + ca2
b3
≥
18
a3 + b3 + c3
.
Chứng minh.
Bất đẳng thức cần chứng minh tương đương với
1
a3
+
1
b3
+
1
c3
+
bc2
a3
+
ca2
b3
+
ab2
c3
≥
18
a3 + b3 + c3
⇔(a3
+ b3
+ c3
)(
1
a3
+
1
b3
+
1
c3
) + (a3
+ b3
+ c3
)
bc2
a3
+
ca2
b3
+
ab2
c3
≥ 18.
Áp dụng bất đẳng thức AM-GM, ta có
a3
+ b3
+ c3 1
a3
+
1
b3
+
1
c3
≥ 3
3
√
a3b3c33
3 1
a3b3c3
= 9
a3
+ b3
+ c3 bc2
a3
+
ca2
b3
+
ab2
c3
≥ 3
3
√
a3b3c3.3
3 a3b3c3
a3b3c3
= 9.
Cộng vế với vế ta có điều phải chứng minh.
Đẳng thức xảy ra khi và chỉ khi a = b = c = 1.
49
Chương 3. Bất đẳng thức có tích các biến không đổi
3.1.2 Sử dụng bất đẳng thức Cauchy-Schwarz
Bài toán 3.7. Với a, b, c là các số thực dương thỏa mãn abc = 1. Chứng minh
rằng
1
1 + a + b2
+
1
1 + b + c2
+
1
1 + c + a2
≤ 1.
Chứng minh.
Đặt a = x3, b = y3, c = x3, với x, y, z > 0 và xyz = 1.
Bất đẳng thức cần chứng minh trở thành
1
1 + x3 + y6
+
1
1 + y3 + z6
+
1
1 + z3 + x6
≤ 1
Sử dụng bất đẳng thức Cauchy- Schwarz kết hợp với xyz = 1, ta có
1
1 + x3 + y6
=
z4 + x +
1
z2
(1 + x3 + y6)(z4 + x +
1
z2
)
≤
z4 + x +
1
z2
(z2 + x2 + y2)2
=
z4 + x2yz + x2y2
(x2 + y2 + z2)2
.
Chứng minh tương tự, ta có
1
1 + y3 + z6
=
x4 + y +
1
z2
(1 + y3 + z6)(x4 + y +
1
z2
)
≤
x4 + y2xz + x2y2
(x2 + y2 + z2)2
1
1 + z3 + x6
=
y4 + z +
1
x2
(1 + z3 + x6)(y4 + z +
1
x2
)
≤
y4 + z2xy + y2z2
(x2 + y2 + z2)2
.
Cộng các bất đẳng thức cùng chiều, ta có
1
1 + x3 + y6
+
1
1 + y3 + z6
+
1
1 + z3 + x6
≤
x4 + y4 + z4 + xyz(x + y + z) + x2y2 + y2z2 + z2x2
(x2 + y2 + z2)2
.
Bất đẳng thức được chứng minh nếu ta chứng minh được
x4 + y4 + z4 + xyz(x + y + z) + x2y2 + y2z2 + z2x2
(x2 + y2 + z2)2
≤ 1
⇔xyz(x + y + z) ≤ x2
y2
+ y2
z2
+ z2
x2
(điều này luôn đúng).
Đẳng thức xảy ra khi và chỉ khi a = b = c = 1.
50
Chương 3. Bất đẳng thức có tích các biến không đổi
Bài toán 3.8. Với các số thực dương x, y, z thỏa mãn xyz = 1. Chứng minh
1
x2 + x + 1
+
1
y2 + y + 1
+
1
z2 + z + 1
≥ 1.
Chứng minh.
Đặt x =
bc
a2
, y =
ca
b2
, z =
ab
c2
, với a, b, c > 0.
Khi đó bất đẳng thức trở thành
a4
b2c2 + a2bc + a4
+
b4
c2a2 + b2ca + b4
+
c4
a2b2 + c2ab + c4
≥ 1.
Áp dụng bất đẳng thức Cauchy- Schwarz, ta có
a4
b2c2 + a2bc + a4
+
b4
c2a2 + b2ca + b4
+
c4
a2b2 + c2ab + c4
≥
(a2 + b2 + c2)2
b2c2 + a2bc + a4 + c2a2 + b2ca + b4 + a2b2 + c2ab + c4
.
Bất đẳng thức được chứng minh nếu ta chỉ ra được
(a2
+ b2
+ c2
)2
≥ a4
+ b4
+ c4
+ abc(a + b + c) + a2
b2
+ b2
c2
+ c2
a2
.
Thật vậy
(a2
+ b2
+ c2
)2
≥ a4
+ b4
+ c4
+ abc(a + b + c) + a2
b2
+ b2
c2
+ c2
a2
⇔a2
b2
+ b2
c2
+ c2
a2
≥ abc(a + b + c)
⇔(ab − ac)2
+ (ab − bc)2
+ (bc − ca)2
≥ 0 ( luôn đúng).
Đẳng thức xảy ra khi và chỉ khi x = y = z = 1.
Bài toán 3.9 (IMO 2008). Với các số thực x, y, z khác 1 và thỏa mãn xyz = 1.
Chứng minh
x
x − 1
2
+
y
y − 1
2
+
z
z − 1
2
≥ 1.
Chứng minh.
Do x, y, z = 1 và xyz = 1 nên ∃a, b, c thỏa mãn
x =
a2
bc
, y =
b2
ca
, z =
c2
ab
và (a2 − bc)(b2 − ca)(c2 − ab) = 0.
Bất đẳng thức cần chứng minh trở thành
a4
(b2 − bc)2
+
b4
(b2 − ca)2
+
c4
(c2 − ab)2
≥ 1.
51
Chương 3. Bất đẳng thức có tích các biến không đổi
Áp dụng bất đẳng thức Cauchy-Schwarz, ta có
a4
(b2 − bc)2
+
b4
(b2 − ca)2
+
c4
(c2 − ab)2
≥
(a2 + b2 + c2)2
(a2 − bc)2 + (b2 − ca)2 + (c2 − ab)2
.
Bất đẳng thức đã cho được chứng minh nếu ta chứng minh được
(a2
+ b2
+ c2
)2
≥ (a2
− bc)2
+ (b2
− ac)2
+ (c2
− ab)2
⇔(ab + bc + ca)2
≥ 0 (luôn đúng).
Nhận xét 3.1. Bài toán trên có thể làm cách hai như sau
Đặt
a =
x
x − 1
, b =
y
y − 1
, c =
z
z − 1
suy ra x =
a
a − 1
, y =
b
b − 1
, z =
c
c − 1
.
Khi đó bất đẳng thức cần chứng minh trở thành
a2
+ b2
+ c2
≥ 1
Theo giả thiết xyz = 1
⇒
a
a − 1
b
b − 1
c
c − 1
= 1
⇔(a − 1)(b − 1)(c − 1) = abc
⇔(a + b + c) − 1 = ab + bc + ca
⇔2(a + b + c) − 2 = 2(ab + bc + ca)
⇔a2
+ b2
+ c2
− 1 = (a + b + c − 1)2
.
Do vế trái luôn không âm nên ta có a2 + b2 + c2 ≥ 1 ( điều phải chứng minh).
Bài toán 3.10 (IMO 1995). Với các số thực dương a, b, c và abc = 1. Chứng
minh
1
a3(b + c)
+
1
b3(c + a)
+
1
c3(a + b)
≥
3
2
.
Chứng minh.
Áp dụng bất đẳng thức Cauchy -Schwarz, ta có
1
a3(b + c)
+
1
b3(c + a)
+
1
c3(a + b)
≥
1
a
+
1
b
+
1
c
2
a(b + c) + b(c + a) + c(a + b)
=
(ab + bc + ca)2
a(b + c) + b(c + a) + c(a + b)
.
52
Chương 3. Bất đẳng thức có tích các biến không đổi
Bất đẳng thức được chứng minh nếu ta chứng minh được
(ab + bc + ca)2
≥
3
2
[a(b + c) + b(c + a) + c(a + b)]
⇔ab + bc + ca ≥ 3 ( luôn đúng thao AM-GM).
Dấu bằng xảy ra khi và chỉ khi a = b = c = 1.
3.1.3 Sử dụng các tính chất của hàm số
Bài toán 3.11 (Romania MO 2003). Với các số thực a, b, c > 0 và thỏa mãn
abc = 1. Chứng minh rằng
1 +
3
a + b + c
≥
6
ab + bc + ca
.
Chứng minh.
Bất đẳng thức cần chứng minh tương đương với
1 +
3
a + b + c
−
6
ab + bc + ca
≥ 0.
Đặt
√
a + b + c = t với t ≥ 3 3
√
abc =
√
3.
Ta có
1 +
3
a + b + c
−
6
ab + bc + ca
≥1 +
3
a + b + c
−
6
3abc(a + b + c)
=1 +
3
a + b + c
−
6
3(a + b + c)
=1 +
3
t2
−
6
√
3t
.
Xét hàm số f(t) = 1 +
3
t2
−
6
√
3t
với t ≥
√
3
f (t) =
−6
t3
+
2
√
3
t2
=
2
t2
√
3 −
3
t
≥ 0 với ∀t ≥
√
3.
Do đó f(t) ≥ f(
√
3) = 0.
Vậy bất đẳng thức được chứng minh.
53
Chương 3. Bất đẳng thức có tích các biến không đổi
Bài toán 3.12. Với a, b, c là các số thực dương thỏa mãn abc = 1. Chứng minh
rằng
1
a2 − a + 1
+
1
b2 − b + 1
+
1
c2 − c + 1
≤ 3.
Chứng minh.
Bất đẳng thức được viết lại như sau
4
3
−
1
a2 − a + 1
+
4
3
−
1
b2 − b + 1
+
4
3
−
1
c2 − c + 1
≥ 1
⇔
(2a − 1)2
a2 − a + 1
+
(2b − 1)2
b2 − b + 1
+
(2c − 1)2
c2 − c + 1
≥ 3.
Áp dụng bất đẳng thức Cauchy-Schwarz, ta được
(2a − 1)2
a2 − a + 1
+
(2b − 1)2
b2 − b + 1
+
(2c − 1)2
c2 − c + 1
≥
(2a + 2b + 2c − 3)2
a2 + b2 + c2 − (a + b + c) + 3
.
Do đó bất đẳng thức được chứng minh nếu ta chỉ ra được
(2a + 2b + 2c − 3)2
≥ 3 a2
+ b2
+ c2
− (a + b + c) + 3
⇔4(a + b + c)2
− 12(a + b + c) + 9 ≥ 3(a2
+ b2
+ c2
) − 3(a + b + c) + 9
⇔(a2
+ b2
+ c2
) − 9(a + b + c) + 8(ab + bc + ca) ≥ 0
⇔(a + b + c)2
+ 6(ab + bc + ca) ≥ 9(a + b + c) ( )
Ta có
(ab + bc + ca)2
≥ 3abc(a + b + c) = 3(a + b + c)
suy ra ab + bc + ca ≥ 3(a + b + c).
Đặt t =
a + b + c
3
với t ≥ 1.
Do đó muốn chứng minh được bất đẳng thức ( ) ta chỉ cần chứng minh
9t4
+ 18t ≥ 9.3t2
⇔t3
− 3t + 2 ≥ 0. ( )
Xét hàm f(t) = t3 − 3t + 2, với t ≥ 1
f (t) = 3t2
− 3 ≥ 0 với ∀t ≥ 1.
Do đó f(t) ≥ f(1) = 0.
Nên bất đẳng thức ( ) được chứng minh.
Đẳng thức xảy ra khi và chỉ khi a = b = c = 1.
54
Chương 3. Bất đẳng thức có tích các biến không đổi
3.1.4 Bài toán liên quan
Bài toán 3.13. Với các số thực dương a, b, c thỏa mãn abc = 1. Chứng minh
rằng
a + b + c ≥
1 + a
1 + b
+
1 + b
1 + c
+
1 + c
1 + a
.
Bài toán 3.14 (IMO -SL 1998). Với các số thực dương a, b, c thỏa mãn abc = 1.
Chứng minh rằng
a3
(1 + b)(1 + c)
+
b3
(1 + a)(1 + c)
+
c3
(1 + a)(1 + b)
≥
3
4
.
Bài toán 3.15 (Ukraine 1998 ). Với các số thực dương a, b, c > 0 và thỏa mãn
abc = 1. Chứng minh rằng
1 + ab
1 + a
+
1 + bc
1 + b
+
1 + ca
1 + a
≥ 3.
Bài toán 3.16. Với các số thực a, b, c > 0 và thỏa mãn abc = 1. Chứng minh
rằng
(a + b)(b + c)(c + a) ≥ 4(a + b + c − 1).
Bài toán 3.17. Với a, b, c là các số thực dương thỏa mãn abc = 1. Chứng minh
rằng
1
a2 − a + 1
+
1
b2 − b + 1
+
1
c2 − c + 1
≤ 3.
Bài toán 3.18. Với các số thực dương a, b, c thỏa mãn điều kiện abc = 1. Chứng
minh rằng
1
(a + 1)(a + 2)
+
1
(b + 1)(b + 2)
+
1
(c + 1)(c + 2)
≥
1
2
.
Bài toán 3.19. Với ba số thực dương a, b, c thỏa mãn điều kiện abc = 1. Chứng
minh bất đẳng thức sau
1
(1 + a)2
+
1
(1 + b)2
+
1
(1 + c)2
+
2
(1 + a)(1 + b)(1 + c)
≥ 1.
55
Chương 3. Bất đẳng thức có tích các biến không đổi
3.2 Bất đẳng thức có tích các biến không đổi với hàm vô tỉ
3.2.1 Sử dụng bất đẳng thức AM-GM
Bài toán 3.20 (Internationnal Compelition SRMC 2006). Với các số thực dương
a, b, c thỏa mãn abc = 1. Chứng minh rằng
4 3
a
b
+
3 b
c
+ 3
c
a
≤ 3 2 + a + b + c +
1
a
+
1
b
+
1
c
2
3 .
Chứng minh.
Do abc = 1 nên tồn tại các số dương x, y, z sao cho a =
y
x
, b =
z
y
, c =
x
z
.
Thay vào bất đẳng thức đã cho, ta có
4
3 y2
xz
+
3 z2
xy
+
3 x2
yz
≤ 3 2 +
y
x
+
z
y
+
x
z
+
x
y
+
y
z
+
z
x
2
3
⇔4
3 y3
xyz
+
3 z3
xyz
+
3 x3
xyz
≤ 3
2xyz + y2z + xz2 + x2y + x2z + y2x + z2y
xyz
2
3
⇔4
x + y + z
3
√
xyz
≤ 3
(x + y)(y + z)(z + x)
xyz
2
3
⇔xyz(x + y + z)3
≤
27
64
(x + y)2
(y + z)2
(z + x)2
.
Mà ta lại chứng minh được bất đẳng thức
(x + y)(y + z)(z + x) ≥
8
9
(x + y + z)(xy + yz + zx)
Do đó bất đẳng thức được chứng minh nếu ta chứng minh được
27
64
64
81
(x + y + z)2
(xy + yz + zx)2
≥ xyz(x + y + z)3
⇔(xy + yz + zx)2
≥ 3xyz(x + y + z) hiển nhiên đúng theo AM-GM .
Đẳng thức xảy ra khi và chỉ khi a = b = c = 1.
Bài toán 3.21. Với a, b, c là các số thực dương thỏa mãn điều kiện abc = 1.
Chứng minh rằng
3
√
a +
3
√
b + 3
√
c ≤ 3
3(3 + a + b + c + ab + bc + ca).
56
Chương 3. Bất đẳng thức có tích các biến không đổi
Chứng minh.
Vì abc = 1 nên đặt
a =
x2
yz
, b =
y2
zx
, c =
z2
xy
.
.
Khi đó bất đẳng thức cần chứng minh trở thành
3 x2
yz
+
3 y2
zx
+
3 z2
xy
≤ 3
3 3 +
x2
yz
+
y2
xz
+
z2
xy
+
xy
z2
+
yz
x2
+
xz
y2
= 3
3
x
y
+
y
z
+
z
x
x
z
+
y
x
+
z
y
= 3
(1 + 1 + 1)
x
y
+
y
z
+
z
x
x
z
+
y
x
+
z
y
.
Áp dụng bất đẳng thức AM − GM, ta có
1
3
+
x
y
x
y + y
z + z
x
+
x
z
x
z + y
x + z
y
≥ 3. 3
x2
yz
3 x
y + y
z + z
x
x
z + y
x + z
y
1
3
+
y
z
x
y + y
z + z
x
+
y
x
x
z + y
x + z
y
≥ 3 3
y2
xz
3 x
y + y
z + z
x
x
z + y
x + z
y
1
3
+
z
x
x
y + y
z + z
x
+
z
y
x
z + y
x + z
y
≥ 3 3
z2
xy
3 x
y + y
z + z
x
x
z + y
x + z
y
.
Cộng các bất đẳng thức cùng chiều ta có điều phải chứng minh.
Đẳng thức xảy ra khi và chỉ khi a = b = c = 1.
Bài toán 3.22 (APMO 2005). Với a, b, c là các số thực dương thỏa mãn abc = 8.
Chứng minh rằng
a2
(a3 + 1)(b3 + 1)
+
b2
(b3 + 1)(c3 + 1)
+
c2
(c3 + 1)(a3 + 1)
≥
4
3
.
Chứng minh.
57
Chương 3. Bất đẳng thức có tích các biến không đổi
Với ∀x ≥ 0, áp dụng bất đẳng thức AM-GM, ta có
x3 + 1 = (x + 1)(x2 − x + 1)
≤
x + 1 + x2 − x + 1
2
=
x2 + 2
2
.
Áp dụng bất đẳng thức này, ta có
a2
(a3 + 1)(b3 + 1)
+
b2
(b3 + 1)(c3 + 1)
+
c2
(c3 + 1)(a3 + 1)
≥
4a2
(a2 + 2)(b2 + 2)
+
4b2
(b2 + 2)(c2 + 2)
+
4c2
(c2 + 2)(a2 + 2)
.
Vậy ta chỉ cần chứng minh
4a2
(a2 + 2)(b2 + 2)
+
4b2
(b2 + 2)(c2 + 2)
+
4c2
(c2 + 2)(a2 + 2)
≥
4
3
⇔a2
(c2
+ 2) + b2
(a2
+ 2) + c2
(b2
+ 2) ≥
1
3
(a2
+ 2)(b2
+ 2)(c2
+ 2)
⇔a2
b2
+ b2
c2
+ c2
a2
+ 2(a2
+ b2
+ c2
) ≥ a2
b2
c2
+ 8 = 72.
Điều này hiển nhiên đúng vì theo bất đẳng thức AM-GM, ta có
a2
b2
+ b2
c2
+ c2
a2
≥ 3
3
√
a4b4c4 = 48
a2
+ b2
+ c2
≥ 3
3
√
a2b2c2 = 12.
Đẳng thức xảy ra khi và chỉ khi a = b = c = 2.
Bài toán 3.23 (Đề thi đại học khối D 2005). Với ba số thực dương x, y, z thỏa
mãn điều kiện xyz = 1. Chứng minh rằng
1 + x3 + y3
xy
+
1 + y3 + z3
yz
+
√
1 + z3 + x3
zx
≥ 3
√
3.
Chứng minh.
Áp dụng bất đẳng thức AM-GM, ta có
1 + x3 + y3
xy
≥
3 3
x3y3
xy
=
3
xy
.
Tương tự, ta có √
1 + z3 + x3
yz
≥
3
yz
58
Chương 3. Bất đẳng thức có tích các biến không đổi
√
1 + z3 + x3
xz
≥
3
xz
.
Cộng các bất đẳng thức cùng chiều ta có
1 + x3 + y3
xy
+
1 + y3 + z3
yz
+
√
1 + z3 + x3
zx
≥
3
xy
+
3
yz
+
3
xz
.
Mặt khác theo bất đẳng thức AM-GM, ta có
3
xy
+
3
yz
+
3
zx
≥ 3 3
√
27
x2y2z2
= 3
√
3.
Bất đẳng thức được chứng minh.
Đẳng thức xảy ra khi và chỉ khi x = y = z = 1.
3.2.2 Sử dụng bất đẳng thức Cauchy-Schwarz
Bài toán 3.24. Với a, b, c là các số thực dương thỏa mãn abc = 1. Chứng minh
rằng √
a
2 + b
√
a
+
√
b
2 + c
√
b
+
√
c
2 + a
√
c
≥ 1.
Chứng minh.
Đặt
√
a =
x
y
,
√
b =
y
z
,
√
c =
z
x
với x, y, z > 0.
Khi đó bất đẳng thức cần chứng minh trở thành
x
y
2 +
y2
z2
.
x
y
+
y
z
2 +
z2
x2
.
y
z
+
z
x
2 +
x2
y2
.
z
x
≥ 1
⇔
x2z2
2xyz2 + x2y2
+
y2x2
2x2yz + y2z2
+
z2y2
2xy2z + x2z2
≥ 1.
Áp dụng bất đẳng thức Cauchy-Schwarz, ta có
x2z2
2xyz2 + x2y2
+
y2x2
2x2yz + y2z2
+
z2y2
2xy2z + x2z2
≥
(xz + xy + yz)2
x2y2 + y2z2 + z2x2 + 2xyz(x + y + z)
=
(xz + xy + yz)2
(xz + xy + yz)2
= 1.
Bất đẳng thức được chứng minh.
Dấu bằng xảy ra khi à chỉ khi a = b = c = 1.
59
Chương 3. Bất đẳng thức có tích các biến không đổi
Bài toán 3.25 (Balkan Shortlist 2002). Với a, b, c, là các số thực dương thỏa
mãn abc = 2. Chứng minh rằng
a3
+ b3
+ c3
≥ a
√
b + c + b
√
c + a + c
√
a + b.
Chứng minh.
Do
2(a3
+ b3
+ c3
) − a2
(b + c) − b2
(c + a) − c2
(a + b)
= a3
+ b3
− ab(a + b) + b3
+ c3
− bc(b + c) + c3
+ a3
− ca(a + c)
=(a + b)(a − b)2
+ (b + c)(b − c)2
+ (c + a)(c − a)2
≥ 0 luôn đúng với ∀a, b, c .
Do đó ta cần chứng minh bất đẳng thức
a2
(b + c) + b2
(c + a) + c2
(a + b) ≥ 2(a
√
b + c + b
√
c + a + c
√
a + b).
Sử dụng Cauchy- Schwarz cho biểu thức ở vế trái, ta có
a2
(b + c) + b2
(c + a) + c2
(a + b) ≥
1
3
(a
√
b + c + b
√
c + a + c
√
a + b)2
.
Do đó ta chỉ cần chứng minh
a
√
b + c + b
√
c + a + c
√
a + b ≥ 6.
Sử dụng Bất đẳng thức AM-GM, ta có
a
√
b + c + b
√
c + a + c
√
a + b ≥
3
abc (a + b)(b + c)(c + a)
≥
3
abc 2
√
ab.2
√
bc.2
√
ca
= 6.
Đẳng thức xảy ra khi và chỉ khi a = b = c = 3
√
2.
3.2.3 Sử dụng các tính chất của hàm số
Bài toán 3.26 (IMO 2002 ). Với a, b, c là các số thực dương thỏa mãn điều kiện
abc = 1. Chứng minh rằng
a − 1 +
1
b
b − 1 +
1
c
c − 1 +
1
a
≤ 1
Chứng minh.
60
Chương 3. Bất đẳng thức có tích các biến không đổi
Vì abc = 1 nên tồn tại các số dương x, y, z sao cho a =
x
y
, b =
y
z
, c =
z
x
. Bất
đẳng thức cần chứng minh trở thành
(x − y + z)(y − z + x)(z − x + y) ≤ xyz.
Ta để ý rằng,
(x − y + z) + (y − z + x) = 2x > 0
(y − z + x) + (z − x + y) = 2y > 0
(z − x + y) + (x − y + z) = 2z > 0.
Do đó, trong ba số x−y +z, y −z +x, z −x+y không thể có trường hợp hai số
cùng âm hoặc cả ba số đều âm. Nếu trong ba số trên có một số âm, hiển nhiên
ta có bất đẳng thức cần chứng minh. Trường hợp cả ba số đều dương, bằng các
lấy logarit hai vế, ta có
ln(x − y + z) + ln(y − z + x) + ln(z − x + y) ≤ lnx + lny + lnz.
Không mất tính tổng quát, giả sử x ≥ y ≥ z. Khi đó,



y − z + x ≥ x
(y − z + x) + (x − y + z) ≥ x + y
(y − z + x) + (x − y + z) + (z − x + y) = x + y + z.
Vì f(x) = lnx là hàm lõm trên (0; +∞), do đó, sử dụng bất đẳng thức Kara-
mata, ta được
ln(y − z + x) + ln(x − y + z) + ln(z − x + y) ≤ lnx + lny + lnz.
Đẳng thức xảy ra khi và chỉ khi x = y = z hay a = b = c = 1.
Bài toán 3.27. Với các số thực dương a, b, c thỏa mãn abc = 1. Chứng minh
rằng
a
√
1 + a
+
b
√
1 + b
+
c
√
1 + c
≥
3
√
2
2
.
Chứng minh.
Từ giả thiết abc = 1 ⇔ ln(abc) = 0 ⇔ ln a + ln b + ln c = 0.
Đặt x = ln a, y = ln b, z = ln c. Khi đó x, y, z ∈ R và x + y + z = 0.
Bất đẳng thức đã cho trở thành
ex
√
1 + ex
+
ey
√
1 + ey
+
ez
1 + ez
≥
3
√
2
2
.
61
Chương 3. Bất đẳng thức có tích các biến không đổi
Xét hàm số f(t) =
et
√
1 + et
−
3
√
2
8
t
f (t) =
e2t + 2et
2 (1 + et)3
−
3
√
2
8
f (t) = 0 ⇔4(e2t
+ 2et
) = 3
√
2 (1 + et)3
⇔ (et
− 1)(16e3t
+ 62e2t
+ 72et
+ 18) = 0
⇔
et = 1
16e3t + 62e2t + 72et + 18 = 0 ( Vô nghiệm)
⇔ t = 0.
Ta có bảng biến thiên
t −∞ 0 +∞
f (t) − 0 +
f(t)
√
2
2
Từ bảng biến thiên, ta có
f(t) ≥
√
2
2
∀t ∈ R ⇔
et
√
1 + et
≥
3
√
2t
8
+
√
2
2
, ∀t ∈ R.
Thay t bởi x, y, z rồi cộng các bất đẳng thức cùng chiều ta có
ex
√
1 + ex
+
ey
√
1 + ey
+
ez
1 + ez
≥
3
√
2
8
(x + y + z) +
3
√
2
2
=
3
√
2
2
.
Đẳng thức xảy ra khi và chỉ khi x = y = z = 0 hay a = b = c = 1.
3.2.4 Bài toán liên quan
Bài toán 3.28. Với các số thực dương a, b, c thỏa mãn abc = 1. Chứng minh
rằng
2(a2 + 1) + 2(b2 + 1) + 2(c2 + 1) ≤ 1 +
5
3
(a + b + c).
Bài toán 3.29. Với các số thực dương a, b, c thỏa mãn abc = 1. Chứng minh
rằng
8a2 + 1 + 8b2 + 1 + 8c2 + 1 ≤ 3 (a + b + c) .
62
Chương 4
Một số lớp bài toán cực trị với đa
thức đối xứng ba biến
4.1 Sử dụng bất đẳng thức AM-GM
Bài toán 4.1. Với các số thực không âm a, b, c thỏa mãn a + b + c = 3. Tìm giá
trị lớn nhất của biểu thức
P = (a2
− ab + b2
)(b2
− bc + c2
)(c2
− ca + a2
).
Chứng minh.
Không mất tính tổng quát ta có thể giả sử c = min{a, b, c}.
Khi đó
b2
− bc + c2
= c(c − b) + b2
≤ b2
c2
− ca + a2
= c(c − a) + a2
≤ a2
.
Do đó
P = (a2
− ab + b2
)(b2
− bc + c2
)(c2
− ca + a2
)
≤ (a2
− ab + b2
)a2
b2
.
Áp dụng bất đẳng thức AM-GM, ta có
(a2
− ab + b2
)a2
b2
=
4
9
3ab
2
3ab
2
(a2
− ab + b2
)
≤
4
9



3ab
2
+
3ab
2
+ (a2 − ab + b2)
3



3
63
Chương 4. Một số lớp bài toán cực trị với đa thức đối xứng ba biến
=
4
9
(a2 + 2ab + b2)3
27
=
4
9
(a + b)6
27
≤
4
9.27
(a + b + c)6
= 12.
Đẳng thức xảy ra khi và chỉ khi a = 2, b = 1, c = 0 và các hoán vị của nó.
Vậy giá trị lớn nhất của P là 12.
Bài toán 4.2. Với a, b, c là các số thực dương thỏa mãn a + b + c = 2. Tìm giá
trị lớn nhất của
M =
ab
√
2c + ab
+
bc
√
2a + bc
+
ca
√
2b + ca
.
Chứng minh.
Sử dụng bất đẳng thức AM-GM, ta có
M =
ab
√
2c + ab
+
bc
√
2a + bc
+
ca
√
2b + ca
=
ab
(a + b + c)c + ab
+
bc
(a + b + c)a + bc
+
ca
(a + b + c)b + ca
=
ab
(b + c)(a + c)
+
bc
(a + b)(a + c)
+
ca
(b + c)(b + a)
≤
1
2
ab
c + a
+
ab
c + b
+
bc
a + b
+
bc
a + c
+
ac
b + a
+
ac
b + c
=
1
2
(a + b + c) = 1.
Dấu bằng xảy ra khi và chỉ khi a = b = c =
2
3
.
Vậy giá trị lớn nhất của biểu thức M là 1.
Bài toán 4.3. Với a, b, c là các số thực dương thỏa mãn a + b + c = 1. Tìm giá
trị lớn nhất của biểu thức
P = ab + bc + ca +
5
2
(a + b)
√
ab + (b + c)
√
bc + (c + a)
√
ca .
Chứng minh.
64
Chương 4. Một số lớp bài toán cực trị với đa thức đối xứng ba biến
Áp dụng bất đẳng thức AM-GM, ta có
2(a + b)2
+ 2ab =
(a + b)2
2
+
(a + b)2
2
+
(a + b)2
2
+
(a + b)2
2
+ 2ab
≥5
5 ab(a + b)8
8
=5(a + b)
5 ab(a + b)3
8
≥5(a + b)
5 ab(2
√
ab)3
8
=5(a + b)
√
ab.
Tương tự, ta có
2(b + c)2
+ 2bc ≥ 5(b + c)
√
bc
2(c + a)2
+ 2ac ≥ 5(c + a)
√
ca.
Cộng các bất đẳng thức cùng chiều, ta có
5 (a + b)
√
ab + (b + c)
√
bc + (c + a)
√
ca ≤ 4(a2
+ b2
+ c2
) + 6(ab + bc + ca)
⇔2(ab + bc + ca) + 5 (a + b)
√
ab + (b + c)
√
bc + (c + a)
√
ca
≤ 4(a2
+ b2
+ c2
) + 8(ab + bc + ca) = 4(a + b + c)2
= 4.
Đẳng thức xảy ra khi và chỉ khi a = b = c =
1
3
.
Vậy giá trị lớn nhất của P là 2.
Bài toán 4.4. Với các số thực dương a, b, c thỏa mãn a + b + c = 1. Tìm giá trị
nhỏ nhất của biêu thức
P =
a2 + b
b + c
+
b2 + c
c + a
+
c2 + a
a + b
.
Chứng minh.
Ta có
P + 1 =
a2 + b
b + c
+ a +
b2 + c
c + a
+ b +
c2 + a
a + b
+ c
=
a(a + b + c) + b
b + c
+
b(a + b + c) + c
c + a
+
c(a + b + c) + a
a + b
=
a + b
b + c
+
b + c
c + a
+
c + a
a + b
.
65
Chương 4. Một số lớp bài toán cực trị với đa thức đối xứng ba biến
Áp dụng bất đẳng thức AM-GM, ta có
P + 1 =
a + b
b + c
+
b + c
c + a
+
c + a
a + b
≥
3 a + b
b + c
.
b + c
c + a
.
c + a
a + b
= 3.
Đẳng thức xảy ra khi và chỉ khi a = b = c =
1
3
.
Vậy giá trị nhỏ nhất của biểu thức P là 2.
Bài toán 4.5. Với ba số thực dương a, b, c thỏa mãn abc = 1. Tìm giá trị lớn
nhất của biểu thức
P =
ab
a + b + ab
+
bc
b + c + bc
+
ca
c + a + ca
.
Chứng minh.
Với giả thiết a, b, c > 0 và abc = 1, ta có
P =
ab
a + b + ab
+
bc
b + c + bc
+
ca
c + a + ca
=
1
1
a
+
1
b
+ 1
+
1
1
b
+
1
c
+ 1
+
1
1
c
+
1
a
+ 1
.
Với các số dương x, y, theo bất đẳng thức AM-GM, ta có
x3
+ x3
+ y3
≥ 3 3
x6y3 = 3x2
y
y3
+ y3
+ x3
≥ 3 3
y6x3 = 3y2
x.
Do đó
x3
+ y3
≥ xy(x + y)
dấu ”=” xảy ra khi và chỉ khi x = y.
Áp dụng bất đẳng thức trên ta có
1
a
+
1
b
+ 1 ≥
3 1
ab
1
3
√
a
+
1
3
√
b
+ 1
= 3
√
c
1
3
√
a
+
1
3
√
b
+ 1
=
3
√
cb + 3
√
ca + 3
√
ab
3
√
ab
.
66
Chương 4. Một số lớp bài toán cực trị với đa thức đối xứng ba biến
Tương tự, ta có
1
b
+
1
c
+ 1 ≥
3
√
cb + 3
√
ca + 3
√
ab
3
√
bc
1
c
+
1
a
+ 1 ≥
3
√
cb + 3
√
ca + 3
√
ab
3
√
ac
.
Do đó
T ≤
3
√
ab
3
√
ab + 3
√
bc + 3
√
ca
+
3
√
bc
3
√
ab + 3
√
bc + 3
√
ca
+
3
√
ca
3
√
ab + 3
√
bc + 3
√
ca
= 1.
Đẳng thức xảy ra khi và chỉ khi a = b = c = 1.
Vậy giá trị lớn nhất của P là 1.
Bài toán 4.6. Với x, y, z là các số thực dương thay đổi thỏa mãn xyz = 1. Tìm
giá trị nhỏ nhất của biểu thức
P =
x2(y + z)
y
√
y + 2z
√
z
+
y2(z + x)
z
√
z + 2x
√
x
+
z2(x + y)
x
√
x + 2y
√
y
.
Chứng minh.
Theo bất đẳng thức AM-GM, ta có
x2
(y + z) ≥ 2x2√
yz = 2x
√
x
y2
(z + x) ≥ 2y2√
zx = 2y
√
y
z2
(x + y) ≥ 2z2√
xy = 2z
√
z.
Do đó
P ≥
2x
√
x
y
√
y + 2z
√
z
+
2y
√
y
z
√
z + 2x
√
x
+
2z
√
z
x
√
x + 2y
√
y
.
Đặt x
√
x = a, y
√
y = b, z
√
z = c, suy ra
P ≥ 2
a
b + 2c
+
b
c + 2a
+
c
a + 2b
= 2
a
b + 2c
+
b
c + 2a
+
c
a + 2b
Do đó áp dụng bất đẳng thức Cauchy-Schwarz và AM-GM, ta có
P ≥ 2
(a + b + c)2
3(ab + bc + ca)
≥
2(ab + bc + ca)
ab + bc + ca
= 2.
67
Chương 4. Một số lớp bài toán cực trị với đa thức đối xứng ba biến
Đẳng thức xảy ra khi và chỉ khi x = y = z = 1.
Vậy giá trị nhỏ nhất của P là 2.
Bài toán 4.7. Với a, b, c là các số thực dương thỏa mãn điều kiện abc = 1. Tìm
giá trị lớn nhất của biểu thức
P =
a3 + b3
a2 + ab + b2
+
b3 + c3
b2 + bc + c2
+
c3 + a3
c2 + ca + a2
.
Chứng minh.
Ta có
P =
(a + b)(a2 − ab + b2)
a2 + ab + b2
+
(b + c)(b2 − bc2
c)
b2 + bc + c2
+
(c + a)(c2 − ca + a2)
c2 + ca + a2
.
Ta sẽ chứng minh với mọi x, y dương, ta có
x2 − xy + y2
x2 + xy + y2
≥
1
3
.
Thật vậy
x2 − xy + y2
x2 + xy + y2
≥
1
3
⇔3(x2
− xy + y2
) ≥ x2
+ xy + y2
⇔2(x − y)2
≥ 0 ( luôn đúng) .
Áp dụng kết quả trên, ta có
P ≥
1
3
(a + b) +
1
3
(b + c) +
1
3
(c + a)
=
2
3
(a + b + c).
Lại theo bất đẳng thức AM-GM, ta có
P ≥
2
3
(a + b + c) ≥
2.3 3
√
abc
3
= 2.
Dấu "= " xảy ra khi và chỉ khi a = b = c = 1.
Vậy giá trị nhỏ nhất của A bằng 2.
4.2 Sử dụng bất đẳng thức Cauchy-Schwarz
Bài toán 4.8. Với các số thực dương a, b, c thỏa mãn a + b + c = 3. Tìm giá trị
nhỏ nhất của biểu thức
P =
1
(1 +
√
ab)2
+
1
(1 +
√
bc)2
+
1
(1 +
√
ca)2
.
68
Chương 4. Một số lớp bài toán cực trị với đa thức đối xứng ba biến
Chứng minh.
Áp dụng liên tiếp bất đẳng thức Cauchy- Schwarz, ta có
3P =3
1
(1 +
√
ab)2
+
1
(1 +
√
bc)2
+
1
(1 +
√
ca)2
≥
1
1 +
√
ab
+
1
1 +
√
bc
+
1
1 +
√
ca
2
≥
(1 + 1 + 1)2
1 +
√
ab + 1 +
√
bc + 1 +
√
ca
2
=
81
(3 +
√
ab +
√
bc +
√
ca)2
.
Mặt khác theo bất đẳng thức AM-GM, ta có
√
ab ≤
a + b
2
√
bc ≤
b + c
2
√
ca ≤
c + a
2
.
Do đó
√
ab +
√
bc +
√
ca ≤ a + b + c
Nên
3P ≥
81
(3 + a + b + c)2
=
9
4
hay P ≥
3
4
.
Đẳng thức xảy ra khi và chỉ khi a = b = c = 1.
Vậy giá trị nhỏ nhất của P =
3
4
.
Bài toán 4.9. Cho các số thực dương x, y, z thỏa mãn điều kiện x + y + z = 3.
Tìm giá trị nhỏ nhất của biểu thức
P = x4
+ 2y4
+ 3z4
Chứng minh.
69
Chương 4. Một số lớp bài toán cực trị với đa thức đối xứng ba biến
Áp dụng bất đẳng thức Cauchy- Schwarz ta có
x4
+ 2y4
+ 3z4
1 +
1
3
√
2
+
1
3
√
3
≥ x2
+
3
√
2y2
+
3
√
3z2 2
và
x2
+
3
√
2y2
+
3
√
3z2
1 +
1
3
√
2
+
1
3
√
3
≥ (x + y + z)2
= 9
Do đó
P = x4
+ 2y4
+ 3z4
≥
81
1 +
1
3
√
2
+
1
3
√
3
3
Dấu bằng xảy ra khi và chỉ khi



x =
3 3
√
6
3
√
6 + 3
√
3 + 3
√
2
y =
3 3
√
3
3
√
6 + 3
√
3 + 3
√
2
z =
3 3
√
2
3
√
6 + 3
√
3 + 3
√
2
Vậy giá trị nhỏ nhất của P là
81
1 +
1
3
√
2
+
1
3
√
3
3
.
Bài toán 4.10. Với a, b, c là các số thực thỏa mãn a + b + c = 3. Tìm giá trị nhỏ
nhất của biểu thức
A =
1
a2 + b2 + c2
+
1
a2
+
1
b2
+
1
c2
.
Chứng minh.
Áp dụng bất đẳng thức AM-GM, ta có
1
a2
+
1
b2
+
1
c2
≥
1
ab
+
1
bc
+
1
ca
.
Do đó
A ≥
1
a2 + b2 + c2
+
1
ab
+
1
bc
+
1
ca
.
70
Chương 4. Một số lớp bài toán cực trị với đa thức đối xứng ba biến
Áp dụng liên tiếp bất đẳng thức Cauchy - Schwarz, ta có
A ≥
1
a2 + b2 + c2
+
32
(ab + bc + ca)
=
1
a2 + b2 + c2
+
92
9(ab + bc + ca)
≥
(1 + 9)2
a2 + b2 + c2 + 9(ab + bc + ca)
=
102
(a + b + c)2 + 7(ab + bc + ca)
.
Áp dụng bất đẳng thức AM-GM ta có
7(ab + bc + ca) ≤
7
3
(a + b + c)2
= 21.
Do đó
A ≥
102
32 + 21
=
10
3
.
Đẳng thức xảy ra khi và chỉ khi a = b = c = 1.
Vậy giá trị nhỏ nhất của A bằng
10
3
.
Bài toán 4.11. Với các số thực dương a, b, c thỏa mãn abc = 1. Tìm giá trị lớn
nhất của biểu thức
F =
√
a2 + 1 +
√
b2 + 1 +
√
c2 + 1
a + b + c
.
Chứng minh.
Áp dụng bất đẳng thức Cauchy-Schwarz, ta có
1 a2 + 1 + 1.
√
2a ≤ (1 + 1)(a2 + 1 + 2a) =
√
2(a + 1).
Tương tự
b2 + 1 +
√
2b ≤
√
2(b + 1)
c2 + 1 +
√
2c ≤
√
2(c + 1).
Cộng các bất đẳng thức cùng chiều ta có
a2 + 1 + b2 + 1 + c2 + 1 +
√
2(
√
a +
√
b +
√
c) ≤
√
2(a + b + c + 3)
⇔ a2 + 1 + b2 + 1 + c2 + 1 ≤
√
2(a + b + c + 3) −
√
2(
√
a +
√
b +
√
c).
71
Chương 4. Một số lớp bài toán cực trị với đa thức đối xứng ba biến
Mặt khác theo bất đẳng thức AM-GM, ta có
√
a +
√
b +
√
c ≥ 3
3 √
abc = 3
⇔ −
√
2(
√
a +
√
b +
√
c) ≤ −3
√
2.
Do đó
a2 + 1 + b2 + 1 + c2 + 1 ≤
√
2(a + b + c + 3) − 3
√
2
=
√
2(a + b + c).
Suy ra F ≤
√
2 dấu "=" xảy ra khi và chỉ khi a = b = c = 1.
Vậy giá trị lớn nhất của F là
√
2.
Bài toán 4.12. Với các số thực dương x, y, z thỏa mãn x + y + z = 1. Tìm giá
trị lớn nhất của biểu thức
P =
xy
x2 + y2 + 2z2
+
yz
y2 + z2 + 2x2
+
zx
z2 + x2 + 2y2
.
Chứng minh.
Áp dụng bất đẳng thức Cauchy -Schwarz, ta có
P =
xy
3
2
1
9
+
1
9
+
2
9
(x2 + y2 + 2z2)
+
yz
3
2
1
9
+
1
9
+
2
9
(y2 + z2 + 2x2)
+
zx
3
2
1
9
+
1
9
+
2
9
(z2 + x2 + 2y2)
≤
xy
3
2
x
3
+
y
3
+
2z
3
+
yz
3
2
y
3
+
z
3
+
2x
3
+
zx
3
2
z
3
+
x
3
+
2y
3
=
2xy
(x + z) + (y + z)
+
2yz
(y + x) + (z + x)
+
2zx
(z + y) + (x + y)
≤
1
4
2xy
x + z
+
2xy
y + z
+
1
4
2yz
y + x
+
2yz
z + x
+
1
4
2zx
z + y
+
2zx
x + y
=
1
4
(2x + 2y + 2z) =
1
2
.
Dấu "=" xảy ra khi và chỉ khi x = y = z =
1
3
.
Vậy giá trị lớn nhất của P là
1
2
.
72
Chương 4. Một số lớp bài toán cực trị với đa thức đối xứng ba biến
Bài toán 4.13. Với các số thực dương a, b, c thỏa mãn điều kiện a + b + c =
3
4
.
Tìm giá trị nhỏ nhất của
P =
1
3
√
a + 3b
+
1
3
√
b + 3c
+
1
3
√
c + 3a
.
Chứng minh.
Áp dụng bất đẳng thức AM-GM, ta có
P ≥
1
a + 3b + 1 + 1
3
+
1
b + 3c + 1 + 1
3
+
1
c + 3a + 1 + 1
3
=
3
a + 3b + 2
+
3
b + 3c + 2
+
3
c + 3a + 1
.
Theo bất đẳng thức Cauchy-Schwarz, ta có
P ≥
3
a + 3b + 2
+
3
b + 3c + 2
+
3
c + 3a + 1
≥ 3
9
4(a + b + c) + 6
= 3.
Dấu"=" xảy ra khi và chỉ khi a = b = c =
1
3
.
Vậy giá trị nhỏ nhất của P là 3.
4.3 Sử dụng các tính chất của hàm số
Bài toán 4.14. Với các số thực không âm a, b, c thỏa mãn a + b + c = 1. Tìm giá
trị nhỏ nhất của
P = 3(a2
b2
+ b2
c2
+ c2
a2
) + 3(ab + bc + ca) + 2 a2 + b2 + c2.
Chứng minh.
Vì a2 + b2 + c2 = (a + b + c)2 − 2(ab + bc + ca) = 1 − 2(ab + bc + ca).
Và theo bất đẳng thức Cauchy- Schwarz, ta có
3(a2
b2
+ b2
c2
+ c2
a2
) ≥ (ab + bc + ca)2
.
Do đó
P ≥ (ab + bc + ca)2
+ 3(ab + bc + ca) + 2 1 − 2(ab + bc + ca).
Đặt ab + bc + ca = t.
73
Chương 4. Một số lớp bài toán cực trị với đa thức đối xứng ba biến
Có 0 ≤ t = ab + bc + ca ≤
(a + b + c)2
3
=
1
3
.
Khi đó
P ≥ t2 + 3t + 2
√
1 − 2t, t ∈ 0;
1
3
.
Xét hàm số f(t) = t2 + 3t + 2
√
1 − 2t, t ∈ 0;
1
3
.
f (t) = 2t + 3 −
2
√
1 − 2t
,
f (t) = 2 −
2
(1 − 2t)3
≤ 0, ∀t ∈ 0;
1
3
.
Suy ra f (t) ≥ f (
1
3
) =
11 − 6
√
3
3
> 0 ⇒ f(t) ≥ f(0) = 2, ∀t ∈ 0;
1
3
.
Do đó P ≥ 2.
Đẳng thức xảy ra khi và chỉ khi



ab + bc + ca = 0
ab = bc = ca
a + b + c = 1
⇔ a = 1; b = c = 0 và các hoán vị .
Vậy giá trị nhỏ nhất của P là 2.
Bài toán 4.15. Với các số thực dương a, b, c thỏa mãn điều kiện a + b + c = 3.
Tìm giá trị nhỏ nhất của
P =
a
b
+
b
c
+
c
a
− 4abc.
Chứng minh.
Áp dụng bất đẳng thức AM- GM, ta có
P =
a
√
ab
+
b
√
bc
+
c
√
ca
− 4abc
≥
2a
a + b
+
2b
b + c
+
2c
c + a
− 4abc
= 2
a
a + b
+
b
b + c
+
c
c + a
− 4abc.
74
Chương 4. Một số lớp bài toán cực trị với đa thức đối xứng ba biến
Áp dụng bất đẳng thức Cauchy-Schwarz, ta có
P ≥ 2
a
a + b
+
b
b + c
+
c
c + a
− 4abc
≥ 2
(
√
a +
√
b +
√
c)2
2(a + b + c)
− 4abc
=
(
√
a +
√
b +
√
c)2
3
− 4abc
≥ 3
3
√
abc − 4abc.
Đặt 3
√
abc = t với 0 < t = 3
√
abc ≤
a + b + c
3
= 1.
Xét hàm f(t) = 3t − 4t3, t ∈ (0; 1],
f (t) = 3 − 12t2
⇒f (t) = 0 ⇔


t =
1
2
∈ (0; 1]
t =
−1
2
/∈ (0; 1].
Bảng biến thiên
t 0
1
2
1
f (t) | + 0 − |
f(t)
0
1
−1
Từ bảng biến thiên ta có f(t) ≥ −1 ⇒ P ≥ f(t) ≥ −1.
Đẳng thức xảy ra khi và chỉ khi a = b = c = 1.
Vậy giá trị nhỏ nhất của P là -1.
Bài toán 4.16. Với a, b, c là các số thực dương thỏa mãn a + b + c = 3. Tìm giá
trị nhỏ nhất của biểu thức
P = ab + bc + ca +
1
abc
− abc.
Chứng minh.
Áp dụng bất đẳng thức AM-GM, ta có 3 = a + b + c ≥ 3 3
√
abc ⇒ abc ≤ 1.
Theo giải thiết ta có 0 < abc ≤ 1.
Khi đó
P = ab + bc + ca +
1
abc
− abc
≥ 3
3
√
a2b2c2 +
1
abc
− abc.
75
Chương 4. Một số lớp bài toán cực trị với đa thức đối xứng ba biến
Đặt 3
√
abc = t với t ∈ (0; 1].
Ta có
P ≥ 3t2
+
1
t3
− t3
.
Xét hàm số f(t) = 3t2 +
1
t3
− t3 với t ∈ (0; 1]
ta có
f (t) =6t −
3
t4
− 3t2
=
−3t6 + 6t5 − 3
t4
=
3(t − 1)(−t5 + t4 + t2 + t + 1)
t4
≤ 0 ∀t ∈ (0; 1] .
Do đó P ≥ f(t) ≥ f(1) = 3.
Đẳng thức xảy ra dấu bằng khi và chỉ khi t = 1 ⇒ a = b = c = 1.
Vậy giá trị nhỏ nhất của P là 3.
Bài toán 4.17. Với 3 số x, y, z thỏa mãn
x, y, z ∈ [−1; 3]
x + y + z = 3
. Tìm giá trị lớn nhất
của
P = x2
+ y2
+ z2
.
Chứng minh.
Giả sử 3 ≥ x ≥ y ≥ z ≥ −1 . Khi đó



3 ≥ x
3 + 1 ≥ x + y
3 + 1 + (−1) = x + y + z
Xét hàm số f(t) = t2, với t ∈ [−1; 3]
f (t) = 2t
f (t) = 2 ≥ 0, với t ∈ [−1; 3].
Do đó với x, y, z ∈ [−1; 3], theo bất đẳng thức Karamata ta có
f(3) + f(1) + f(−1) ≥ f(x) + f(y) + f(z)
⇔x2
+ y2
+ z2
≤ 11.
Đẳng thức xảy ra khi và chỉ khi x = 3; y = 1; z = −1 và các hoán vị của chúng.
76
Chương 4. Một số lớp bài toán cực trị với đa thức đối xứng ba biến
Bài toán 4.18. Với −1 ≤ a, b, c ≤ 1 và a + b + c =
−1
2
. Hãy tìm giá trị lớn nhất
của biểu thức
F = a12
+ b12
+ c12
.
Chứng minh.
Không mất tính tổng quát, giả sử 1 ≥ a ≥ b ≥ c ≥ −1. Khi đó



a ≤ 1
a + b ≤ 1 −
1
2
a + b + c = 1 −
1
2
− 1
.
Vì hàm số f(x) = x12 lồi trên [−1; 1], theo bất đẳng thức Karamata, ta có
a12
+ b12
+ c12
= f(a) + f(b) + f(c) ≤ f(1) + f(
−1
2
) + f(−1) = 2 +
1
212
Đẳng thức xảy ra khi a = 1, b =
−1
2
, c = −1 và các hoán vị của chúng.
Vậy giá trị lớn nhất của F bằng 2 +
1
212
.
4.4 Bài toán liên quan
Bài toán 4.19. Với ba số thực dương x, y, z thỏa mãn xyz = 1. Tìm giá trị nhỏ
nhất của biểu thức
P =
x3
(1 + y)(1 + z)
+
y3
(1 + z)(1 + x)
+
z3
(1 + x)(1 + y)
.
Bài toán 4.20. Với x, y, z là các số thực dương thỏa mãn x + y + z = 6. Tìm giá
trị lớn nhất của biểu thức
P =
x2
2
+
y3
12
+
z4
108
.
Bài toán 4.21 (HSG Hà nội 2014-2015). Vơi ba số thực dương a, b, c thỏa mãn
a + b + c = 1. Tìm giá trị nhỏ nhất của biểu thức
P =
1
a2 + b2 + c2
+
1
abc
.
Bài toán 4.22 (Đại học khối B năm 2012). Với các số thực x, y, z thỏa mãn
x + y + z = 0; x2 + y2 + z2 = 1. Tìm giá trị lớn nhất của
P = x5
+ y5
+ z5
.
77
KẾT LUẬN
Các bài toán bất đẳng thức và cực trị với đa thức đối xứng vẫn là một chủ
đề hay và khó với học sinh và ngay cả giáo viên. Tuy nhiên trong các kì thi Đại
học, thi học sinh giỏi quốc gia, thi Olympic Toán học quốc tế lại thường có dạng
toán này. Đặc biệt là các bài toán bất đẳng thức và cực trị với đa thức đối xứng
ba ba biến. Luận văn hoàn thành và đạt được các kết quả sau:
1. Giới thiệu lại các kiến thức cơ bản của đa thức đối xứng ba biến, của bất
đẳng thức và các bất đẳng thức AM-GM, bất đẳng thức Cauchy - Schwarz
và bất đẳng thức Karamata cùng bổ đề cơ bản.
2. Đưa ra các cách tiếp cận đối với bất đẳng thức đối xứng ba biến có tổng -
tích không đổi và hệ thống các ví dụ và đề thi quốc gia và quốc tế.
3. Đưa ra lớp các bài toán cực trị với đa thức đối xứng ba biến và các cách
giải quyết bài toán cung hệ thống ví dụ đa dạng.
Với những gì đạt được, tôi hi vọng luận văn sẽ là một tài liệu có ích cho
những ai quan tâm tới bất đẳng thức và bài toán cực trị với đa thức đối xứng
ba biến.
Trong điều kiên về thời gian và khuôn khổ của luận văn, tôi chưa thể nghiên
cứu thêm về bất đẳng thức và bài toán cực trị với đa thức đối xứng khác. Đó là
hướng phát triển của luận văn.
Tác giả rất mong nhận được sự góp ý của các thầy cô và các bạn để luận văn
được hoàn thiện hơn.
Xin chân thành cảm ơn!
78
Chương 4. Một số lớp bài toán cực trị với đa thức đối xứng ba biến
TÀI LIỆU THAM KHẢO
Tiếng Việt
[1] Võ Quốc Bá Cẩn - Trần Quốc Anh, Sử dụng AM-GM để chứng minh bất
đẳng thức, NXBĐH Sư Phạm.
[2] Phạm Kim Hùng, Sáng tạo bất đẳng thức, NXB Hà Nội.
[3] Nguyễn Văn Mậu, 2002, Bất đẳng thức, định lý và áp dụng, NXBGD.
[4] Nguyễn Văn Mậu, Các bài toán nội suy và áp dụng,NXB GD .
[5] Nguyễn Văn Mậu, Đa thức đại số và phân thức hữu tỷ, NXB GD 2002.
[6] N.V. Mậu, T.N. Dũng, N.Đ. Phất, N.T. Thanh, Số phức và áp dụng, NXB
GD 2009.
[7] Trần Phương, Võ Quốc Bá Cẩn, Trần Quốc Anh, Vẻ đẹp bất đẳng thức
trong các kì thi Olympic toán học, NXBĐHQG Hà Nội.
[8] Cao Minh Quang,Một số dạng toán về bất đẳng thức ba biến với tích các
biến không đổi, Hội thảo khoa học Các chuyên đề toán học bồi dưỡng học sinh
giỏi Đồng Tháp 2013.
[9] Phạn Văn Thuận, Lê Vĩ, Bất đẳng thức suy luận và khám phá, NXBĐHQG
Hà Nội.
Tiếng Anh
[10] D. Djukic, V. Jankovic, I. Matic and N. Petrovic, The IMO Compendium
1959-2004, Springer-Verlag 2004.
[11] D, S. Mitrinovic, J. E. Pecaric, ” Recent Advances in Geometric Inequal-
ities”, Kluwer Academic Publishers, 1989.
79

More Related Content

What's hot

Bài toán liên quan về Phân số tối giản trong Toán lớp 6
Bài toán liên quan về Phân số tối giản trong Toán lớp 6Bài toán liên quan về Phân số tối giản trong Toán lớp 6
Bài toán liên quan về Phân số tối giản trong Toán lớp 6Bồi dưỡng Toán lớp 6
 
Toán lớp 9 - Tổng hợp kiến thức lý thuyết Đại số 9 và Hình học 9
Toán lớp 9 - Tổng hợp kiến thức lý thuyết Đại số 9 và Hình học 9Toán lớp 9 - Tổng hợp kiến thức lý thuyết Đại số 9 và Hình học 9
Toán lớp 9 - Tổng hợp kiến thức lý thuyết Đại số 9 và Hình học 9Bồi dưỡng Toán lớp 6
 
Bài toán số học liên quan tới lũy thữa
Bài toán số học liên quan tới lũy thữaBài toán số học liên quan tới lũy thữa
Bài toán số học liên quan tới lũy thữaThế Giới Tinh Hoa
 
Nhị thức newton và Các bài tập ứng dụng
Nhị thức newton và Các bài tập ứng dụngNhị thức newton và Các bài tập ứng dụng
Nhị thức newton và Các bài tập ứng dụngLinh Nguyễn
 
TÁCH ĐỀ TUYỂN SINH VÀO LỚP 10 MÔN TOÁN CÁC TỈNH NĂM HỌC 2023-2024 - CÓ LỜI GI...
TÁCH ĐỀ TUYỂN SINH VÀO LỚP 10 MÔN TOÁN CÁC TỈNH NĂM HỌC 2023-2024 - CÓ LỜI GI...TÁCH ĐỀ TUYỂN SINH VÀO LỚP 10 MÔN TOÁN CÁC TỈNH NĂM HỌC 2023-2024 - CÓ LỜI GI...
TÁCH ĐỀ TUYỂN SINH VÀO LỚP 10 MÔN TOÁN CÁC TỈNH NĂM HỌC 2023-2024 - CÓ LỜI GI...Nguyen Thanh Tu Collection
 
Cđ đồng dư thức trong toán 7
Cđ đồng dư thức trong toán 7Cđ đồng dư thức trong toán 7
Cđ đồng dư thức trong toán 7Cảnh
 
Tuyển tập 500 bài Bất Đẳng Thức cổ điển
Tuyển tập 500 bài Bất Đẳng Thức cổ điểnTuyển tập 500 bài Bất Đẳng Thức cổ điển
Tuyển tập 500 bài Bất Đẳng Thức cổ điểnNguyễn Việt Long
 
Bộ sưu tập bất đẳng thức của võ quốc bá cẩn
Bộ sưu tập bất đẳng thức của võ quốc bá cẩnBộ sưu tập bất đẳng thức của võ quốc bá cẩn
Bộ sưu tập bất đẳng thức của võ quốc bá cẩnThế Giới Tinh Hoa
 
Bồi dưỡng nâng cao HSG Toán lớp 7 qua 16 chuyên đề - Thầy Thích
Bồi dưỡng nâng cao HSG Toán lớp 7 qua 16 chuyên đề - Thầy ThíchBồi dưỡng nâng cao HSG Toán lớp 7 qua 16 chuyên đề - Thầy Thích
Bồi dưỡng nâng cao HSG Toán lớp 7 qua 16 chuyên đề - Thầy ThíchBồi dưỡng Toán lớp 6
 
kỹ thuật giải phương trình hàm
kỹ thuật giải phương trình hàmkỹ thuật giải phương trình hàm
kỹ thuật giải phương trình hàmljmonking
 
Một số dạng toán về đa thức qua các kỳ thi Olympic
Một số dạng toán về đa thức qua các kỳ thi OlympicMột số dạng toán về đa thức qua các kỳ thi Olympic
Một số dạng toán về đa thức qua các kỳ thi OlympicMan_Ebook
 
Tổng hợp các phương pháp giải phương trình và hệ phương trình môn toán
Tổng hợp các phương pháp giải phương trình và hệ phương trình môn toánTổng hợp các phương pháp giải phương trình và hệ phương trình môn toán
Tổng hợp các phương pháp giải phương trình và hệ phương trình môn toánhuyenltv274
 
Phuong trinh vo ty
Phuong trinh vo tyPhuong trinh vo ty
Phuong trinh vo tytututhoi1234
 

What's hot (20)

Bài toán liên quan về Phân số tối giản trong Toán lớp 6
Bài toán liên quan về Phân số tối giản trong Toán lớp 6Bài toán liên quan về Phân số tối giản trong Toán lớp 6
Bài toán liên quan về Phân số tối giản trong Toán lớp 6
 
Toán lớp 9 - Tổng hợp kiến thức lý thuyết Đại số 9 và Hình học 9
Toán lớp 9 - Tổng hợp kiến thức lý thuyết Đại số 9 và Hình học 9Toán lớp 9 - Tổng hợp kiến thức lý thuyết Đại số 9 và Hình học 9
Toán lớp 9 - Tổng hợp kiến thức lý thuyết Đại số 9 và Hình học 9
 
Bài toán số học liên quan tới lũy thữa
Bài toán số học liên quan tới lũy thữaBài toán số học liên quan tới lũy thữa
Bài toán số học liên quan tới lũy thữa
 
Nhị thức newton và Các bài tập ứng dụng
Nhị thức newton và Các bài tập ứng dụngNhị thức newton và Các bài tập ứng dụng
Nhị thức newton và Các bài tập ứng dụng
 
Dãy số và giới hạn
Dãy số và giới hạnDãy số và giới hạn
Dãy số và giới hạn
 
Luận văn: Định lý bézout và chiều ngược lại, HAY, 9đ
Luận văn: Định lý bézout và chiều ngược lại, HAY, 9đLuận văn: Định lý bézout và chiều ngược lại, HAY, 9đ
Luận văn: Định lý bézout và chiều ngược lại, HAY, 9đ
 
TÁCH ĐỀ TUYỂN SINH VÀO LỚP 10 MÔN TOÁN CÁC TỈNH NĂM HỌC 2023-2024 - CÓ LỜI GI...
TÁCH ĐỀ TUYỂN SINH VÀO LỚP 10 MÔN TOÁN CÁC TỈNH NĂM HỌC 2023-2024 - CÓ LỜI GI...TÁCH ĐỀ TUYỂN SINH VÀO LỚP 10 MÔN TOÁN CÁC TỈNH NĂM HỌC 2023-2024 - CÓ LỜI GI...
TÁCH ĐỀ TUYỂN SINH VÀO LỚP 10 MÔN TOÁN CÁC TỈNH NĂM HỌC 2023-2024 - CÓ LỜI GI...
 
Cđ đồng dư thức trong toán 7
Cđ đồng dư thức trong toán 7Cđ đồng dư thức trong toán 7
Cđ đồng dư thức trong toán 7
 
Tuyển tập 500 bài Bất Đẳng Thức cổ điển
Tuyển tập 500 bài Bất Đẳng Thức cổ điểnTuyển tập 500 bài Bất Đẳng Thức cổ điển
Tuyển tập 500 bài Bất Đẳng Thức cổ điển
 
Dãy số namdung
Dãy số namdungDãy số namdung
Dãy số namdung
 
Bdt thuần nhất
Bdt thuần nhấtBdt thuần nhất
Bdt thuần nhất
 
Chuyen de dao ham
Chuyen de dao ham Chuyen de dao ham
Chuyen de dao ham
 
Bộ sưu tập bất đẳng thức của võ quốc bá cẩn
Bộ sưu tập bất đẳng thức của võ quốc bá cẩnBộ sưu tập bất đẳng thức của võ quốc bá cẩn
Bộ sưu tập bất đẳng thức của võ quốc bá cẩn
 
Bồi dưỡng nâng cao HSG Toán lớp 7 qua 16 chuyên đề - Thầy Thích
Bồi dưỡng nâng cao HSG Toán lớp 7 qua 16 chuyên đề - Thầy ThíchBồi dưỡng nâng cao HSG Toán lớp 7 qua 16 chuyên đề - Thầy Thích
Bồi dưỡng nâng cao HSG Toán lớp 7 qua 16 chuyên đề - Thầy Thích
 
kỹ thuật giải phương trình hàm
kỹ thuật giải phương trình hàmkỹ thuật giải phương trình hàm
kỹ thuật giải phương trình hàm
 
Một số dạng toán về đa thức qua các kỳ thi Olympic
Một số dạng toán về đa thức qua các kỳ thi OlympicMột số dạng toán về đa thức qua các kỳ thi Olympic
Một số dạng toán về đa thức qua các kỳ thi Olympic
 
TOÁN LỚP 5 NÂNG CAO - CHUYÊN ĐỀ VỀ DÃY SỐ
TOÁN LỚP 5 NÂNG CAO - CHUYÊN ĐỀ VỀ DÃY SỐTOÁN LỚP 5 NÂNG CAO - CHUYÊN ĐỀ VỀ DÃY SỐ
TOÁN LỚP 5 NÂNG CAO - CHUYÊN ĐỀ VỀ DÃY SỐ
 
Bìa tập đại số tổ hợp
Bìa tập đại số tổ hợpBìa tập đại số tổ hợp
Bìa tập đại số tổ hợp
 
Tổng hợp các phương pháp giải phương trình và hệ phương trình môn toán
Tổng hợp các phương pháp giải phương trình và hệ phương trình môn toánTổng hợp các phương pháp giải phương trình và hệ phương trình môn toán
Tổng hợp các phương pháp giải phương trình và hệ phương trình môn toán
 
Phuong trinh vo ty
Phuong trinh vo tyPhuong trinh vo ty
Phuong trinh vo ty
 

Similar to Đề tài: Lớp bất đẳng thức, bài toán cực trị với đa thức đối xứng, 9đ

Luận văn: Một số lớp bài toán về loại phương trình hàm, HAY - Gửi miễn phí qu...
Luận văn: Một số lớp bài toán về loại phương trình hàm, HAY - Gửi miễn phí qu...Luận văn: Một số lớp bài toán về loại phương trình hàm, HAY - Gửi miễn phí qu...
Luận văn: Một số lớp bài toán về loại phương trình hàm, HAY - Gửi miễn phí qu...Dịch vụ viết bài trọn gói ZALO: 0909232620
 
108 bai toan chon loc lop 7
108 bai toan chon loc lop 7108 bai toan chon loc lop 7
108 bai toan chon loc lop 7leroben
 
06 mat102-bai 3-v1.0
06 mat102-bai 3-v1.006 mat102-bai 3-v1.0
06 mat102-bai 3-v1.0Yen Dang
 
Một số dạng toán về đa thức qua các kỳ thi Olympic 6732069.pdf
Một số dạng toán về đa thức qua các kỳ thi Olympic 6732069.pdfMột số dạng toán về đa thức qua các kỳ thi Olympic 6732069.pdf
Một số dạng toán về đa thức qua các kỳ thi Olympic 6732069.pdfTieuNgocLy
 
1 dạng bđt xoay vòng
1 dạng bđt xoay vòng1 dạng bđt xoay vòng
1 dạng bđt xoay vòngcuong4012
 
bo-de-tham-khao-giua-hoc-ky-2-toan-8-nam-2023-2024-phong-gddt-tp-hai-duong.pdf
bo-de-tham-khao-giua-hoc-ky-2-toan-8-nam-2023-2024-phong-gddt-tp-hai-duong.pdfbo-de-tham-khao-giua-hoc-ky-2-toan-8-nam-2023-2024-phong-gddt-tp-hai-duong.pdf
bo-de-tham-khao-giua-hoc-ky-2-toan-8-nam-2023-2024-phong-gddt-tp-hai-duong.pdfLinhTrnTh14
 
Luận văn: Đẳng thức và bất đẳng thức trong lớp hàm hyperbolic
Luận văn: Đẳng thức và bất đẳng thức trong lớp hàm hyperbolicLuận văn: Đẳng thức và bất đẳng thức trong lớp hàm hyperbolic
Luận văn: Đẳng thức và bất đẳng thức trong lớp hàm hyperbolicViết thuê trọn gói ZALO 0934573149
 
Do do tich-phan-thai_thuan_quang mearsure and intergral
Do do tich-phan-thai_thuan_quang mearsure and intergralDo do tich-phan-thai_thuan_quang mearsure and intergral
Do do tich-phan-thai_thuan_quang mearsure and intergralBui Loi
 
Tai lieu on thi tn thpt mon toan www.mathvn.com
Tai lieu on thi tn thpt mon toan   www.mathvn.comTai lieu on thi tn thpt mon toan   www.mathvn.com
Tai lieu on thi tn thpt mon toan www.mathvn.comtrongphuckhtn
 
Chuyên đề phương trình vô tỷ
Chuyên đề phương trình vô tỷChuyên đề phương trình vô tỷ
Chuyên đề phương trình vô tỷtuituhoc
 
Tuyen tap de dh 2002 2012 theo chu de
Tuyen tap de dh  2002 2012 theo chu deTuyen tap de dh  2002 2012 theo chu de
Tuyen tap de dh 2002 2012 theo chu deĐăng Trọng
 

Similar to Đề tài: Lớp bất đẳng thức, bài toán cực trị với đa thức đối xứng, 9đ (20)

Luận văn: Bất đẳng thức trong lớp hàm siêu việt, HAY, 9đ
Luận văn: Bất đẳng thức trong lớp hàm siêu việt, HAY, 9đLuận văn: Bất đẳng thức trong lớp hàm siêu việt, HAY, 9đ
Luận văn: Bất đẳng thức trong lớp hàm siêu việt, HAY, 9đ
 
Luận văn: Một số lớp bài toán về loại phương trình hàm, HAY - Gửi miễn phí qu...
Luận văn: Một số lớp bài toán về loại phương trình hàm, HAY - Gửi miễn phí qu...Luận văn: Một số lớp bài toán về loại phương trình hàm, HAY - Gửi miễn phí qu...
Luận văn: Một số lớp bài toán về loại phương trình hàm, HAY - Gửi miễn phí qu...
 
Luận văn: Một số lớp bài toán về phương trình hàm, HAY, 9đ
Luận văn: Một số lớp bài toán về phương trình hàm, HAY, 9đLuận văn: Một số lớp bài toán về phương trình hàm, HAY, 9đ
Luận văn: Một số lớp bài toán về phương trình hàm, HAY, 9đ
 
Luận văn: Một số phương pháp giải phương trình hàm, HOT, 9đ
Luận văn: Một số phương pháp giải phương trình hàm, HOT, 9đLuận văn: Một số phương pháp giải phương trình hàm, HOT, 9đ
Luận văn: Một số phương pháp giải phương trình hàm, HOT, 9đ
 
Luận văn: Phương pháp giải phương trình chứa ẩn dưới dấu căn
Luận văn: Phương pháp giải phương trình chứa ẩn dưới dấu cănLuận văn: Phương pháp giải phương trình chứa ẩn dưới dấu căn
Luận văn: Phương pháp giải phương trình chứa ẩn dưới dấu căn
 
108 bai toan chon loc lop 7
108 bai toan chon loc lop 7108 bai toan chon loc lop 7
108 bai toan chon loc lop 7
 
Luận văn: Phép biến đổi phân tuyến tính, HAY, 9đ
Luận văn: Phép biến đổi phân tuyến tính, HAY, 9đLuận văn: Phép biến đổi phân tuyến tính, HAY, 9đ
Luận văn: Phép biến đổi phân tuyến tính, HAY, 9đ
 
06 mat102-bai 3-v1.0
06 mat102-bai 3-v1.006 mat102-bai 3-v1.0
06 mat102-bai 3-v1.0
 
Luận văn: Một số phương pháp giải phương trình hàm, HOT, 9đ
Luận văn: Một số phương pháp giải phương trình hàm, HOT, 9đLuận văn: Một số phương pháp giải phương trình hàm, HOT, 9đ
Luận văn: Một số phương pháp giải phương trình hàm, HOT, 9đ
 
Luận văn: Giải hệ phương trình trong chương trình toán THPT, HOT
Luận văn: Giải hệ phương trình trong chương trình toán THPT, HOTLuận văn: Giải hệ phương trình trong chương trình toán THPT, HOT
Luận văn: Giải hệ phương trình trong chương trình toán THPT, HOT
 
Luận văn: Phương pháp giải hệ phương trình trong toán THPT
Luận văn: Phương pháp giải hệ phương trình trong toán THPTLuận văn: Phương pháp giải hệ phương trình trong toán THPT
Luận văn: Phương pháp giải hệ phương trình trong toán THPT
 
Một số dạng toán về đa thức qua các kỳ thi Olympic 6732069.pdf
Một số dạng toán về đa thức qua các kỳ thi Olympic 6732069.pdfMột số dạng toán về đa thức qua các kỳ thi Olympic 6732069.pdf
Một số dạng toán về đa thức qua các kỳ thi Olympic 6732069.pdf
 
1 dạng bđt xoay vòng
1 dạng bđt xoay vòng1 dạng bđt xoay vòng
1 dạng bđt xoay vòng
 
Luận văn thạc sĩ: Quy hoạch toàn phương, HAY, 9đ
Luận văn thạc sĩ: Quy hoạch toàn phương, HAY, 9đLuận văn thạc sĩ: Quy hoạch toàn phương, HAY, 9đ
Luận văn thạc sĩ: Quy hoạch toàn phương, HAY, 9đ
 
bo-de-tham-khao-giua-hoc-ky-2-toan-8-nam-2023-2024-phong-gddt-tp-hai-duong.pdf
bo-de-tham-khao-giua-hoc-ky-2-toan-8-nam-2023-2024-phong-gddt-tp-hai-duong.pdfbo-de-tham-khao-giua-hoc-ky-2-toan-8-nam-2023-2024-phong-gddt-tp-hai-duong.pdf
bo-de-tham-khao-giua-hoc-ky-2-toan-8-nam-2023-2024-phong-gddt-tp-hai-duong.pdf
 
Luận văn: Đẳng thức và bất đẳng thức trong lớp hàm hyperbolic
Luận văn: Đẳng thức và bất đẳng thức trong lớp hàm hyperbolicLuận văn: Đẳng thức và bất đẳng thức trong lớp hàm hyperbolic
Luận văn: Đẳng thức và bất đẳng thức trong lớp hàm hyperbolic
 
Do do tich-phan-thai_thuan_quang mearsure and intergral
Do do tich-phan-thai_thuan_quang mearsure and intergralDo do tich-phan-thai_thuan_quang mearsure and intergral
Do do tich-phan-thai_thuan_quang mearsure and intergral
 
Tai lieu on thi tn thpt mon toan www.mathvn.com
Tai lieu on thi tn thpt mon toan   www.mathvn.comTai lieu on thi tn thpt mon toan   www.mathvn.com
Tai lieu on thi tn thpt mon toan www.mathvn.com
 
Chuyên đề phương trình vô tỷ
Chuyên đề phương trình vô tỷChuyên đề phương trình vô tỷ
Chuyên đề phương trình vô tỷ
 
Tuyen tap de dh 2002 2012 theo chu de
Tuyen tap de dh  2002 2012 theo chu deTuyen tap de dh  2002 2012 theo chu de
Tuyen tap de dh 2002 2012 theo chu de
 

More from Dịch vụ viết bài trọn gói ZALO: 0909232620

Danh Sách 200 Đề Tài Tiểu Luận Chuyên Viên Chính Về Bảo Hiểm Xã Hội Mới Nhất
Danh Sách 200 Đề Tài Tiểu Luận Chuyên Viên Chính Về Bảo Hiểm Xã Hội Mới NhấtDanh Sách 200 Đề Tài Tiểu Luận Chuyên Viên Chính Về Bảo Hiểm Xã Hội Mới Nhất
Danh Sách 200 Đề Tài Tiểu Luận Chuyên Viên Chính Về Bảo Hiểm Xã Hội Mới NhấtDịch vụ viết bài trọn gói ZALO: 0909232620
 
Danh Sách 200 Đề Tài Báo Cáo Thực Tập Luật Phòng, Chống Hiv, Mới Nhất, Điểm Cao
Danh Sách 200 Đề Tài Báo Cáo Thực Tập Luật Phòng, Chống Hiv, Mới Nhất, Điểm CaoDanh Sách 200 Đề Tài Báo Cáo Thực Tập Luật Phòng, Chống Hiv, Mới Nhất, Điểm Cao
Danh Sách 200 Đề Tài Báo Cáo Thực Tập Luật Phòng, Chống Hiv, Mới Nhất, Điểm CaoDịch vụ viết bài trọn gói ZALO: 0909232620
 

More from Dịch vụ viết bài trọn gói ZALO: 0909232620 (20)

Danh Sách 200 Đề Tài Tiểu Luận Chuyên Viên Chính Về Bảo Hiểm Xã Hội Mới Nhất
Danh Sách 200 Đề Tài Tiểu Luận Chuyên Viên Chính Về Bảo Hiểm Xã Hội Mới NhấtDanh Sách 200 Đề Tài Tiểu Luận Chuyên Viên Chính Về Bảo Hiểm Xã Hội Mới Nhất
Danh Sách 200 Đề Tài Tiểu Luận Chuyên Viên Chính Về Bảo Hiểm Xã Hội Mới Nhất
 
Danh Sách 200 Đề Tài Luận Văn Thạc Sĩ Quản Trị Nguồn Nhân Lực, 9 Điểm
Danh Sách 200 Đề Tài Luận Văn Thạc Sĩ Quản Trị Nguồn Nhân Lực, 9 ĐiểmDanh Sách 200 Đề Tài Luận Văn Thạc Sĩ Quản Trị Nguồn Nhân Lực, 9 Điểm
Danh Sách 200 Đề Tài Luận Văn Thạc Sĩ Quản Trị Nguồn Nhân Lực, 9 Điểm
 
Danh Sách 200 Đề Tài Luận Văn Thạc Sĩ Quản Lý Văn Hóa Giúp Bạn Thêm Ý Tưởng
Danh Sách 200 Đề Tài Luận Văn Thạc Sĩ Quản Lý Văn Hóa Giúp Bạn Thêm Ý TưởngDanh Sách 200 Đề Tài Luận Văn Thạc Sĩ Quản Lý Văn Hóa Giúp Bạn Thêm Ý Tưởng
Danh Sách 200 Đề Tài Luận Văn Thạc Sĩ Quản Lý Văn Hóa Giúp Bạn Thêm Ý Tưởng
 
Danh Sách 200 Đề Tài Báo Cáo Thực Tập Quản Lý Giáo Dục Dễ Làm Điểm Cao
Danh Sách 200 Đề Tài Báo Cáo Thực Tập Quản Lý Giáo Dục Dễ Làm Điểm CaoDanh Sách 200 Đề Tài Báo Cáo Thực Tập Quản Lý Giáo Dục Dễ Làm Điểm Cao
Danh Sách 200 Đề Tài Báo Cáo Thực Tập Quản Lý Giáo Dục Dễ Làm Điểm Cao
 
Danh Sách 200 Đề Tài Báo Cáo Thực Tập Quan Hệ Lao Động Từ Sinh Viên Giỏi
Danh Sách 200 Đề Tài Báo Cáo Thực Tập Quan Hệ Lao Động Từ Sinh Viên GiỏiDanh Sách 200 Đề Tài Báo Cáo Thực Tập Quan Hệ Lao Động Từ Sinh Viên Giỏi
Danh Sách 200 Đề Tài Báo Cáo Thực Tập Quan Hệ Lao Động Từ Sinh Viên Giỏi
 
Danh Sách 200 Đề Tài Báo Cáo Thực Tập Nuôi Trồng Thủy Sản Dễ Làm Nhất
Danh Sách 200 Đề Tài Báo Cáo Thực Tập Nuôi Trồng Thủy Sản Dễ Làm NhấtDanh Sách 200 Đề Tài Báo Cáo Thực Tập Nuôi Trồng Thủy Sản Dễ Làm Nhất
Danh Sách 200 Đề Tài Báo Cáo Thực Tập Nuôi Trồng Thủy Sản Dễ Làm Nhất
 
Danh Sách 200 Đề Tài Báo Cáo Thực Tập Luật Sư, Mới Nhất, Điểm Cao
Danh Sách 200 Đề Tài Báo Cáo Thực Tập Luật Sư, Mới Nhất, Điểm CaoDanh Sách 200 Đề Tài Báo Cáo Thực Tập Luật Sư, Mới Nhất, Điểm Cao
Danh Sách 200 Đề Tài Báo Cáo Thực Tập Luật Sư, Mới Nhất, Điểm Cao
 
Danh Sách 200 Đề Tài Báo Cáo Thực Tập Luật Phòng, Chống Hiv, Mới Nhất, Điểm Cao
Danh Sách 200 Đề Tài Báo Cáo Thực Tập Luật Phòng, Chống Hiv, Mới Nhất, Điểm CaoDanh Sách 200 Đề Tài Báo Cáo Thực Tập Luật Phòng, Chống Hiv, Mới Nhất, Điểm Cao
Danh Sách 200 Đề Tài Báo Cáo Thực Tập Luật Phòng, Chống Hiv, Mới Nhất, Điểm Cao
 
Danh Sách 200 Đề Tài Báo Cáo Thực Tập Luật Phá Sản, Mới Nhất
Danh Sách 200 Đề Tài Báo Cáo Thực Tập Luật Phá Sản, Mới NhấtDanh Sách 200 Đề Tài Báo Cáo Thực Tập Luật Phá Sản, Mới Nhất
Danh Sách 200 Đề Tài Báo Cáo Thực Tập Luật Phá Sản, Mới Nhất
 
Danh Sách 200 Đề Tài Báo Cáo Thực Tập Luật Nhà Ở, Điểm Cao
Danh Sách 200 Đề Tài Báo Cáo Thực Tập Luật Nhà Ở, Điểm CaoDanh Sách 200 Đề Tài Báo Cáo Thực Tập Luật Nhà Ở, Điểm Cao
Danh Sách 200 Đề Tài Báo Cáo Thực Tập Luật Nhà Ở, Điểm Cao
 
Danh Sách 200 Đề Tài Báo Cáo Thực Tập Luật Ngân Hàng, Mới Nhất
Danh Sách 200 Đề Tài Báo Cáo Thực Tập Luật Ngân Hàng, Mới NhấtDanh Sách 200 Đề Tài Báo Cáo Thực Tập Luật Ngân Hàng, Mới Nhất
Danh Sách 200 Đề Tài Báo Cáo Thực Tập Luật Ngân Hàng, Mới Nhất
 
Danh Sách 200 Đề Tài Báo Cáo Thực Tập Luật Môi Trường, Mới Nhất
Danh Sách 200 Đề Tài Báo Cáo Thực Tập Luật Môi Trường, Mới NhấtDanh Sách 200 Đề Tài Báo Cáo Thực Tập Luật Môi Trường, Mới Nhất
Danh Sách 200 Đề Tài Báo Cáo Thực Tập Luật Môi Trường, Mới Nhất
 
Danh Sách 200 Đề Tài Báo Cáo Thực Tập Luật Hộ Tịch, Điểm Cao
Danh Sách 200 Đề Tài Báo Cáo Thực Tập Luật Hộ Tịch, Điểm CaoDanh Sách 200 Đề Tài Báo Cáo Thực Tập Luật Hộ Tịch, Điểm Cao
Danh Sách 200 Đề Tài Báo Cáo Thực Tập Luật Hộ Tịch, Điểm Cao
 
Danh Sách 200 Đề Tài Báo Cáo Thực Tập Luật Hình Sự , Dễ Làm Điểm Cao
Danh Sách 200 Đề Tài Báo Cáo Thực Tập Luật Hình Sự , Dễ Làm Điểm CaoDanh Sách 200 Đề Tài Báo Cáo Thực Tập Luật Hình Sự , Dễ Làm Điểm Cao
Danh Sách 200 Đề Tài Báo Cáo Thực Tập Luật Hình Sự , Dễ Làm Điểm Cao
 
Danh Sách 200 Đề Tài Báo Cáo Thực Tập Luật Hành Chính, Dễ Làm Điểm Cao
Danh Sách 200 Đề Tài Báo Cáo Thực Tập Luật Hành Chính, Dễ Làm Điểm CaoDanh Sách 200 Đề Tài Báo Cáo Thực Tập Luật Hành Chính, Dễ Làm Điểm Cao
Danh Sách 200 Đề Tài Báo Cáo Thực Tập Luật Hành Chính, Dễ Làm Điểm Cao
 
Danh Sách 200 Đề Tài Báo Cáo Thực Tập Luật Giáo Dục, Điểm Cao
Danh Sách 200 Đề Tài Báo Cáo Thực Tập Luật Giáo Dục, Điểm CaoDanh Sách 200 Đề Tài Báo Cáo Thực Tập Luật Giáo Dục, Điểm Cao
Danh Sách 200 Đề Tài Báo Cáo Thực Tập Luật Giáo Dục, Điểm Cao
 
Danh Sách 200 Đề Tài Báo Cáo Thực Tập Luật Đấu Thầu, Từ Sinh Viên Khá Giỏi
Danh Sách 200 Đề Tài Báo Cáo Thực Tập Luật Đấu Thầu, Từ Sinh Viên Khá GiỏiDanh Sách 200 Đề Tài Báo Cáo Thực Tập Luật Đấu Thầu, Từ Sinh Viên Khá Giỏi
Danh Sách 200 Đề Tài Báo Cáo Thực Tập Luật Đấu Thầu, Từ Sinh Viên Khá Giỏi
 
Danh Sách 200 Đề Tài Báo Cáo Thực Tập Luật Đầu Tư, Dễ Làm Điểm Cao
Danh Sách 200 Đề Tài Báo Cáo Thực Tập Luật Đầu Tư, Dễ Làm Điểm CaoDanh Sách 200 Đề Tài Báo Cáo Thực Tập Luật Đầu Tư, Dễ Làm Điểm Cao
Danh Sách 200 Đề Tài Báo Cáo Thực Tập Luật Đầu Tư, Dễ Làm Điểm Cao
 
Danh Sách 200 Đề Tài Báo Cáo Thực Tập Luật Đầu Tư Công, Dễ Làm Điểm Cao
Danh Sách 200 Đề Tài Báo Cáo Thực Tập Luật Đầu Tư Công, Dễ Làm Điểm CaoDanh Sách 200 Đề Tài Báo Cáo Thực Tập Luật Đầu Tư Công, Dễ Làm Điểm Cao
Danh Sách 200 Đề Tài Báo Cáo Thực Tập Luật Đầu Tư Công, Dễ Làm Điểm Cao
 
Danh Sách 200 Đề Tài Báo Cáo Thực Tập Luật Đất Đai, Từ Sinh Viên Khá Giỏi
Danh Sách 200 Đề Tài Báo Cáo Thực Tập Luật Đất Đai, Từ Sinh Viên Khá GiỏiDanh Sách 200 Đề Tài Báo Cáo Thực Tập Luật Đất Đai, Từ Sinh Viên Khá Giỏi
Danh Sách 200 Đề Tài Báo Cáo Thực Tập Luật Đất Đai, Từ Sinh Viên Khá Giỏi
 

Recently uploaded

xemsomenh.com-Vòng Tràng Sinh - Cách An 12 Sao Và Ý Nghĩa Từng Sao.pdf
xemsomenh.com-Vòng Tràng Sinh - Cách An 12 Sao Và Ý Nghĩa Từng Sao.pdfxemsomenh.com-Vòng Tràng Sinh - Cách An 12 Sao Và Ý Nghĩa Từng Sao.pdf
xemsomenh.com-Vòng Tràng Sinh - Cách An 12 Sao Và Ý Nghĩa Từng Sao.pdfXem Số Mệnh
 
xemsomenh.com-Vòng Thái Tuế và Ý Nghĩa Các Sao Tại Cung Mệnh.pdf
xemsomenh.com-Vòng Thái Tuế và Ý Nghĩa Các Sao Tại Cung Mệnh.pdfxemsomenh.com-Vòng Thái Tuế và Ý Nghĩa Các Sao Tại Cung Mệnh.pdf
xemsomenh.com-Vòng Thái Tuế và Ý Nghĩa Các Sao Tại Cung Mệnh.pdfXem Số Mệnh
 
Trắc nghiệm CHƯƠNG 5 môn Chủ nghĩa xã hội
Trắc nghiệm CHƯƠNG 5 môn Chủ nghĩa xã hộiTrắc nghiệm CHƯƠNG 5 môn Chủ nghĩa xã hội
Trắc nghiệm CHƯƠNG 5 môn Chủ nghĩa xã hộiNgocNguyen591215
 
Bài học phòng cháy chữa cháy - PCCC tại tòa nhà
Bài học phòng cháy chữa cháy - PCCC tại tòa nhàBài học phòng cháy chữa cháy - PCCC tại tòa nhà
Bài học phòng cháy chữa cháy - PCCC tại tòa nhàNguyen Thi Trang Nhung
 
Bài tập nhóm Kỹ Năng Gỉai Quyết Tranh Chấp Lao Động (1).pptx
Bài tập nhóm Kỹ Năng Gỉai Quyết Tranh Chấp Lao Động (1).pptxBài tập nhóm Kỹ Năng Gỉai Quyết Tranh Chấp Lao Động (1).pptx
Bài tập nhóm Kỹ Năng Gỉai Quyết Tranh Chấp Lao Động (1).pptxDungxPeach
 
SD-05_Xây dựng website bán váy Lolita Alice - Phùng Thị Thúy Hiền PH 2 7 8 6 ...
SD-05_Xây dựng website bán váy Lolita Alice - Phùng Thị Thúy Hiền PH 2 7 8 6 ...SD-05_Xây dựng website bán váy Lolita Alice - Phùng Thị Thúy Hiền PH 2 7 8 6 ...
SD-05_Xây dựng website bán váy Lolita Alice - Phùng Thị Thúy Hiền PH 2 7 8 6 ...ChuThNgnFEFPLHN
 
Kiến thức cơ bản về tư duy số - VTC Net Viet
Kiến thức cơ bản về tư duy số - VTC Net VietKiến thức cơ bản về tư duy số - VTC Net Viet
Kiến thức cơ bản về tư duy số - VTC Net VietNguyễn Quang Huy
 
powerpoint mẫu họp phụ huynh cuối kì 2 học sinh lớp 7 bgs
powerpoint mẫu họp phụ huynh cuối kì 2 học sinh lớp 7 bgspowerpoint mẫu họp phụ huynh cuối kì 2 học sinh lớp 7 bgs
powerpoint mẫu họp phụ huynh cuối kì 2 học sinh lớp 7 bgsNmmeomeo
 
ĐỀ CHÍNH THỨC KỲ THI TUYỂN SINH VÀO LỚP 10 THPT CÁC TỈNH THÀNH NĂM HỌC 2020 –...
ĐỀ CHÍNH THỨC KỲ THI TUYỂN SINH VÀO LỚP 10 THPT CÁC TỈNH THÀNH NĂM HỌC 2020 –...ĐỀ CHÍNH THỨC KỲ THI TUYỂN SINH VÀO LỚP 10 THPT CÁC TỈNH THÀNH NĂM HỌC 2020 –...
ĐỀ CHÍNH THỨC KỲ THI TUYỂN SINH VÀO LỚP 10 THPT CÁC TỈNH THÀNH NĂM HỌC 2020 –...Nguyen Thanh Tu Collection
 
TÀI LIỆU BỒI DƯỠNG HỌC SINH GIỎI KỸ NĂNG VIẾT ĐOẠN VĂN NGHỊ LUẬN XÃ HỘI 200 C...
TÀI LIỆU BỒI DƯỠNG HỌC SINH GIỎI KỸ NĂNG VIẾT ĐOẠN VĂN NGHỊ LUẬN XÃ HỘI 200 C...TÀI LIỆU BỒI DƯỠNG HỌC SINH GIỎI KỸ NĂNG VIẾT ĐOẠN VĂN NGHỊ LUẬN XÃ HỘI 200 C...
TÀI LIỆU BỒI DƯỠNG HỌC SINH GIỎI KỸ NĂNG VIẾT ĐOẠN VĂN NGHỊ LUẬN XÃ HỘI 200 C...Nguyen Thanh Tu Collection
 
Bài giảng môn Truyền thông đa phương tiện
Bài giảng môn Truyền thông đa phương tiệnBài giảng môn Truyền thông đa phương tiện
Bài giảng môn Truyền thông đa phương tiệnpmtiendhti14a5hn
 
xemsomenh.com-Vòng Lộc Tồn - Vòng Bác Sĩ và Cách An Trong Vòng Lộc Tồn.pdf
xemsomenh.com-Vòng Lộc Tồn - Vòng Bác Sĩ và Cách An Trong Vòng Lộc Tồn.pdfxemsomenh.com-Vòng Lộc Tồn - Vòng Bác Sĩ và Cách An Trong Vòng Lộc Tồn.pdf
xemsomenh.com-Vòng Lộc Tồn - Vòng Bác Sĩ và Cách An Trong Vòng Lộc Tồn.pdfXem Số Mệnh
 
SLIDE - Tu van, huong dan cong tac tuyen sinh-2024 (đầy đủ chi tiết).pdf
SLIDE - Tu van, huong dan cong tac tuyen sinh-2024 (đầy đủ chi tiết).pdfSLIDE - Tu van, huong dan cong tac tuyen sinh-2024 (đầy đủ chi tiết).pdf
SLIDE - Tu van, huong dan cong tac tuyen sinh-2024 (đầy đủ chi tiết).pdfhoangtuansinh1
 
3-BẢNG MÃ LỖI CỦA CÁC HÃNG ĐIỀU HÒA .pdf - ĐIỆN LẠNH BÁCH KHOA HÀ NỘI
3-BẢNG MÃ LỖI CỦA CÁC HÃNG ĐIỀU HÒA .pdf - ĐIỆN LẠNH BÁCH KHOA HÀ NỘI3-BẢNG MÃ LỖI CỦA CÁC HÃNG ĐIỀU HÒA .pdf - ĐIỆN LẠNH BÁCH KHOA HÀ NỘI
3-BẢNG MÃ LỖI CỦA CÁC HÃNG ĐIỀU HÒA .pdf - ĐIỆN LẠNH BÁCH KHOA HÀ NỘIĐiện Lạnh Bách Khoa Hà Nội
 
Giáo trình nhập môn lập trình - Đặng Bình Phương
Giáo trình nhập môn lập trình - Đặng Bình PhươngGiáo trình nhập môn lập trình - Đặng Bình Phương
Giáo trình nhập môn lập trình - Đặng Bình Phươnghazzthuan
 
26 Truyện Ngắn Sơn Nam (Sơn Nam) thuviensach.vn.pdf
26 Truyện Ngắn Sơn Nam (Sơn Nam) thuviensach.vn.pdf26 Truyện Ngắn Sơn Nam (Sơn Nam) thuviensach.vn.pdf
26 Truyện Ngắn Sơn Nam (Sơn Nam) thuviensach.vn.pdfltbdieu
 
TUYỂN TẬP 50 ĐỀ LUYỆN THI TUYỂN SINH LỚP 10 THPT MÔN TOÁN NĂM 2024 CÓ LỜI GIẢ...
TUYỂN TẬP 50 ĐỀ LUYỆN THI TUYỂN SINH LỚP 10 THPT MÔN TOÁN NĂM 2024 CÓ LỜI GIẢ...TUYỂN TẬP 50 ĐỀ LUYỆN THI TUYỂN SINH LỚP 10 THPT MÔN TOÁN NĂM 2024 CÓ LỜI GIẢ...
TUYỂN TẬP 50 ĐỀ LUYỆN THI TUYỂN SINH LỚP 10 THPT MÔN TOÁN NĂM 2024 CÓ LỜI GIẢ...Nguyen Thanh Tu Collection
 
các nội dung phòng chống xâm hại tình dục ở trẻ em
các nội dung phòng chống xâm hại tình dục ở trẻ emcác nội dung phòng chống xâm hại tình dục ở trẻ em
các nội dung phòng chống xâm hại tình dục ở trẻ emTrangNhung96
 
Đề thi tin học HK2 lớp 3 Chân Trời Sáng Tạo
Đề thi tin học HK2 lớp 3 Chân Trời Sáng TạoĐề thi tin học HK2 lớp 3 Chân Trời Sáng Tạo
Đề thi tin học HK2 lớp 3 Chân Trời Sáng Tạowindcances
 
30 ĐỀ PHÁT TRIỂN THEO CẤU TRÚC ĐỀ MINH HỌA BGD NGÀY 22-3-2024 KỲ THI TỐT NGHI...
30 ĐỀ PHÁT TRIỂN THEO CẤU TRÚC ĐỀ MINH HỌA BGD NGÀY 22-3-2024 KỲ THI TỐT NGHI...30 ĐỀ PHÁT TRIỂN THEO CẤU TRÚC ĐỀ MINH HỌA BGD NGÀY 22-3-2024 KỲ THI TỐT NGHI...
30 ĐỀ PHÁT TRIỂN THEO CẤU TRÚC ĐỀ MINH HỌA BGD NGÀY 22-3-2024 KỲ THI TỐT NGHI...Nguyen Thanh Tu Collection
 

Recently uploaded (20)

xemsomenh.com-Vòng Tràng Sinh - Cách An 12 Sao Và Ý Nghĩa Từng Sao.pdf
xemsomenh.com-Vòng Tràng Sinh - Cách An 12 Sao Và Ý Nghĩa Từng Sao.pdfxemsomenh.com-Vòng Tràng Sinh - Cách An 12 Sao Và Ý Nghĩa Từng Sao.pdf
xemsomenh.com-Vòng Tràng Sinh - Cách An 12 Sao Và Ý Nghĩa Từng Sao.pdf
 
xemsomenh.com-Vòng Thái Tuế và Ý Nghĩa Các Sao Tại Cung Mệnh.pdf
xemsomenh.com-Vòng Thái Tuế và Ý Nghĩa Các Sao Tại Cung Mệnh.pdfxemsomenh.com-Vòng Thái Tuế và Ý Nghĩa Các Sao Tại Cung Mệnh.pdf
xemsomenh.com-Vòng Thái Tuế và Ý Nghĩa Các Sao Tại Cung Mệnh.pdf
 
Trắc nghiệm CHƯƠNG 5 môn Chủ nghĩa xã hội
Trắc nghiệm CHƯƠNG 5 môn Chủ nghĩa xã hộiTrắc nghiệm CHƯƠNG 5 môn Chủ nghĩa xã hội
Trắc nghiệm CHƯƠNG 5 môn Chủ nghĩa xã hội
 
Bài học phòng cháy chữa cháy - PCCC tại tòa nhà
Bài học phòng cháy chữa cháy - PCCC tại tòa nhàBài học phòng cháy chữa cháy - PCCC tại tòa nhà
Bài học phòng cháy chữa cháy - PCCC tại tòa nhà
 
Bài tập nhóm Kỹ Năng Gỉai Quyết Tranh Chấp Lao Động (1).pptx
Bài tập nhóm Kỹ Năng Gỉai Quyết Tranh Chấp Lao Động (1).pptxBài tập nhóm Kỹ Năng Gỉai Quyết Tranh Chấp Lao Động (1).pptx
Bài tập nhóm Kỹ Năng Gỉai Quyết Tranh Chấp Lao Động (1).pptx
 
SD-05_Xây dựng website bán váy Lolita Alice - Phùng Thị Thúy Hiền PH 2 7 8 6 ...
SD-05_Xây dựng website bán váy Lolita Alice - Phùng Thị Thúy Hiền PH 2 7 8 6 ...SD-05_Xây dựng website bán váy Lolita Alice - Phùng Thị Thúy Hiền PH 2 7 8 6 ...
SD-05_Xây dựng website bán váy Lolita Alice - Phùng Thị Thúy Hiền PH 2 7 8 6 ...
 
Kiến thức cơ bản về tư duy số - VTC Net Viet
Kiến thức cơ bản về tư duy số - VTC Net VietKiến thức cơ bản về tư duy số - VTC Net Viet
Kiến thức cơ bản về tư duy số - VTC Net Viet
 
powerpoint mẫu họp phụ huynh cuối kì 2 học sinh lớp 7 bgs
powerpoint mẫu họp phụ huynh cuối kì 2 học sinh lớp 7 bgspowerpoint mẫu họp phụ huynh cuối kì 2 học sinh lớp 7 bgs
powerpoint mẫu họp phụ huynh cuối kì 2 học sinh lớp 7 bgs
 
ĐỀ CHÍNH THỨC KỲ THI TUYỂN SINH VÀO LỚP 10 THPT CÁC TỈNH THÀNH NĂM HỌC 2020 –...
ĐỀ CHÍNH THỨC KỲ THI TUYỂN SINH VÀO LỚP 10 THPT CÁC TỈNH THÀNH NĂM HỌC 2020 –...ĐỀ CHÍNH THỨC KỲ THI TUYỂN SINH VÀO LỚP 10 THPT CÁC TỈNH THÀNH NĂM HỌC 2020 –...
ĐỀ CHÍNH THỨC KỲ THI TUYỂN SINH VÀO LỚP 10 THPT CÁC TỈNH THÀNH NĂM HỌC 2020 –...
 
TÀI LIỆU BỒI DƯỠNG HỌC SINH GIỎI KỸ NĂNG VIẾT ĐOẠN VĂN NGHỊ LUẬN XÃ HỘI 200 C...
TÀI LIỆU BỒI DƯỠNG HỌC SINH GIỎI KỸ NĂNG VIẾT ĐOẠN VĂN NGHỊ LUẬN XÃ HỘI 200 C...TÀI LIỆU BỒI DƯỠNG HỌC SINH GIỎI KỸ NĂNG VIẾT ĐOẠN VĂN NGHỊ LUẬN XÃ HỘI 200 C...
TÀI LIỆU BỒI DƯỠNG HỌC SINH GIỎI KỸ NĂNG VIẾT ĐOẠN VĂN NGHỊ LUẬN XÃ HỘI 200 C...
 
Bài giảng môn Truyền thông đa phương tiện
Bài giảng môn Truyền thông đa phương tiệnBài giảng môn Truyền thông đa phương tiện
Bài giảng môn Truyền thông đa phương tiện
 
xemsomenh.com-Vòng Lộc Tồn - Vòng Bác Sĩ và Cách An Trong Vòng Lộc Tồn.pdf
xemsomenh.com-Vòng Lộc Tồn - Vòng Bác Sĩ và Cách An Trong Vòng Lộc Tồn.pdfxemsomenh.com-Vòng Lộc Tồn - Vòng Bác Sĩ và Cách An Trong Vòng Lộc Tồn.pdf
xemsomenh.com-Vòng Lộc Tồn - Vòng Bác Sĩ và Cách An Trong Vòng Lộc Tồn.pdf
 
SLIDE - Tu van, huong dan cong tac tuyen sinh-2024 (đầy đủ chi tiết).pdf
SLIDE - Tu van, huong dan cong tac tuyen sinh-2024 (đầy đủ chi tiết).pdfSLIDE - Tu van, huong dan cong tac tuyen sinh-2024 (đầy đủ chi tiết).pdf
SLIDE - Tu van, huong dan cong tac tuyen sinh-2024 (đầy đủ chi tiết).pdf
 
3-BẢNG MÃ LỖI CỦA CÁC HÃNG ĐIỀU HÒA .pdf - ĐIỆN LẠNH BÁCH KHOA HÀ NỘI
3-BẢNG MÃ LỖI CỦA CÁC HÃNG ĐIỀU HÒA .pdf - ĐIỆN LẠNH BÁCH KHOA HÀ NỘI3-BẢNG MÃ LỖI CỦA CÁC HÃNG ĐIỀU HÒA .pdf - ĐIỆN LẠNH BÁCH KHOA HÀ NỘI
3-BẢNG MÃ LỖI CỦA CÁC HÃNG ĐIỀU HÒA .pdf - ĐIỆN LẠNH BÁCH KHOA HÀ NỘI
 
Giáo trình nhập môn lập trình - Đặng Bình Phương
Giáo trình nhập môn lập trình - Đặng Bình PhươngGiáo trình nhập môn lập trình - Đặng Bình Phương
Giáo trình nhập môn lập trình - Đặng Bình Phương
 
26 Truyện Ngắn Sơn Nam (Sơn Nam) thuviensach.vn.pdf
26 Truyện Ngắn Sơn Nam (Sơn Nam) thuviensach.vn.pdf26 Truyện Ngắn Sơn Nam (Sơn Nam) thuviensach.vn.pdf
26 Truyện Ngắn Sơn Nam (Sơn Nam) thuviensach.vn.pdf
 
TUYỂN TẬP 50 ĐỀ LUYỆN THI TUYỂN SINH LỚP 10 THPT MÔN TOÁN NĂM 2024 CÓ LỜI GIẢ...
TUYỂN TẬP 50 ĐỀ LUYỆN THI TUYỂN SINH LỚP 10 THPT MÔN TOÁN NĂM 2024 CÓ LỜI GIẢ...TUYỂN TẬP 50 ĐỀ LUYỆN THI TUYỂN SINH LỚP 10 THPT MÔN TOÁN NĂM 2024 CÓ LỜI GIẢ...
TUYỂN TẬP 50 ĐỀ LUYỆN THI TUYỂN SINH LỚP 10 THPT MÔN TOÁN NĂM 2024 CÓ LỜI GIẢ...
 
các nội dung phòng chống xâm hại tình dục ở trẻ em
các nội dung phòng chống xâm hại tình dục ở trẻ emcác nội dung phòng chống xâm hại tình dục ở trẻ em
các nội dung phòng chống xâm hại tình dục ở trẻ em
 
Đề thi tin học HK2 lớp 3 Chân Trời Sáng Tạo
Đề thi tin học HK2 lớp 3 Chân Trời Sáng TạoĐề thi tin học HK2 lớp 3 Chân Trời Sáng Tạo
Đề thi tin học HK2 lớp 3 Chân Trời Sáng Tạo
 
30 ĐỀ PHÁT TRIỂN THEO CẤU TRÚC ĐỀ MINH HỌA BGD NGÀY 22-3-2024 KỲ THI TỐT NGHI...
30 ĐỀ PHÁT TRIỂN THEO CẤU TRÚC ĐỀ MINH HỌA BGD NGÀY 22-3-2024 KỲ THI TỐT NGHI...30 ĐỀ PHÁT TRIỂN THEO CẤU TRÚC ĐỀ MINH HỌA BGD NGÀY 22-3-2024 KỲ THI TỐT NGHI...
30 ĐỀ PHÁT TRIỂN THEO CẤU TRÚC ĐỀ MINH HỌA BGD NGÀY 22-3-2024 KỲ THI TỐT NGHI...
 

Đề tài: Lớp bất đẳng thức, bài toán cực trị với đa thức đối xứng, 9đ

  • 1. ĐẠI HỌC QUỐC GIA HÀ NỘI TRƯỜNG ĐẠI HỌC KHOA HỌC TỰ NHIÊN - - - - - - - - - - - - - - - - - - - - - - - NGUYỄN TÀI TUỆ MỘT SỐ LỚP BẤT ĐẲNG THỨC VÀ BÀI TOÁN CỰC TRỊ VỚI ĐA THỨC ĐỐI XỨNG BA BIẾN Chuyên ngành: PHƯƠNG PHÁP TOÁN SƠ CẤP Mã số: 60.46.01.13 LUẬN VĂN THẠC SỸ KHOA HỌC NGƯỜI HƯỚNG DẪN KHOA HỌC: GS.TSKH. NGUYỄN VĂN MẬU Hà Nội – Năm 2014
  • 2. Mục lục MỞ ĐẦU 3 1 Một số kiến thức bổ trợ 5 1.1 Đa thức đối xứng ba biến . . . . . . . . . . . . . . . . . . . . . . . 5 1.2 Tính chất cơ bản của bất đẳng thức . . . . . . . . . . . . . . . . . 6 1.3 Bất đẳng thức thường dùng . . . . . . . . . . . . . . . . . . . . . . 6 1.3.1 Bất đẳng thức AM-GM . . . . . . . . . . . . . . . . . . . . 6 1.3.2 Bất đẳng thức Cauchy - Schwarz . . . . . . . . . . . . . . . 7 1.3.3 Bất đẳng thức Karamata . . . . . . . . . . . . . . . . . . . 7 2 Bất đẳng thức có tổng các biến không đổi 9 2.1 Bất đẳng thức có tổng các biến không đổi với hàm phân thức hữu tỉ 9 2.1.1 Sử dụng bất đẳng thức AM-GM . . . . . . . . . . . . . . . 9 2.1.2 Sử dụng bất đẳng thức Cauchy-Schwarz . . . . . . . . . . . 15 2.1.3 Sử dụng các tính chất của hàm số . . . . . . . . . . . . . . 21 2.1.4 Bài toán liên quan . . . . . . . . . . . . . . . . . . . . . . . 31 2.2 Bất đẳng thức có tổng các biến không đổi với hàm vô tỉ . . . . . . 33 2.2.1 Sử dụng bất đẳng thức AM-GM . . . . . . . . . . . . . . . 33 2.2.2 Sử dụng bất đẳng thức Cauchy-Schwarz . . . . . . . . . . . 36 2.2.3 Sử dụng các tính chất của hàm số . . . . . . . . . . . . . . 41 2.2.4 Bài toán liên quan . . . . . . . . . . . . . . . . . . . . . . . 43 3 Bất đẳng thức có tích các biến không đổi 45 3.1 Bất đẳng thức có tích các biến không đổi với hàm phân thức hữu tỉ 45 3.1.1 Sử dụng bất đẳng thức AM-GM . . . . . . . . . . . . . . . 45 3.1.2 Sử dụng bất đẳng thức Cauchy-Schwarz . . . . . . . . . . . 50 3.1.3 Sử dụng các tính chất của hàm số . . . . . . . . . . . . . . 53 3.1.4 Bài toán liên quan . . . . . . . . . . . . . . . . . . . . . . . 55 3.2 Bất đẳng thức có tích các biến không đổi với hàm vô tỉ . . . . . . 56 3.2.1 Sử dụng bất đẳng thức AM-GM . . . . . . . . . . . . . . . 56 3.2.2 Sử dụng bất đẳng thức Cauchy-Schwarz . . . . . . . . . . . 59 3.2.3 Sử dụng các tính chất của hàm số . . . . . . . . . . . . . . 60 3.2.4 Bài toán liên quan . . . . . . . . . . . . . . . . . . . . . . . 62 1
  • 3. MỤC LỤC 4 Một số lớp bài toán cực trị với đa thức đối xứng ba biến 63 4.1 Sử dụng bất đẳng thức AM-GM . . . . . . . . . . . . . . . . . . . 63 4.2 Sử dụng bất đẳng thức Cauchy-Schwarz . . . . . . . . . . . . . . . 68 4.3 Sử dụng các tính chất của hàm số . . . . . . . . . . . . . . . . . . . 73 4.4 Bài toán liên quan . . . . . . . . . . . . . . . . . . . . . . . . . . . . 77 KẾT LUẬN 78 TÀI LIỆU THAM KHẢO 79 2
  • 4. MỞ ĐẦU Bất đẳng thức là một nội dung cổ điển và quan trọng của Toán học. Ngay từ đầu, sự ra đời và phát triển của bất đẳng thức đã đặt dấu ấn quan trọng, chúng có sức hút mạnh mẽ đối với những người yêu toán, không chỉ ở vẻ đẹp hình thức mà cả những bí ẩn nó mang đến luôn thôi thúc người làm toán phải tìm tòi, sáng tạo. Bất đẳng thức còn có nhiều ứng dụng trong các môn khoa học khác và trong thực tế. Ngày nay, bất đẳng thức vẫn luôn chiếm một vai trò quan trọng và vẫn thường xuất hiện trong các kì thi quốc gia, quốc tế, Olympic. Là một giáo viên THPT, tôi muốn nghiên cứu sâu hơn về bất đẳng thức nhằm nâng cao chuyên môn phục vụ cho quá trình giảng dạy và bồi dưỡng học sinh giỏi, vậy nên tôi đã chọn bất đẳng thức làm luận văn thạc sĩ của mình. Bất đẳng thức vô cùng rộng lớn, trong thời gian ngắn, tôi chỉ có thể nghiên cứu lĩnh vực nhỏ trong đó. Dưới sự hướng dẫn của GS. TSKH Nguyễn Văn Mậu, tác giả đã hoàn thành luận văn với để tài "Một số lớp bất đẳng thức và bài toán cực trị với đa thức đối xứng ba biến." Luận văn được chia làm bốn chương: • Chương 1: Một số kiến thức bổ trợ. • Chương 2: Bất đẳng thức với tổng không đổi. • Chương 3: Bất đẳng thức có tích không đổi. • Chương 4: Một số lớp bài toán cực trị với đa thức đối xứng ba biến. Mặc dù có nhiều cố gắng, song do thời gian và trình độ còn hạn chế nên luận văn khó tránh khỏi những thiếu sót. Vì vậy tác giả rất mong nhận được sự góp ý của các thầy cô và các bạn để luận văn được hoàn thiện hơn. Qua luận văn này, tác giả xin bày tỏ lòng biết ơn sâu sắc đến GS.TSKH Nguyễn Văn Mậu, người Thầy đã truyền cho tác giả có niềm say mê nghiên cứu 3
  • 5. MỞ ĐẦU toán học. Thầy đã tận tình hướng dẫn, giúp đỡ tác giả trong suốt quá trình học tập và hoàn thiện luận văn này. Tác giả xin chân thành cảm ơn Ban giám hiệu, Phòng Đào tạo Sau đại học, Khoa Toán- Cơ - Tin, các thầy cô đã tạo điều kiện thuận lợi cho em hoàn thành bản luận văn này. Em xin chân thành cảm ơn! Hà Nội, ngày 01 tháng 12 năm 2014 Tác giả 4
  • 6. Chương 1 Một số kiến thức bổ trợ 1.1 Đa thức đối xứng ba biến 1.1.1 Các khái niệm cơ bản Định nghĩa 1.1. Một đơn thức ϕ(x, y, z) của các biến x, y, z được hiểu là hàm số có dạng ϕ(x, y, z) = aklmxk yl zm , trong đó k, l, m ∈ N được gọi là bậc của biến x, y, z, số aklm ∈ R∗ = R{0} được gọi là hệ số của đơn thức, còn số k +l+m được gọi là bậc của đơn thức ϕ(x, y, z). Định nghĩa 1.2. Một hàm số P(x, y, z) của các biến x, y, z được gọi là một đa thức nếu nó có thể được biểu diễn ở dạng tổng hữu hạn các đơn thức P(x, y, z) = k,l,m∈N k+l+m=n aklmxk yl zm , n ∈ N. Bậc lớn nhất của các đơn thức trong đa thức được gọi là bậc của đa thức. Định nghĩa 1.3. Đa thức P(x, y, z) được gọi là đối xứng, nếu nó không thay đổi với mọi hoán vị của x, y, z, nghĩa là P(x, y, z) = P(y, x, z) = P(z, y, x) = P(x, z, y). Định nghĩa 1.4. Đa thức f(x, y, z) được gọi là thuần nhất bậc m, nếu f(tx, ty, tz) = tm f(x, y, z), t = 0 Định nghĩa 1.5. Các đa thức σ1 = x + y + z, σ2 = xy + yz + zx, σ3 = xyz, được gọi là đa thức đối xứng cơ sở của các biến x, y, z. 1.1.2 Tổng lũy thừa 5
  • 7. Chương 1. Một số kiến thức bổ trợ Định nghĩa 1.6. Các đa thức sk = xk + yk + zk, (k = 0, 1, ...), được gọi là tổng lũy thừa bậc k của các biến x, y, z. Định lý 1.1 ( Công thức Newton). Với mọi k ∈ Z, ta có hệ thức sk = σ1sk−1 − σ2sk−2 + σ3sk−3. Định lý 1.2. Một tổng lũy thừa sk = xk + yk + zk đều có thể biểu diễn được dưới dạng một đa thức bậc n theo các biến σ1, σ2, σ3. Định lý 1.3 (Công thức Waring). Tổng lũy thừa sk được biểu diễn qua cá đa thức đối xứng cở sở theo công thức sk k = 0≤l,m,n l+2m+3n=k (−1)k−l−m−n(l + m + n − 1)! l!m!n! σl 1σm 2 σn 3 . 1.2 Tính chất cơ bản của bất đẳng thức 1. a > b ⇔ a + c > b + c. 2. a > b, b > c thì a > c. 3. a > b thì ca > cb khi c > 0 ca < cb khi c < 0. 4. a > b, c > d thì a + c > b + d. 5. a > b > 0, c > d > 0 thì ac > bd. 6. Với n nguyên dương, ta có a < b ⇔ a2n+1 < b2n+1 0 < a < b ⇒ a2n < b2n . 1.3 Bất đẳng thức thường dùng 1.3.1 Bất đẳng thức AM-GM Định lý 1.4. Giả sử a1, a2, . . . , an là các số thực không âm, khi đó ta luôn có a1 + a2 + · · · + an n ≥ n √ a1a2 . . . an. Đẳng thức xảy ra khi và chỉ khi a1 = a2 = · · · = an. 6
  • 8. Chương 1. Một số kiến thức bổ trợ Hệ quả 1.1. Với mọi số thực dương a1, a2, . . . , an ta có 1 a1 + 1 a2 + · · · + 1 an (a1 + a2 + · · · + an) ≥ n2 . Đẳng thức xảy ra khi và chỉ khi a1 = a2 = · · · = an. Hệ quả 1.2. Với mọi số thực a, b, c, ta luôn có 1. a2 + b2 + c2 ≥ ab + bc + ca 2. a2 + b2 + c2 ≥ (a + b + c)2 3 3. (a + b + c)2 ≥ 3(ab + bc + ca) 4. a2b2 + b2c2 + c2a2 ≥ abc(a + b + c) 5. (ab + bc + ca)2 ≥ 3abc(a + b + c). 1.3.2 Bất đẳng thức Cauchy - Schwarz Định lý 1.5. Nếu a1, a2, . . . , an, b1, b2, . . . , bn là các số thực tùy ý thì (a1b1 + a2b2 + · · · + anbn)2 ≤ a2 1 + a2 2 + · · · + a2 n b2 1 + b2 2 + · · · + b2 n . ( ) Đẳng thức xảy ra khi và chỉ khi a1 b1 = a2 b2 = · · · = an bn ( ở đây ta sử dụng quy ước nếu mẫu bằng 0 thì tử cũng bằng 0). Nhận xét 1.1. Theo bất đẳng thức ( ), chọn ai = xi √ yi và bi = √ yi với xi, yi ∈ R, yi > 0. Ta thu được bất đẳng thức Cauchy-Schwarz dạng phân thức ( hay còn gọi là bất đẳng thức Cauchy-Schwarz dạng Engel). Hệ quả 1.3. Nếu x1, x2, . . . , xn là các số thực và y1, y2, . . . , yn là các số thực dương thì x2 1 y1 + x2 2 y2 + · · · + x2 n yn ≥ (x1 + x2 + . . . xn)2 y1 + y2 + · · · + yn . Đẳng thức xảy ra khi và chỉ khi x1 y1 = x2 y2 = · · · = xn yn . 1.3.3 Bất đẳng thức Karamata Định lý 1.6. Cho hai dãy số {xk, yk ∈ I(a, b), k = 1, 2, . . . , n}, thỏa mãn điều kiện x1 ≥ x2 ≥ · · · ≥ xn, y1 ≥ y2 ≥ · · · ≥ yn 7
  • 9. Chương 1. Một số kiến thức bổ trợ và    x1 ≥ y1 x1 + x2 ≥ y1 + y2 ........ x1 + x2 + · · · + xn−1 ≥ y1 + y2 + · · · + yn−1 x1 + x2 + · · · + xn = y1 + y2 + · · · + yn Khi đó, ứng với hàm số lồi f(x)(f (x) ≥ 0) trên I(a, b), ta đều có f(x1) + f(x2) + · · · + f(xn) ≥ f(y1) + f(y2) + · · · + f(yn). Đẳng thức xảy ra khi và chỉ khi xi = yi, i = 1, 2, . . . n. Ta cũng phát biểu tương tự đối với hàm số lõm bằng cách đổi chiều dấu bất đẳng thức. Bổ đề 1.1. Cho hàm số y = f(x) liên tục và có đạo hàm cấp 2 trên I(a; b). a. Nếu f (x) ≥ 0, ∀x ∈ I(a; b) thì f(x) ≥ f (x0)(x − x0) + f(x0), ∀x0 ∈ I(a; b). b. Nếu f (x) ≤ 0, ∀x ∈ I(a; b) thì f(x) ≤ f (x0)(x − x0) + f(x0), ∀x0 ∈ I(a; b). Đẳng thức trong hai bất đẳng thức trên xảy ra khi và chỉ khi x = x0. 8
  • 10. Chương 2 Bất đẳng thức có tổng các biến không đổi 2.1 Bất đẳng thức có tổng các biến không đổi với hàm phân thức hữu tỉ 2.1.1 Sử dụng bất đẳng thức AM-GM Đối với bất đẳng thức P(x, y, z) ≥ 0 (≤ 0), Trong đó P(x, y, z) là đa thức hoặc phân thức hữu tỉ và có tổng x + y + z không đổi, thì khi đó sử dụng các kĩ thuật của bất đẳng thức AM − GM như dự đoán dấu bằng xảy ra, AM − GM ngược dấu, đặt ẩn phụ, ... tỏ ra rất hiệu quả. Bài toán 2.1. Với a, b, c là các số thực dương thỏa mãn a + b + c = 3. Chứng minh rằng a2 b + 2 + b2 c + 2 + c2 a + 2 ≥ 1. Chứng minh. Áp dụng bất đẳng thức AM-GM, ta có a2 b + 2 + b + 2 9 ≥ 2a 3 b2 c + 2 + c + 2 9 ≥ 2b 3 c2 a + 2 + a + 2 9 ≥ 2c 3 . Cộng các bất đẳng thức cùng chiều ta được a2 b + 2 + b2 c + 2 + c2 a + 2 ≥ 5 9 (a + b + c) − 2 3 = 1. Đẳng thức xảy ra khi và chỉ khi a = b = c = 1. 9
  • 11. Chương 2. Bất đẳng thức có tổng các biến không đổi Bài toán 2.2. Với a, b, c là các số thực dương thỏa mãn a + b + c = 3. Chứng minh rằng 2(ab + bc + ca) + 1 ab + 1 bc + 1 ca ≥ 9. Chứng minh. Cộng cả hai vế của bất đẳng thức cần chứng minh với a2 + b2 + c2, ta được a2 + b2 + c2 + 2(ab + bc + ca) + 1 ab + 1 bc + 1 ca ≥ 9 + a2 + b2 + c2 ⇔(a + b + c)2 + 1 ab + 1 bc + 1 ca ≥ 9 + a2 + b2 + c2 ⇔ 1 ab + 1 bc + 1 ca ≥ a2 + b2 + c2 ⇔ a + b + c abc ≥ a2 + b2 + c2 ⇔ 3 abc ≥ a2 + b2 + c2 ⇔abc(a2 + b2 + c2 ) ≤ 3 ⇔abc(a + b + c)(a2 + b2 + c2 ) ≤ 9. Ta có abc(a + b + c)(a2 + b2 + c2 ) ≤ (ab + bc + ca)2 3 (a2 + b2 + c2 ). Do đó ta chứng minh (ab + bc + ca)2 3 (a2 + b2 + c2 ) ≤ 9 ⇔ (ab + bc + ca)2 (a2 + b2 + c2 ) ≤ 27. Áp dụng bất đẳng thức AM-GM, ta có (ab + bc + ca)2 (a2 + b2 + c2 ) ≤ (ab + bc + ca) + (ab + bc + ca) + a2 + b2 + c2 3 3 = (a + b + c)2 3 3 = 27. Vậy bất đẳng thức được chứng minh. Đẳng thức xảy ra khi và chỉ khi a = b = c = 1. Bài toán 2.3. Với các số thực không âm a, b, c thỏa mãn a + b + c = 3. Chứng minh rằng 2 a2 b + b2 c + c2 a + 3 a2 + b2 + c2 + 4abc ≥ 19. Chứng minh. 10
  • 12. Chương 2. Bất đẳng thức có tổng các biến không đổi Ta có 19 = 3 (a + b + c)2 − 8 = 3 a2 + b2 + c2 + 6 (ab + bc + ca) − 8. Khi đó bất đẳng thức cần chứng minh trở thành 2 a2 b + b2 c + c2 a + 4abc ≥ 6 (ab + bc + ca) − 8 ⇔a2 b + b2 c + c2 a + 2abc + 4 ≥ 3 (ab + bc + ca) . Mà ta lại có 3 (ab + bc + ca) = (a + b + c) (ab + bc + ca) = a2 b + b2 c + c2 a + ab2 + bc2 + ca2 + 3abc. Do đó bất đẳng thức cần chứng minh tương đương với 4 ≥ ab2 + bc2 + ca2 + abc. Không mất tính tổng quát giả sử a ≥ b ≥ c ≥ 0 ⇒ (b − a) (b − c) ≤ 0 ⇔b2 − bc − ab + ac ≤ 0 ⇔ab2 + a2 c ≤ a2 b + abc. Như vậy ab2 + bc2 + ca2 + abc ≤ bc2 + abc + a2 b + abc = bc2 + 2abc + a2 b = b (a + c)2 = 4b a + c 2 a + c 2 ≤ 4   b + a + c 2 + a + c 2 3   3 = 4. Vậy bất đẳng thức được chứng minh. Đẳng thức xảy ra khi và chỉ khi a = b = c = 1. 11
  • 13. Chương 2. Bất đẳng thức có tổng các biến không đổi Bài toán 2.4 (Bulgaria TST 2003). Với ba số thực dương a, b, c thỏa mãn điều kiện a + b + c = 3. Chứng minh rằng a 1 + b2 + b 1 + c2 + c 1 + a2 ≥ 3 2 . Chứng minh. Áp dụng bất đẳng thức AM-GM, ta có a 1 + b2 = a − ab2 1 + b2 ≥ a − ab2 2b = a − ab 2 . Tương tự, ta có b 1 + c2 ≥ b − bc 2 c a2 + 1 ≥ c − ac 2 . Cộng vế với vế, ta có a 1 + b2 + b 1 + c2 + c 1 + a2 ≥ a + b + c − 1 2 (ab + bc + ca) = 3 − ab + bc + ca 2 . Mặt khác, ta có ab + bc + ca ≤ (a + b + c)2 3 = 3 do đó a 1 + b2 + b 1 + c2 + c 1 + a2 ≥ 3 − ab + bc + ca 2 ≥ 3 2 . Đẳng thức xảy ra khi và chỉ khi a = b = c = 1. Bài toán 2.5 (Turkey TST 2007). Với a, b, c là các số thực dương thỏa mãn a + b + c = 1. Chứng minh rằng 1 ab + 2c2 + 2c + 1 bc + 2a2 + 2a + 1 ca + 2b2 + 2b ≥ 1 ab + bc + ca . Chứng minh. Áp dụng bất đẳng thức AM-GM, ta có 12
  • 14. Chương 2. Bất đẳng thức có tổng các biến không đổi ab + 2c2 + 2c = ab + 2c2 + 2c(a + b + c) = ab + 4c2 + 2ac + 2bc = (b + 2c)(a + 2c) = a(b + 2c)b(a + 2c) ab = (ab + 2ac)(ab + 2bc) ab ≤ [ab + 2ac + ab + 2bc]2 4ab = (ab + bc + ca)2 ab từ đó suy ra 1 ab + 2c2 + 2c ≥ ab (ab + bc + ca)2 . Tương tự, ta có 1 bc + 2a2 + 2a ≥ bc (ab + bc + ca)2 1 ca + 2b2 + 2b ≥ ca (ab + bc + ca)2 . Cộng các bất đẳng thức cùng chiều, ta có 1 ab + 2c2 + 2c + 1 bc + 2a2 + 2a + 1 ca + 2b2 + 2b ≥ 1 ab + bc + ca . Đẳng thức xảy ra khi và chỉ khi a = b = c = 1 3 . Bài toán 2.6. Với các số thực dương a, b, c thỏa mãn a + b + c = 3. Chứng minh rằng a2 + bc b + ca + b2 + ca c + ab + c2 + ab a + bc ≥ 3. Chứng minh. Bất đẳng thức cần chứng minh tương đương với a2 + bc 3b + 3ca + b2 + ca 3c + 3ab + c2 + ab 3a + 3bc ≥ 1 ⇔ a2 + bc b(a + b + c) + 3ca + b2 + ca c(a + b + c) + 3ab + c2 + ab a(a + b + c) + 3bc ≥ 1. Mặt khác 3b + 3ac = b(a + b + c) + 3ac ≤ b(a + b + c) + ac + a2 + c2 = a2 + b2 + c2 + ab + bc + ca 13
  • 15. Chương 2. Bất đẳng thức có tổng các biến không đổi do đó a2 + bc 3b + 3ac ≥ a2 + bc a2 + b2 + c2 + ab + bc + ca . Tương tự, ta có b2 + ca 3c + 3ab ≥ b2 + ca a2 + b2 + c2 + ab + bc + ca c2 + ab 3a + 3bc ≥ c2 + ab a2 + b2 + c2 + ab + bc + ca . Cộng các bất đẳng thức cùng chiều, ta có a2 + bc 3b + 3ca + b2 + ca 3c + 3ab + c2 + ab 3a + 3bc ≥ 1. Bất đẳng thức được chứng minh. Đẳng thức xảy ra khi và chỉ khi a = b = c = 1. 14
  • 16. Chương 2. Bất đẳng thức có tổng các biến không đổi 2.1.2 Sử dụng bất đẳng thức Cauchy-Schwarz Nếu yêu cầu của bất đẳng thức là chứng minh P(x, y, z) ≥ 0(≤ 0) với P(x, y, z) có dạng tổng các bình phương hoặc phân thức với tử số của mỗi phân thức có dạng bình phương như vậy ta có thể nghĩ đến việc áp dụng bất đẳng thức Cauchy- Schwarz. Hoặc sử dụng giả thiết x + y + z không đổi ta có thể biến đổi để đưa bất đẳng thức về dạng trên để sử dụng Cauchy-Schwarz. Bài toán 2.7. Với các số thực dương a, b, c thỏa mãn điều kiện a + b + c = 1. Chứng minh rằng a 1 + b − a + b 1 + c − b + c 1 + a − c ≥ 1. Chứng minh. Ta luôn có a 1 + b − a + b 1 + c − b + c 1 + a − c = a 2b + c + b 2c + a + c 2a + b = a2 2ab + ac + b2 2bc + ab + c2 2ac + bc . Áp dụng bất đẳng thức Cauchy-Schwarz, ta có a2 2ab + ac + b2 2bc + ab + c2 2ac + bc ≥ (a + b + c)2 3 (ab + bc + ca) . Theo hệ quả của bất đẳng thức AM-GM, ta có (a + b + c)2 ≥ 3(ab + bc + ca) do đó a2 2ab + ac + b2 2bc + ab + c2 2ac + bc ≥ (a + b + c)2 3 (ab + bc + ca) ≥ 1. Đẳng thức xảy ra khi và chỉ khi a = b = c = 1. Bài toán 2.8. Với các số thực dương a, b, c sao cho a + b + c = 1. Chứng minh 1 a2 + b2 + c2 + 1 ab + 1 bc + 1 ca ≥ 30. Chứng minh. Áp dụng bất đẳng thức Cauchy-Schwarz cho hai bộ số 1 √ a2 + b2 + c2 , 1 √ ab , 1 √ bc , 1 √ cd 15
  • 17. Chương 2. Bất đẳng thức có tổng các biến không đổi và a2 + b2 + c2, 3 √ ab, 3 √ bc, 3 √ ca ta có 1 a2 + b2 + c2 + 1 ab + 1 bc + 1 ca a2 + b2 + c2 + 9ab + 9bc + 9ca ≥ (1 + 3 + 3 + 3)2 ⇔ 1 a2 + b2 + c2 + 1 ab + 1 bc + 1 ca (a + b + c)2 + 7(ab + bc + ca) ≥ 100 ⇔ 1 a2 + b2 + c2 + 1 ab + 1 bc + 1 ca ≥ 100 (a + b + c)2 + 7(ab + bc + ca) . Theo hệ quả của bất đẳng thức AM − GM, ta có (a + b + c)2 3 ≥ ab + bc + ca nên 1 a2 + b2 + c2 + 1 ab + 1 bc + 1 ca ≥ 100 1 + 7 (a + b + c)2 3 = 30. Đẳng thức xảy ra khi và chỉ khi a = b = c = 1 3 . Bài toán 2.9 (Iran MO TST 2009). Với các số thực dương a, b, c thỏa mãn điều kiện a + b + c = 3. Chứng minh rằng 1 a2 + b2 + 2 + 1 b2 + c2 + 2 + 1 c2 + a2 + 2 ≤ 3 4 . Chứng minh. Bất đẳng thức cần chứng minh ⇔ 1 2 − 1 a2 + b2 + 2 + 1 2 − 1 b2 + c2 + 2 + 1 2 − 1 c2 + a2 + 2 ≥ 3 2 − 3 4 ⇔ a2 + b2 a2 + b2 + 2 + b2 + c2 b2 + c2 + 2 + c2 + a2 c2 + a2 + 2 ≥ 3 2 ⇔ (a + b)2 + (a − b)2 a2 + b2 + 2 + (b + c)2 + (b − c)2 b2 + c2 + 2 + (c + a)2 + (c − a)2 c2 + a2 + 2 ≥ 3 ⇔ (a + b)2 a2 + b2 + 2 + (b + c)2 b2 + c2 + 2 + (c + a)2 c2 + a2 + 2 + (a − b)2 a2 + b2 + 2 + (b − c)2 b2 + c2 + 2 + (c − a)2 c2 + a2 + 2 ≥ 3. 16
  • 18. Chương 2. Bất đẳng thức có tổng các biến không đổi Theo bất đẳng thức Cauchy-Schwarz, ta có (a + b)2 a2 + b2 + 2 + (b + c)2 b2 + c2 + 2 + (c + a)2 c2 + a2 + 2 ≥ [2(a + b + c)]2 2a2 + 2b2 + 2c2 + 6 (1) và (b − a)2 a2 + b2 + 2 + (b − c)2 b2 + c2 + 2 + (c − a)2 c2 + a2 + 2 ≥ (2b − 2a)2 2a2 + 2b2 + 2c2 + 6 . (2) Bất đẳng thức được chứng minh nếu 4 (a + b + c)2 + (2b − 2a)2 2a2 + 2b2 + 2c2 ≥ 3 ⇔2(a + b + c)2 + 2(b − a) ≥ 3(a2 + b2 + c2 + 3) ⇔ a2 + b2 + c2 2 + 2 (b − a)2 ≥ a2 + b2 + c2 ⇔bc − ab + ca − c2 ≥ 0 ⇔(c − a)(b − c) ≥ 0. (3) Có thể nhận thấy bất đẳng thức (3) không phải luôn đúng nhưng ta có thể ép nó đúng. Thật vậy sử dụng các đánh giá tương tự (3), ta có (a − b)2 a2 + b2 + 2 + (c − b)2 b2 + c2 + 2 + (c − a)2 c2 + a2 + 2 ≥ (2c − 2b)2 2a2 + 2b2 + 2c2 + 6 (a − b)2 a2 + b2 + 2 + (b − c)2 b2 + c2 + 2 + (a − c)2 c2 + a2 + 2 ≥ (2a − 2c)2 2a2 + 2b2 + 2c2 + 6 và cùng lần lượt đưa bài toán về chứng minh (a − b)(c − a) ≥ 0 (4) (b − c)(a − b) ≥ 0. (5) Như vậy nếu trong các bất đẳng thức (3),(4) và (5) có một bất đẳng thức đúng thì bài toán sẽ được chứng minh song. Ta thấy rằng [(c − a)(b − c)] [(a − b)(c − a)] [(b − c)(a − b)] = (a − b)2 (b − c)2 (c − a)2 ≥ 0 nên ít nhất một trong ba số (c-a)(b-c), (a-b)(c-a),(b-c)(a-b) sẽ có một số không âm. Tức là phải có ít nhất một bất đẳng thức đúng hay bất đẳng thức được chứng minh. Đẳng thức xảy ra khi và chỉ khi a = b = c = 1. 17
  • 19. Chương 2. Bất đẳng thức có tổng các biến không đổi Nhận xét 2.1. Có nhiều công cụ hỗ trợ ta thực hiện phương pháp dồn biến, dưới đây ta xem xét ứng dụng yếu tố "ít nhất" và bất đẳng thức Cauchy-Schwarz trong việc giảm biến số của bất đẳng thức. Cụ thể có thể đưa bất đẳng thức ba biến về bất đẳng thức một biến để chứng minh. Ý tưởng của kĩ thuật như sau: Với bốn số thực a, b, c, k ta có [(a − k)(b − k)] [(b − k)(c − k)] [(c − k)(a − k)] = (a − k)2 (b − k)2 (c − k)2 ≥ 0. Do đó trong ba số (a − k)(b − k), (b − k)(c − k), (c − k)(a − k) sẽ có "ít nhất" một số không âm. Giả sử (a − k)(b − k) ≥ 0 thế thì a2 + b2 = k2 + (a + b − k)2 − 2(a − k)(b − k) ≤ k2 + (a + b − k)2 . Như vậy để chứng minh bất đẳng thức có giả thiết dạng a + b + c = s và đẳng thức xảy ra khi 2 biến đằng một giá trị nào đó ta có thể sử dụng đánh giá trên để làm giảm biến số của bất đẳng thức ban đầu. Cụ thể chọn k = m ( đảm bảo dấu bằng) và ta có a2 + b2 ≤ m2 + (a + b − m)2 = m2 + (s − c − m)2 . Nghĩa là có thể sử dụng đánh giá này vào bài toán thì ta sẽ chỉ còn phải chứng minh bất đẳng thức của một biến c. Bài toán 2.10. Với a, b, c là các số thực thỏa mãn điều kiện a+b+c = 3. Chứng minh rằng a 7a2 + 11 + b 7b2 + 11 + c 7c2 + 11 ≤ 1 6 . Chứng minh. Bất đẳng thức cần chứng minh ⇔1 − 14a 7a2 + 11 + 1 − 14b 7b2 + 11 + 1 − 14c 7c2 + 11 ≥ 3 − 14 6 ⇔ 7(a − 1)2 + 4 7a2 + 11 + 7(b − 1)2 + 4 7b2 + 11 + 7(c − 1)2 + 4 7c2 + 11 ≥ 2 3 ⇔4 1 7a2 + 11 + 1 7b2 + 11 + 1 7c2 + 11 + 7 (a − 1)2 7a2 + 11 + (b − 1)2 7b2 + 11 + (c − 1)2 7c2 + 11 ≥ 2 3 . Áp dụng bất đẳng thức Cauchy -Schwarz, ta có 1 7a2 + 11 + 1 7b2 + 11 + 1 7c2 + 11 ≥ (1 + 1 + 1)2 7(a2 + b2 + c2) + 33 = 9 7(a2 + b2 + c2) + 33 = 3(a + b + c)2 21(a2 + b2 + c2) + 11(a + b + c)2 18
  • 20. Chương 2. Bất đẳng thức có tổng các biến không đổi và cũng theo bất đẳng thức Cauchy-Schwarz, ta có (1 − a)2 7a2 + 11 + (b − 1)2 7b2 + 11 + (c − 1)2 7c2 + 11 ≥ (1 − a + b − 1 + c − 1)2 7(a+b2 + c2) + 33 = (b + c − a − 1)2 7(a2 + b2 + c2) + 11.3 = 3 b + c − a − a + b + c 3 2 21(a2 + b2 + c2) + 11.(a + b + c)2 = 4(b + c − 2a)2 3 [21(a2 + b2 + c2) + 11.(a + b + c)2] . Bất đẳng thức được chứng minh nếu 4.3(a + b + c)2 21(a2 + b2 + c2) + 11(a + b + c)2 + 7.4(b + c − 2a)2 3 [21(a2 + b2 + c2) + 11.(a + b + c)2] ≥ 2 3 ⇔2.9(a + b + c)2 + 7.2(b + c − 2a)2 ≥ 21(a2 + b2 + c2 ) + 11(a + b + c)2 ⇔42 [a(a − c) − b(a − c)] ≥ 0 ⇔42(a − c)(a − b) ≥ 0. (1) Rõ ràng (1 ) không phải lúc nào cũng đúng, nhưng từ đó vẫn đi đến điều phải chứng minh bằng cách đánh giá tương tự (a − 1)2 7a2 + 11 + (1 − b)2 7b2 + 11 + (c − 1)2 7c2 + 11 ≥ 4(a + c − 2b)2 3 [21(a2 + b2 + c2) + 11.(a + b + c)2] (a − 1)2 7a2 + 11 + (b − 1)2 7b2 + 11 + (1 − c)2 7c2 + 11 ≥ 4(a + b − 2c)2 3 [21(a2 + b2 + c2) + 11.(a + b + c)2] và ta lần lượt đưa bài toán về xét tính đúng sai của 42(b − a)(b − c) ≥ 0 (2) 42(c − a)(c − b) ≥ 0 (3) mà ta lại có (a − b)(a − c) + (b − a)(b − c) + (c − a)(c − b) = a2 + b2 + c2 − ab − bc − ca ≥ 0. Do đó trong (1),(2),(3) có ít nhất một bất đẳng thức đúng. Từ đó suy ra điều phải chứng minh. Đẳng thức xảy ra khi và chỉ khi a = b = c = 1. 19
  • 21. Chương 2. Bất đẳng thức có tổng các biến không đổi Bài toán 2.11. Chứng minh rằng với các số thực dương a, b, c tùy ý ta đều có (b + c − a)2 a2 + (b + c)2 + (c + a − b)2 b2 + (c + a)2 + (a + b − c)2 c2 + (a + b)2 ≥ 3 5 . Chứng minh. Do bất đẳng thức đã cho thuần nhất với biến a, b, c nên ta hoàn toàn có thể chuẩn hóa cho a + b + c = 1. Khi đó bất đẳng thức đã cho được viết dưới dạng (1 − 2a)2 2a2 − 2a + 1 + (1 − 2b)2 2b2 − 2b + 1 + (1 − 2c)2 2c2 − 2c + 1 ≥ 3 5 . Ta có a − 1 3 b − 1 3 b − 1 3 c − 1 3 c − 1 3 a − 1 3 = a − 1 3 2 b − 1 3 2 c − 1 3 2 ≥ 0. Không mất tính tổng quát, giả sử a − 1 3 b − 1 3 ≥ 0. Ta có a2 + b2 ≤ 1 9 + (a + b − 1 3 )2 = 1 9 + 2 3 − c 2 . Áp dụng bất đẳng thức Cauchy-Schwarz, ta có (1 − 2a)2 2a2 − 2a + 1 + (1 − 2b)2 2b2 − 2b + 1 ≥ [2 − 2(a + b)]2 2(a2 + b2) − 2(a + b) + 2 ≥ [2 − 2(a + b)]2 2 1 9 + ( 2 3 − c)2 − 2 (1 − c) + 2 = 2c2 1 9 + 2 3 − c 2 + c = 18c2 9c2 − 3c + 5 . Bất đẳng thức được chứng minh nếu ta chứng minh được (1 − 2c)2 2c2 − 2c + 1 + 18c2 9c2 − 3c + 5 ≥ 35 ⇔(3c − 1)2 (17c2 − 8c + 5) ≥ 0 luôn đúng . Đẳng thức xảy ra khi và chỉ khi a = b = c. 20
  • 22. Chương 2. Bất đẳng thức có tổng các biến không đổi 2.1.3 Sử dụng các tính chất của hàm số Trong các bài toán chứng minh bất đẳng thức dạng P(x, y, z) ≥ 0(≤ 0), dựa vào tổng không đổi và dựa vào dự đoán dấu đẳng thức xảy ra khi nào. Ta có thể đưa bài toán về dạng hàm một biến và sử dụng các tính chất của hàm số để khảo sát. Hoặc sử dụng bất đẳng thức Karamata và bổ đề cơ bản. Bài toán 2.12. Với x, y, z > 0 thỏa mãn x + y + z = 1. Chứng minh rằng 1 xz + 1 yz ≥ 16. Chứng minh. Yêu cầu bài toán ⇔ 1 z 1 x + 1 y ≥ 16. Theo bất đẳng thức AM-GM, ta có 1 x + 1 y ≥ 4 x + y = 4 1 − z . Ta đi chứng minh 1 z . 4 1 − z ≥ 16, z ∈ (0; 1). Xét hàm số f(z) = 1 z . 4 1 − z − 16 = 4 −z2 + z − 16, với z ∈ (0; 1) f (z) = −4(−2z + 1) (−z2 + z)2 . f (z) = 0 ⇔ z = 1 2 . Ta lập bảng biến thiên z 0 1 2 1 f (z) − 0 + f(z) +∞ 0 +∞ Từ bảng biến thiên, ta có f(z) ≥ 0, với ∀z ∈ (0; 1). Đẳng thức xảy ra khi và chỉ khi x = y = 1 4 , z = 1 2 . Bài toán 2.13 (Tiệp Khắc MO ,1984). Với x, y, z ≥ 0 thỏa mãn x + y + z = 1. Chứng minh rằng xy + yz + zx − 2xyz ≤ 7 27 . 21
  • 23. Chương 2. Bất đẳng thức có tổng các biến không đổi Chứng minh. Đặt xy + yz + zx − 2xyz = P. Giả sử x = min{x, y, z}. Từ giả thiết x, y, z ≥ 0, x + y + z = 1 ⇒ 0 ≤ x ≤ 1 3 . Theo bất đẳng thức AM-GM, ta có 1 − x = y + z ≥ 2 √ yz ⇒ (1 − x)2 4 ≥ yz. Do đó P = xy + yz + zx − 2xyz = (xy + zx) + (yz − 2xyz) = x(y + z) + yz(1 − 2x) ≤ x(1 − x) + (1 − x)2 4 (1 − 2x) = −2x3 + x2 + 1 4 = f(x). Xét hàm số f(x) = −2x3 + x2 + 1 4 với x ∈ 0; 1 3 , f (x) = −6x2 + 2x 4 ⇒f (x) = 0 ⇔ x = 0 x = 1 3 . Ta có bảng biến thiên x 0 1 3 f (x) 0 + 0 f(a) 1 4 7 27 Từ bảng biên thiên ta thấy với ∀x ∈ 0; 1 3 , ta có P ≤ f(x) ≤ 7 27 . Đẳng thức xảy ra khi và chỉ khi x = y = z = 1 3 . 22
  • 24. Chương 2. Bất đẳng thức có tổng các biến không đổi Bài toán 2.14. Với x, y, z ≥ 0 thỏa mãn x + y + z = 1. Chứng minh rằng xy + yz + zx ≤ 1 4 + 9 4 xyz. Chứng minh. Giả sử z = min{x, y, z} ⇒ 0 ≤ z ≤ 1 3 . Có x + y + z = 1 ⇒ 1 − z = x + y. Theo bất đẳng thức AM-GM, ta có (x + y)2 4 ≥ xy. Bất đẳng thức cần chứng minh tương đương với xy + yz + zx − 9 4 xyz ≤ 1 4 . Ta có xy + yz + zx − 9 4 xyz = xy + z(x + y) − 9 4 xyz = xy(1 − 9 4 z) + z(x + y) ≤ (x + y)2 4 (1 − 9 4 z) + z(1 − z) = (1 − z)2 4 (1 − 9z 4 ) + z − z2 = 1 16 (−9z3 + 6z2 − z + 4). Xét hàm f(z) = −9z3 + 6z2 − z + 4, với z ∈ 1; 1 3 . Ta có f (z) = −27z2 + 12z − 1 ⇒ f (z) = 0 ⇔   z = 1 3 z = 1 9 . Ta lập bảng biến thiên z 0 1 9 1 3 f (z) | − 0 + 0 f(z) 4 320 81 4 Từ bảng biến thiên ta thấy f(z) ≤ 4, ∀z ∈ 0; 1 3 . Vậy xy + yz + zx ≤ 1 4 + 9 4 xyz. Đẳng thức xảy ra khi và chỉ khi   z = 0, x = y = 1 2 x = y = z = 1 3 . 23
  • 25. Chương 2. Bất đẳng thức có tổng các biến không đổi Bài toán 2.15. Với x, y, z > 0 và x + y + z = 1. Chứng minh rằng 2(x3 + y3 + z3 ) + 3(x2 + y2 + z2 ) + 12xyz ≥ 5 3 . Chứng minh. Giả sử x = min{x, y, z} ⇒ x ∈ 0; 1 3 . Từ giả thiết ta có y + z = 1 − x và theo bất đẳng thức AM-GM, ta có yz ≤ (y + z)2 4 . Đặt 2(x3 + y3 + z3) + 3(x2 + y2 + z2) + 12xyz = P. Khi đó, ta có P = 2 x3 + (y + z)3 − 3yz(y + z) + 3 x2 + (y + z)2 − 2yz + 12xyz = 2x3 + 2(1 − x)3 − 6yz(1 − x) + 3x2 + 3(1 − x)2 − 6yz + 12xyz = 12x2 − 12x + 5 − 6yz(2 − 3x) ≥ 12x2 − 12x + 5 − 6 (y + z)2 4 (2 − 3x) = 9 2 x3 − 3 2 x + 2. Đặt 9 2 x3 − 3 2 x + 2 = f(x) với x ∈ 0; 1 3 . Ta có f (x) = 27 2 x2 − 3 2 ⇒ f (x) = 0 ⇔ x = 1 3 . Ta có bảng biến thiên x 0 1 3 f (x) | − 0 f(x) 2 5 3 Từ bảng biến thiên ta suy ra P ≥ f(x) ≥ 5 3 với x ∈ 0; 1 3 . Đẳng thức xảy ra khi và chỉ khi x = y = z = 1 3 . Bài toán 2.16. Với x, y, z ∈ [0; 2] thỏa mãn điều kiện x + y + z = 3. Chứng minh rằng x3 + y3 + z3 ≤ 9. 24
  • 26. Chương 2. Bất đẳng thức có tổng các biến không đổi Chứng minh. Giả sử 2 ≥ x ≥ y ≥ z ≥ 0 . Khi đó    2 ≥ x 2 + 1 ≥ x + y 2 + 1 + 0 = x + y + z Xét hàm số f(t) = t3, với t ∈ [0; 2] f (t) = 3t2 f (t) = 6t ≥ 0, với t ∈ [0; 2]. Do đó với x, y, z ∈ [0; 2], theo bất đẳng thức Karamata ta có f(2) + f(1) + f(0) ≥ f(x) + f(y) + f(z) ⇔x3 + y3 + z3 ≤ 9. Đẳng thức xảy ra khi và chỉ khi x = 2; y = 1; z = 0 và các hoán vị của chúng. Bài toán 2.17. Cho x, y, z ∈ [1; 3] , x + y + z = 6. Chứng minh rằng x2 + y2 + z2 ≤ 14. Chứng minh. Giả sử 3 ≥ x ≥ y ≥ z ≥ −1 . Khi đó    3 ≥ x 3 + 2 ≥ x + y 3 + 2 + 1 = x + y + z Xét hàm số f(t) = t2, với t ∈ [1; 3] f (t) = 2t f (t) = 2 ≥ 0, với t ∈ [1; 3]. Do đó với x, y, z ∈ [1; 3], theo bất đẳng thức Karamata ta có f(3) + f(2) + f(1) ≥ f(x) + f(y) + f(z) ⇔x2 + y2 + z2 ≤ 14. Đẳng thức xảy ra khi và chỉ khi x = 3; y = 2; z = 1 và các hoán vị của chúng. 25
  • 27. Chương 2. Bất đẳng thức có tổng các biến không đổi Bài toán 2.18. Với x, y, z là các số thực không âm và thỏa mãn x + y + z = 1. Chứng minh rằng x x + 1 + y y + 1 + z z + 1 ≤ 3 4 . Chứng minh. Xét hàm số f(t) = t t + 1 , t ∈ [0; 1] . Ta có f (t) = 1 (t + 1)2 ⇒ f (t) = −2 (t + 1)3 < 0, ∀x ∈ [0; 1] . Áp dụng bổ đề 1.1,ta có f(x) = x 1 + x ≤ f 1 3 x − 1 3 + f 1 3 f(y) = y 1 + y ≤ f 1 3 y − 1 3 + f 1 3 f(z) = z 1 + z ≤ f 1 3 z − 1 3 + f 1 3 . Cộng các bất đẳng thức cùng chiều, ta có x x + 1 + y y + 1 + z z + 1 ≤ 3f 1 3 (x + y + z − 1) + 3f 1 3 = 3 4 . Đẳng thức xảy ra khi và chỉ khi x = y = z = 1 3 . Bài toán 2.19. Với a, b, c là các số thực dương thỏa mãn a + b + c = 1. Chứng minh rằng a a + bc + b 1 + ac + c 1 + ab ≥ 9 10 . Chứng minh. Ta có a 1 + bc ≥ a 1 + (b + c)2 4 = 4a 4 + (1 − a)2 = 4a a2 − 2a + 5 . Tương tự, ta có b 1 + ac ≥ 4b b2 − 2b + 5 c 1 + ab ≥ 4c c2 − 2c + 5 . Cộng các bất đẳng thức cùng chiều, ta có a a + bc + b 1 + ac + c 1 + ab ≥ 4a a2 − 2a + 5 + 4b b2 − 2b + 5 + 4c c2 − 2c + 5 . 26
  • 28. Chương 2. Bất đẳng thức có tổng các biến không đổi Xét hàm số f(x) = 4x x2 − 2x + 5 , với x ∈ (0; 1) . Ta có f (x) = −4x2 + 20 (x2 − 2x + 5)2 Ta đi chứng minh với ∀ ∈ (0; 1), ta luôn có f(x) ≥ f ( 1 3 ) x − 1 3 + f 1 3 ⇔ 4x x2 − 2x + 5 ≥ 99 100 x − 1 3 + 3 10 ⇔ −99x3 + 201x2 − 101x + 15 100(x2 − 2x + 5) ≥ 0 ⇔ x − 1 3 2 (15 − 11x) x2 − 2x + 5 ≥ 0 luôn đúng ∀x ∈ (0; 1) . Áp dụng bất đẳng thức trên với a, b, c ∈ (0; 1), ta có f(a) ≥ f 1 3 a − 1 3 + f 1 3 f(b) ≥ f 1 3 b − 1 3 + f 1 3 f(c) ≥ f 1 3 c − 1 3 + f 1 3 . Cộng các bất đẳng thức cùng chiều, ta có a a + bc + b 1 + ac + c 1 + ab ≥ 4a a2 − 2a + 5 + 4b b2 − 2b + 5 + 4c c2 − 2c + 5 ≥ f 1 3 (a + b + c − 1) + 3f 1 3 = 9 10 . Đẳng thức xảy ra khi và chỉ khi a = b = c = 1 3 . Bài toán 2.20 (Olympic 30-4-06). Với a, b, c là các số thực dương . Chứng minh a(b + c) a2 + (b + c)2 + b(c + a) b2 + (c + a)2 + c(c + b) c2 + (a + b)2) ≤ 6 5 . Chứng minh. Bất đẳng thức cần chứng minh là một bất đẳng thức thuần nhất với ba biến a, b, c Không mất tính tổng quát giả sử a + b + c = 1. Khi đó bất đẳng thức cần chứng minh trở thành a(1 − a) a2 + (1 − a)2 + b(1 − b) b2 + (1 − b)2 + c(1 − c) c2 + (1 − c)2 ≤ 6 5 ⇔f(a) + f(b) + f(c) ≤ 6 5 27
  • 29. Chương 2. Bất đẳng thức có tổng các biến không đổi với f(x) = x(1 − x) x2 + (1 − x)2 = −x2 + x 2x2 − 2x + 1 , x ∈ (0; 1), f (x) = −2x + 1 (2x2 − 2x + 1)2 ⇒ f ( 1 3 ) = 27 25 . Ta đi chứng minh với ∀x ∈ (0; 1), ta có f(x) ≤ f ( 1 3 )(x − 1 3 ) + f( 1 3 ) ⇔ −x2 + x 2x2 − 2x + 1 ≤ 27 25 (x − 1 3 ) + 2 5 ⇔ −54x3 + 27x2 − 1 25(2x2 − 2x + 1) ≤ 0 ⇔ −54(x + 1 6 )(x − 1 3 )2 25(2x2 − 2x + 1) ≤ 0 luôn đúng ∀x ∈ (0; 1) . Do đó với a, b, c ∈ (0; 1), ta có f(a) + f(b) + f(c) ≤ f ( 1 3 )(a + b + c − 3 1 3 ) + 3f( 1 3 ) = 6 5 . Đẳng thức xảy ra khi và chỉ khi a = b = c = 1 3 . Bài toán 2.21 (USA MO -2003). Với các số thực dương a, b, c. Chứng minh rằng (2a + b + c)2 2a2 + (b + c)2 + (2b + a + c)2 2b2 + (c + a)2 + (2c + a + b)2 2c2 + (a + b)2 ≤ 8. Chứng minh. Giả sử a + b + c = 1. Khi đó bất đẳng thức cần chứng minh trở thành (a + 1)2 2a2 + (1 − a)2 + (b + 1)2 2b2 + (1 − b)2 + (c + 1)2 2c2 + (1 − c)2 ≤ 8 ⇔ a2 + 2a + 1 3a2 − 2a + 1 + b2 + 2b + 1 3b2 − 2b + 1 + c2 + 2c + 1 3c2 − 2c + 1 ≤ 8 hay f(a) + f(b) + f(c) ≤ 8 với f(x) = x2 + 2x + 1 3x2 − 2x + 1 , x ∈ (0; 1) f (x) = −8x2 − 4x + 4 (3x2 − 2x + 1)2 . 28
  • 30. Chương 2. Bất đẳng thức có tổng các biến không đổi Ta chứng minh với ∀x ∈ (0; 1), ta có f(x) ≤ f ( 1 3 )(x − 1 3 ) + f( 1 3 ) ⇔ x2 + 2x + 1 3x2 − 2x + 1 ≤ 4(x − 1 3 ) + 8 3 ⇔ x2 + 2x + 1 3x2 − 2x + 1 − 12x + 4 3 ≤ 0 ⇔ −36x3 + 15x2 + 2x − 1 3(3x2 − 2x + 1) ≤ 0 ⇔ −36(x + 1 4 )(x − 1 3 )2 3(3x2 − 2x + 1) ≤ 0, luôn đúng ∀x ∈ (0; 1) . Từ đó, ta có f(a) ≤ f ( 1 3 )(a − 1 3 ) + f( 1 3 ) f(b) ≤ f ( 1 3 )(b − 1 3 ) + f( 1 3 ) f(c) ≤ f ( 1 3 )(c − 1 3 ) + f( 1 3 ). Cộng các bất đẳng thức cùng chiều ta có f(a) + f(b) + f(c) ≤ f ( 1 3 )(a + b + c − 1) + 3f( 1 3 ) = 8. Đẳng thức xảy ra khi và chỉ khi a = b = c = 1 3 . Bài toán 2.22 (Olympic Balkan 1996 và Olympic 30-4 -1999). Với các số thực a, b, c thỏa mãn a + b + c = 1. Chứng minh rằng a 1 + a2 + b 1 + b2 + c 1 + c2 ≤ 9 10 . Chứng minh. Đặt f(x) = x 1 + x2 . Khi đó bất đẳng thức đã cho trở thành f(a) + f(b) + f(c) ≤ 9 10 . Ta có f (x) = 1 − x2 (1 + x2)2 ⇒ f (x) = 0 ⇔ x = −1 x = 1 29
  • 31. Chương 2. Bất đẳng thức có tổng các biến không đổi Ta có bảng biến thiên ( đưa thêm một số giá trị như x = −3; x = −1 3 ; x = 2 và giá trị của hàm số tại đó để so sánh) x −∞ −3 −1 −1 3 1 2 +∞ f (x) − 0 + 0 + f(x) 0 −3 10 −1 2 −3 10 1 2 2 5 0 Ta xét các trường hợp sau: + Trường hợp 1. Trong ba số a, b, c có một số thuộc (−∞; −3]. Giả sử a ∈ (−∞; −3] ⇒ b + c ≥ 4, giả sử b ≥ 2. Khi đó từ bảng biến thiên, ta có f(a) + f(b) + f(c) < 0 + 1 2 + 2 5 = 9 10 + Trường hợp 2. Có một số, giả sử a ∈ −3; −1 3 . Khi đó có f(a) + f(b) + f(c) ≤ −3 10 + 1 2 + 1 2 = 7 10 < 9 10 . + Trường hợp 3. Cả ba số a, b, c ∈ −1 3 ; +∞ . Khi đó tiếp tuyến của hàm số y = f(x) tại x = 1 3 có phương trình y = f 1 3 x − 1 3 + f 1 3 hay y = 18 25 x + 3 50 . Ta chứng minh f(x) ≤ 18 25 x + 3 50 ⇔ x 1 + x2 − 18 25 x − 3 50 ≤ 0 ⇔ −36x3 − 3x2 + 14x − 3 50(1 + x2) ≤ 0 ⇔ −36(x + 3 4 (x − 1 3 )2) 50(1 + x2) ≤ 0 luôn đúng với ∀x ≥ −1 3 . Do đó f(a) + f(b) + f(c) ≤ 18 25 (a + b + c) + 3 3 50 = 9 10 . Đẳng thức xảy ra khi và chỉ khi a = b = c = 1 3 . 30
  • 32. Chương 2. Bất đẳng thức có tổng các biến không đổi 2.1.4 Bài toán liên quan Bài toán 2.23. Với các số dương x, y, z thỏa mãn điều kiện x+y +z = 1. Chứng minh 3 xy + yz + zx + 2 x2 + y2 + z2 ≥ 14. Bài toán 2.24. Với a, b, c > 0 thỏa mãn a + b + c = 3. Chứng minh rằng a + 1 b2 + 1 + b + 1 c2 + 1 + c + 1 a2 + 1 ≥ 3. Bài toán 2.25. Với a, b, c > 0 và thỏa mãn a + b + c = 3. Chứng minh rằng a b3 + ab + b c3 + ca + c a3 + ca ≥ 3 2 . Bài toán 2.26. Với a, b, c là các số thực dương thỏa mãn a + b + c = 3. Chứng minh bất đẳng thức sau ab + bc + ca + 1 abc ≥ abc + 3. Bài toán 2.27. Với a, b, c là các số thực dương thỏa mãn a + b + c = 3. Chứng minh rằng a3 b(2c + a) + b3 c(2a + b) + c3 a(2b + c) ≥ 1. Bài toán 2.28. Với a, b, c ≥ 0 thỏa mãn a + b + c = 1. Chứng minh a2 + b2 + c2 + 3abc ≥ 4 9 . Bài toán 2.29. Với a, b, c là độ dài ba cạnh của tam giác. Chứng minh rằng 1 a + 1 b + 1 c + 9 a + b + c ≥ 4 1 a + b + 1 b + c + 1 c + a . Bài toán 2.30. Với a, b, c là các số thực dương . Chứng minh rằng a (b + c)2 + b (c + a)2 + c (a + b)2 ≥ 9 4(a + b + c) . Bài toán 2.31 (Japan Mathematical Olympiad Finals 1997). Với a, b, c là các số thực dương . Chứng minh rằng (b + c − a)2 (b + c)2 + a2 + (c + a − b)2 (c + a)2 + b2 + (a + b − c)2 (a + b)2 + c2 ≥ 3 5 . Bài toán 2.32 (China MO 2005). Với a, b, c không âm thỏa mãn a + b + c = 1. Chứng minh rằng 10(a3 + b3 + c3 ) − 9(a5 + b5 + c5 ) ≥ 1. 31
  • 33. Chương 2. Bất đẳng thức có tổng các biến không đổi Bài toán 2.33 (China Northern MO 2006). Với các số thực a, b, c thỏa mãn a + b + c = 3. Chứng minh rằng a2 + 9 2a2 + (b + c)2 + b2 + 9 2b2 + (a + c)2 + c2 + 9 2c2 + (a + b)2 ≤ 5. Bài toán 2.34. Với a, b, c là các số thực thỏa mãn điều kiện a+b+c = 3. Chứng minh rằng 18a − 35 a2 − 4a + 6 + 18b − 35 b2 − 4b + 6 + 18c − 35 c2 − 4c + 6 ≥ −17. Bài toán 2.35. Với a, b, c là các số thực thỏa mãn a + b + c = 1. Chứng minh rằng a2 6a2 − 4a + 1 + b2 6b2 − 4b + 1 + c2 6c2 − 4c + 1 ≤ 1. Bài toán 2.36. Với a, b, c là các số thực thỏa mãn điều kiện a+b+c = 1. Chứng minh rằng a2 3a2 − 2a + 3 + b2 3b2 − 2b + 3 + c2 3c2 − 2c + 3 ≥ 1 8 . 32
  • 34. Chương 2. Bất đẳng thức có tổng các biến không đổi 2.2 Bất đẳng thức có tổng các biến không đổi với hàm vô tỉ 2.2.1 Sử dụng bất đẳng thức AM-GM Trong phần này ta đi chứng minh các bất đẳng thức với hàm vô tỉ với tổng không đổi bằng bất đẳng thức AM-GM. Điều quan trọng là ta vận dụng bất đẳng thức AM-GM khéo léo để phá được dấu căn và tận dụng tổng không đổi. Bài toán 2.37. Với x, y, z là các số thực không âm thỏa mãn x + y + z = 2. Chứng minh x3y + y3z + z3x + xy3 + yz3 + zx3 ≤ 2. Chứng minh. Trước hết ta chứng minh bất đẳng thức √ a + √ b ≤ 2(a + b) (∗) với a, b ≥ 0. Thật vậy, bình phương hai vế của bất đẳng thức (*), ta có a + 2 √ ab + b ≤ 2(a + b) ⇔ a + b ≥ 2 √ ab luôn đúng theo AM-GM . Áp dụng bất đẳng thức (* ) ta có x3y + y3z + z3x+ xy3 + yz3 + zx3 ≤ 2[xy(x2 + y2) + yz(y2 + z2) + zx(z2 + x2)]. Yêu cầu của bài toán được thỏa mãn nếu chứng minh được xy(x2 + y2 ) + yz(y2 + z2 ) + zx(z2 + x2 ) ≤ 2. Ta có xy(x2 + y2 ) + yz(y2 + z2 ) + zx(z2 + x2 ) ≤xy(x2 + y2 + z2 ) + yz(x2 + y2 + z2 ) + zx(x2 + y2 + z2 ) =(xy + yz + zx)(x2 + y2 + z2 ). Đến đây ta cần chứng minh (xy + yz + zx)(x2 + y2 + z2 ) ≤ 2. Áp dụng bất đẳng thức AM-GM, ta có (xy + yz + zx)(x2 + y2 + z2 ) = 1 2 2(xy + yz + zx)(x2 + y2 + z2 ) ≤ 1 2 (2xy + 2yz + 2zx + x2 + y2 + z2)2 4 = 1 8 (x + y + z)4 = 2. Đẳng thức xảy ra khi và chỉ khi x = y = 1, z = 0 hoặc các hoán vị của chúng. 33
  • 35. Chương 2. Bất đẳng thức có tổng các biến không đổi Bài toán 2.38. Với a, b, c là các số thực dương thỏa mãn a + b + c = 3. Chứng minh rằng a(b + 3c) + b(c + 3a) + c(a + 3b) ≤ 6. Chứng minh. Áp dụng bất đẳng thức AM-GM, ta có a(b + 3c) = 1 2 4a(b + 3c) ≤ 1 2 4a + b + 3c 2 . Tương tự b(c + 3a) ≤ 1 2 4b + c + 3a 2 c(a + 3b) ≤ 1 2 4c + a + 3b 2 . Cộng các bất đẳng thức cùng chiều, ta được a(b + 3c) + b(c + 3a) + c(a + 3b) ≤ 1 2 8(a + b + c) 2 = 6. Đẳng thức xảy ra khi và chỉ khi a = b = c = 1. Bài toán 2.39. [Russian MO 2002]Với a, b, c > 0 thỏa mãn a + b + c = 3. Chứng minh rằng √ a + √ b + √ c ≥ ab + bc + ca. Chứng minh. Bất đẳng thức cần chứng minh tương đương với a2 + b2 + c2 + 2( √ a + √ b + √ c) ≥ 2(ab + bc + ca) + a2 + b2 + c2 ⇔a2 + b2 + c2 + 2( √ a + √ b + √ c) ≥ (a + b + c)2 = 9. Áp dụng bất đẳng thức AM-GM, ta có a2 + √ a + √ a ≥ 3a b2 + √ b + √ b ≥ 3b c2 + √ c + √ c ≥ 3c. Cộng các bất đẳng thức cùng chiều, ta có a2 + b2 + c2 + 2 √ a + 2 √ b + 2 √ c ≥ 3(a + b + c) = 9. Đẳng thức xảy ra khi và chỉ khi a = b = c = 1. 34
  • 36. Chương 2. Bất đẳng thức có tổng các biến không đổi Bài toán 2.40 (Mexico MO 2007). Nếu a, b, c là các số thực dương thỏa mãn a + b + c = 1. Chứng minh rằng √ a + bc + √ b + ca + √ c + ab ≤ 2. Chứng minh. Sử dụng điều kiện a + b + c = 1, ta có a + bc = a(a + b + c) + bc = (a + b)(a + c). Theo bất đẳng thức AM-GM, ta có √ a + bc = (a + b)(a + c) ≤ 2a + b + c 2 . Tương tự √ b + ac ≤ 2b + a + c 2 √ c + ab ≤ 2c + a + b 2 . Cộng các bất đẳng thức cùng chiều, ta có √ a + bc + √ b + ca + √ c + ab ≤ 4a + 4b + 4c 2 = 2(a + b + c) = 2. Đẳng thức xảy ra khi và chỉ khi a = b = c = 1 3 . Bài toán 2.41. Với các số thực dương thỏa mãn a+b+c = 3. Chứng minh rằng ab √ c2 + 3 + bc √ a2 + 3 + ca √ b2 + 3 ≤ 3 2 . Chứng minh. Ta luôn có (a + b + c)2 3 ≥ ab + bc + ca ⇔ 3 ≥ ab + bc + ca. Suy ra ab √ c2 + 3 ≤ ab √ c2 + ab + bc + ca = ab (a + b) (a + c) . Từ đó áp dụng bất đẳng thức AM-GM, ta có ab (b + c) (c + a) ≤ 1 2 ab b + c + ab c + a . 35
  • 37. Chương 2. Bất đẳng thức có tổng các biến không đổi Chứng minh tương tự bc √ a2 + 3 ≤ bc (a + c) (a + b) ≤ 1 2 bc c + a + bc a + b ca √ b2 + 3 ≤ ca (b + a) (b + c) ≤ 1 2 ca b + a + ca b + c . Cộng các bất đẳng thức cùng chiều, ta có ab √ c2 + 3 + bc √ a2 + 3 + ca √ b2 + 3 ≤ 1 2 ab b + c + ca b + c + ab c + a + bc c + a + bc a + b + ca a + b = a + b + c 3 = 3 2 . Đẳng thức xảy ra khi và chỉ khi a = b = c = 1. 2.2.2 Sử dụng bất đẳng thức Cauchy-Schwarz Bài toán 2.42 (Bosnia and Hercegovina MO 2005). Với các số thực dương a, b, c thỏa mãn a + b + c = 1. Chứng minh rằng a √ b + b √ c + c √ a ≤ 1 √ 3 . Chứng minh. Sử dụng bất đẳng thức quen thuộc (a + b + c)2 ≥ 3(ab + bc + ca) và theo giả thiết a + b + c = 1. Khi đó ta có ab + bc + ca ≤ 1 3 . Áp dụng bất đẳng thức Cauchy- Schwarz, ta có a √ b + b √ c + c √ a 2 = √ a √ ab + √ b √ bc + √ c √ ca 2 ≤ (a + b + c) (ab + bc + ca) ≤ 1 3 . 36
  • 38. Chương 2. Bất đẳng thức có tổng các biến không đổi Vậy bất đẳng thức được chứng minh. Đẳng thức xảy ra khi và chỉ khi a = b = c = 1 3 . Bài toán 2.43. Với a, b, c là các số thực dương và a + b + c = 1. Chứng minh rằng √ a + bc + √ b + ca + √ c + ab ≥ √ ab + √ bc + √ ca + 1. Chứng minh. Sử dụng bất đẳng thức Cauchy -Schwarz, ta có √ a + bc + √ b + ca + √ c + ab = a(a + b + c) + bc + b(a + b + c) + ca + c(a + b + c) + ab = (a + c)(a + b) + (b + a)(b + c) + (c + b)(c + a) ≥a + √ bc + b + √ ac + c + √ ab =1 + √ ab + √ bc + √ ca. Vậy bất đẳng thức được chứng minh. Đẳng thức xảy ra khi và chỉ khi a = b = c = 1 3 . Nhận xét 2.2. Bài toán trên được xây dựng từ bất đẳng thức APMO 2002 như sau Với x, y, z > 0 thỏa mãn 1 x + 1 y + 1 z = 1. Chứng minh rằng √ x + yz + √ y + zx + √ z + xy ≥ √ xyz + √ x + √ y + √ z. Bài toán 2.44. Với các số thực dương a, b, c thỏa mãn a+b+c = 1. Chứng minh rằng ab(1 − c) + bc(1 − a) + ca(1 − b) ≤ 2 3 . Chứng minh. Theo hệ quả của bất đẳng thức AM-GM, ta có (a + b + c)2 ≥ 3(ab + bc + ca). 37
  • 39. Chương 2. Bất đẳng thức có tổng các biến không đổi Áp dụng bất đẳng thức Cauchy- Schwarz, ta có ab(1 − c) + bc(1 − a) + ca(1 − b) 2 ≤ (ab + bc + ca) (1 − c + 1 − a + 1 − b) = 2(ab + bc + ca) = 2 3 3(ab + bc + ca) ≤ 2 3 (a + b + c)2 = 2 3 . Do đó ab(1 − c) + bc(1 − a) + ca(1 − b) ≤ 2 3 . Đẳng thức xảy ra khi và chỉ khi a = b = c = 1 3 . Bài toán 2.45 (China MO 2008). Với các số thực dương a, b, c thỏa mãn a+b+c = 1. Chứng minh rằng a + (b − c)2 4 + √ b + √ c ≤ √ 3 Chứng minh. Đặt √ bc = x suy ra √ b + √ c = b + c + 2 √ bc = 1 − a + 2 √ bc = √ 1 − a + 2x. Và ta có a + (b − c)2 4 = a + (b + c)2 − 4bc 4 = a + (1 − a)2 − 4x2 4 = 1 2 (1 + a)2 − 4x2. Khi đó bất đẳng thức ban đầu trở thành (1 + a)2 − 4x2 + 2 √ 1 − a + 2x ≤ 2 √ 3. Áp dụng bất đẳng thức Cauchy-Schwarz, ta có (1 + a)2 − 4x2 + 2 (1 − a) + 2x ≤ 3 [(1 + a)2 − 4x2 + 2(1 − a + 2x)]. Vậy ta chỉ cần chứng minh (1 + a)2 − 4x2 + 2(1 − a + 2x) ≤ 4 ⇔a2 − (2x − 1)2 ≤ 0 ⇔(a − 2x + 1)(a + 2x − 1) ≤ 0 ⇔(1 − 2x − a)(1 + a − 2x) ≥ 0. 38
  • 40. Chương 2. Bất đẳng thức có tổng các biến không đổi Bất đẳng thức này hiển nhiên đúng vì 1 + a − 2x ≥ 1 − 2x − a = 1 − 2 √ bc − a ≥ 1 − (b + c) − a = 0. Bất đẳng thức đã cho được chứng minh. Đẳng thức xảy ra khi và chỉ khi a = b = c = 1 3 . Bài toán 2.46 (China MO 2006). Với các số thực dương a, b, c thỏa mãn a+b+c = 1. Chứng minh rằng ab √ ab + bc + bc √ bc + ca + ca √ ca + ab ≤ √ 2 2 . Chứng minh. Áp dụng bất đẳng thức Cauchy- Schwarz, ta có ab √ ab + bc + bc √ bc + ca + ca √ ca + ab 2 = √ ab √ ab + bc + ca a(ab + bc + ca) √ a + c + √ bc √ ab + bc + ca b(ab + bc + ca) √ b + a + √ ca √ ab + bc + ca c(ab + bc + ca) √ c + b 2 ≤ ab + bc + ca ab + bc + ca a(ab + bc + ca) a + c + b(ab + bc + ca) b + a + c(ab + bc + ca) c + b = (ab + bc + ca) a a + c + b b + a + c c + b . Bất đẳng thức được chứng minh nếu ta chứng minh được (ab + bc + ca) a a + c + b b + a + c c + b ≤ 1 2 ⇔2 (ab + bc + ca) a a + c + b b + a + c c + b ≤ (a + b + c)2 ⇔2 a2b a + c + ab2 b + a + abc c + b + abc a + c + b2c b + a + bc2 c + b + ca2 a + c + abc a + b + c2a c + b ≤ (a + b + c)2 ⇔2 ab + bc + ca + ab2 a + b + bc2 b + c + ca2 a + c ≤ (a + b + c)2 ⇔2 ab2 a + b + bc2 b + c + ca2 a + c ≤ a2 + b2 + c2 ⇔2b2 − 2ab2 a + b + 2c2 − 2bc2 b + c + 2a2 − 2ca2 c + a ≥ a2 + b2 + c2 ⇔ 2b3 a + b + 2c3 b + c + 2a3 c + a ≥ a2 + b2 + c2 . 39
  • 41. Chương 2. Bất đẳng thức có tổng các biến không đổi Mặt khác theo bất đẳng thức Cauchy -Schwarz và bất đẳng thức AM-GM, ta có 2 b3 a + b + c3 b + c + a3 c + a ≥ 2(b2 + c2 + a2)2 b(a + b) + c(b + c) + a(c + a) = 2(a2 + b2 + c2)2 a2 + b2 + c2 + ab + bc + ca ≥ (a2 + b2 + c2)2 a2 + b2 + c2 + a2 + b2 2 + b2 + c2 2 + c2 + a2 2 =a2 + b2 + c2 . Bất đẳng thức được chứng minh. Đẳng thức xảy ra khi và chỉ khi a = b = c = 1 3 . Bài toán 2.47. Với các số thực dương a, b, c thỏa mãn điều kiện a + b + c = 1. Chứng minh rằng a2 9a + 1 + b2 9b + 1 + c2 9c + 1 ≤ 1 12 3(ab + bc + ca) . Chứng minh. Ta có a2 9a + 1 = a 9 − a 9(9a + 1) b2 9b + 1 = b 9 − b 9(9b + 1) c2 9c + 1 = c 9 − c 9(9c + 1) . Khi đó, bất đẳng thức cần chứng minh trở thành a 9a + 1 + b 9b + 1 + c 9c + 1 + 3 4 3(ab + bc + ca) ≥ 1. Áp dụng bất đẳng thức Cauchy- Schwarz, ta có a 9a + 1 + b 9b + 1 + c 9c + 1 ≥ (a + b + c)2 a(9a + 1) + b(9b + 1) + c(9c + 1) = 1 9(a2 + b2 + c2) + 1 . 40
  • 42. Chương 2. Bất đẳng thức có tổng các biến không đổi Theo bất đẳng thức AM-GM, ta có 3 4 3(ab + bc + ca) ≥ 3 2 [1 + 3(ab + bc + ca)] . Do đó, bất đẳng thức được chứng minh nếu ta chứng minh được 1 9(a2 + b2 + c2) + 1 + 3 6(ab + bc + ca) + 2 ≥ 1. Áp dụng bất đẳng thức Cauchy- Schwarz, ta có 1 9(a2 + b2 + c2) + 1 + 3 6(ab + bc + ca) + 2 ≥ (1 + 3)2 9(a2 + b2 + c2) + 18(ab + bc + ca) + 7 = 16 9(a + b + c)2 + 7 = 1. Bất đẳng thức được chứng minh. Đẳng thức xảy ra khi và chỉ khi a = b = c = 1 3 . 2.2.3 Sử dụng các tính chất của hàm số Bài toán 2.48. Với các số thực dương a, b, c thỏa mãn a+b+c = 1. Chứng minh rằng a √ a2 + 1 + b √ b2 + 1 + c √ c2 + 1 ≤ 3 √ 10 . Chứng minh. Xét hàm số f(t) = t √ t2 + 1 , t ∈ (0; 1) f (t) = 1 (t2 + 1)3 , f (t) = − 3t (t2 + 1)5 < 0, ∀t ∈ (0; 1). Theo bổ đề 1.1, ta có f(a) ≤ f ( 1 3 )(a − 1 3 ) + f( 1 3 ) f(b) ≤ f ( 1 3 )(b − 1 3 ) + f( 1 3 ) f(c) ≤ f ( 1 3 )(c − 1 3 ) + f( 1 3 ). Cộng các bất đẳng thức cùng chiều, ta có f(a) + f(b) + f(c) ≤ f ( 1 3 )(a + b + c − 1) + 3f( 1 3 ) = 3 √ 10 . Đẳng thức xảy ra khi và chỉ khi a = b = c = 1 3 . 41
  • 43. Chương 2. Bất đẳng thức có tổng các biến không đổi Bài toán 2.49. Với a, b, c là các số thực dương thỏa mãn a + b + c = 3. Chứng minh rằng 1 1 + 8 √ a + 1 1 + 8 √ b + 1 1 + 8 √ b ≥ 1. Chứng minh. Xét hàm số f(t) = 1 1 + 8 √ t , t ∈ (0; 3). Ta có f (t) = −2 t(1 + 8 √ t)3 f (x) = 20 √ t + 1 (8 √ t + 1)5t3 > 0, ∀t ∈ (0; 3). Với a, b, c ∈ (0; 3), theo bổ đề 1.1, ta có f(a) ≥ f (1)(a − 1) + f(1) f(b) ≥ f (1)(b − 1) + f(1) f(c) ≥ f (1)(c − 1) + f(1). Cộng các bất đẳng thức cùng chiều, ta có f(a) + f(b) + f(c) ≥ f (1)(a + b + c − 3) + 3f(1) = 1. Đẳng thức xay ra khi và chỉ khi a = b = c = 1. Nhận xét 2.3. Bài toán có thể cho dưới hình thức sau: Với các số thực dương a, b, c thỏa mãn a2 + b2 + c2 = 3. Chứng minh rằng 1 √ 1 + 8a + 1 √ 1 + 8b + 1 √ 1 + 8c ≥ 1. Bài toán 2.50 (Đề đại học khối A năm 2003). Với các số thực dương x, y, z thỏa mãn x + y + z = 1. Chứng minh rằng x2 + 1 x2 + y2 + 1 y2 + z2 + 1 z2 ≥ √ 82. Chứng minh. 42
  • 44. Chương 2. Bất đẳng thức có tổng các biến không đổi Xét hàm số f(t) = t2 + 1 t2 , t ∈ (0; 1), f (t) = (t2 + 1 t2 ) 2 t2 + 1 t2 = t4 − 1 √ t8 + t4 f (t) = 4t3 √ t8 + t4 − (t4 − 1) 8t7 + 4t3 2 √ t8 + t4 t8 + t4 = 6t4 + 2 (t5 + t) √ t8 + t4 > 0, ∀t ∈ (0; 1). Áp dụng bổ đề 1.1, ta có f(x) ≥ f ( 1 3 )(x − 1 3 ) + f( 1 3 ) f(y) ≥ f ( 1 3 )(y − 1 3 ) + f( 1 3 ) f(z) ≥ f ( 1 3 )(z − 1 3 ) + f( 1 3 ). Cộng các bất đẳng thức cùng chiều, ta có x2 + 1 x2 + y2 + 1 y2 + z2 + 1 z2 ≥ f ( 1 3 )(x + y + z − 1) + 3f( 1 3 ) = 3f( 1 3 ) = √ 82. Đẳng thức xảy ra khi và chỉ khi x = y = z = 1 3 . 2.2.4 Bài toán liên quan Bài toán 2.51. Với a, b, c không âm thỏa mãn a + b + c = 1. Chứng minh rằng 4 √ ab3 + 4 √ bc3 + 4 √ ca3 ≤ 1. Bài toán 2.52. Với a, b, c > 0 thỏa mãn a + b + c = 3. Chứng minh rằng a + b c + ab + b + c a + bc + c + a b + ac ≥ 3. 43
  • 45. Chương 2. Bất đẳng thức có tổng các biến không đổi Bài toán 2.53. Với a, b, c > 0 thỏa mãn a + b + c = 3. Chứng minh rằng với ∀k > 0 thì (b + c) k bc + 1 a2 + 1 + (c + a) k ca + 1 b2 + 1 + (a + b) k ab + 1 c2 + 1 ≥ 6. Bài toán 2.54 (Ukraine MO 2001). Với các số thực dương a, b, c, x, y, z thỏa mãn x + y + z = 1. Chứng minh rằng ax + by + cz + 2 (xy + yz + zx)(ab + bc + ca) ≤ a + b + c. 44
  • 46. Chương 3 Bất đẳng thức có tích các biến không đổi Khi bất đẳng thức có dạng P(x, y, z) ≥ 0 hoặc(≤ 0) với tích xyz không đổi, thì trong các kĩ thuật chứng minh bất đẳng thức bằng các sử dụng AM-GM, bất đẳng thức Cauchy-Schwarz, các tính chất của hàm số thường dùng, ta có thể sử dụng phép thế để đưa bất đẳng thức cần chứng minh về bất đẳng thức đơn giản hơn. Cho các số thực dương a, b, c. Nếu abc = 1 thì có thể đặt a = 1 x , b = 1 y , c = 1 z hoặc a = x y , b = y z , c = z x hoặc a = yz x2 , b = xz y2 , c = xy z2 , . . . 3.1 Bất đẳng thức có tích các biến không đổi với hàm phân thức hữu tỉ 3.1.1 Sử dụng bất đẳng thức AM-GM Bài toán 3.1. Với các số dương x, y, z thỏa mãn điều kiện xyz = 1. Chứng minh rằng với ∀n ∈ N∗, ta luôn có xn y + yn z + zn x ≥ xy + yz + zx. Chứng minh. Áp dụng bất đẳng thức AM-GM cho (n+1) số, ta có xn y + xn y + yn z + yz + yz + ... + yz n−2 ≥ (n + 1) n+1 (xny)2ynz(yz)n−2 = (n + 1) n+1 x2ny2nzn−1 = (n + 1) n+1 (xy)2n 1 (xy)n−1 = (n + 1)xy. 45
  • 47. Chương 3. Bất đẳng thức có tích các biến không đổi Tương tự, ta có yn z + yn z + zn x + zx + zx + ... + zx n−2 ≥ (n + 1) n+1 (ynz)2znx(zx)n−2 = (n + 1)yz zn x + zn x + xn y + xy + xy + ... + xy n−2 ≥ (n + 1) n+1 (znx)2xny(xy)n−2 = (n + 1)zx. Cộng các bất đẳng thức cùng chiều trên, ta có 3(xn y + yn z + zn x) + (n − 2)(xy + yz + zx) ≥ (n + 1)(xy + yz + zx) ⇔ xn y + yn z + zn x ≥ xy + yz + zx. Đẳng thức xảy ta khi và chỉ khi x = y = z = 1. Bài toán 3.2. Với các số thực dương x, y, z có xyz = 1. Chứng minh rằng x4y x2 + 1 + y4z y2 + 1 + z4x z2 + 1 ≥ 3 2 . Chứng minh. Sử dụng bất đẳng thức AM-GM, ta có x4y x2 + 1 = x2 y − x2y x2 + 1 ≥ x2 y − x2y 2x = x2 y − xy 2 . Chứng minh tương tự, ta có y4z y2 + 1 ≥ y2 z − yz 2 z4x z2 + 1 ≥ z2 x − zx 2 . Cộng các bất đẳng thức cùng chiều chúng ta có x4y x2 + 1 + y4z y2 + 1 + z4x z2 + 1 ≥ x2 y + y2 z + z2 x − 1 2 (xy + yz + zx) . 46
  • 48. Chương 3. Bất đẳng thức có tích các biến không đổi Bất đẳng thức được chứng minh nếu ta chứng minh được x2 y + y2 z + z2 x ≥ 3 2 + xy + yz + zx 2 . Theo bất đẳng thức AM-GM, ta có x2y + y2z + z2x 2 ≥ 3 3 x3y3z3 2 = 3 2 . Vậy bất đẳng thức được chứng minh hoàn tất nếu ta chứng minh được x2y + y2z + z2x 2 ≥ xy + yz + zx. Thật vậy sử dụng bất đẳng thức AM-GM, ta có x2 y + x2 y + y2 z ≥ 3 3 x2y.x2y.y2z = 3 3 x4y4 1 xy = 3xy y2 z + y2 z + z2 x ≥ 3 3 y2z.y2z.z2x = 3 3 y4z4 1 yz = 3yz z2 z + z2 x + x2 y ≥ 3 3 z2x.z2x.x2y = 3 3 x4z4 1 xz = 3zx. Cộng các bất đẳng thức trên và rút gọn ta có điều phải chứng minh. Đẳng thức xảy ra khi và chỉ khi x = y = z = 1. Bài toán 3.3 (IMO 1995). Với a, b, c là các số thực dương và abc = 1. Chứng minh rằng 1 a3(b + c) + 1 b3(c + a) + 1 c3(a + b) ≥ 3 2 . Chứng minh. Theo bất đẳng thức AM-GM, ta có 1 a3(b + c) + b + c 4bc ≥ 2 b + c 4a3bc(b + c) = 1 a 1 b3(a + c) + a + c 4ac ≥ 2 a + c 4b3ac(a + c) = 1 b 1 c3(a + b) + a + b 4ab ≥ 2 a + b 4c3ab(a + b) = 1 c . Cộng các bất đẳng thức cùng chiều, ta có 1 a3(b + c) + 1 b3(c + a) + 1 c3(a + b) + 1 4 b + c bc + a + c ac + a + b ab ≥ 1 a + 1 b + 1 c ⇔ 1 a3(b + c) + 1 b3(c + a) + 1 c3(a + b) ≥ 1 2 1 a + 1 b + 1 c ≥ 3 2 3 1 abc = 3 2 . Đẳng thức xảy ra khi và chỉ khi a = b = c = 1. 47
  • 49. Chương 3. Bất đẳng thức có tích các biến không đổi Bài toán 3.4 (Romania MO 1997 ). Với các số thực dương x, y, z thỏa mãn xyz = 1. Chứng minh rằng x9 + y9 x6 + x3y3 + y6 + y9 + z9 y6 + y3z3 + z6 + z9 + x9 z6 + z3x3 + x6 ≥ 2. Chứng minh. Đặt a = x3, b = y3, c = z3. Khi đó bất đẳng thức cần chứng minh trở thành a3 + b3 a2 + ab + b2 + b3 + c3 b2 + bc + c2 + c3 + a3 c2 + ca + a2 ≥ 2. Mặt khác, với ∀u, v > 0, ta có u3 + v3 u2 + uv + v2 − u + v 3 = 2(u + v)(u − v)2 3(u2 + uv + v2) ≥ 0 luôn đúng . Do đó a3 + b3 a2 + ab + b2 + b3 + c3 b2 + bc + c2 + c3 + a3 c2 + ca + a2 ≥ a + b 3 + b + c 3 + c + a 3 ≥ 2 3 3 3 √ abc = 2. Đẳng thức xảy ra khi và chỉ khi a = b = c hay x = y = z = 1. Bài toán 3.5 (Korea MO 1999). Với các số thực dương a, b, c thỏa mãn abc = 1. Chứng minh rằng 1 a + b4 + c4 + 1 a4 + b + c4 + 1 a4 + b4 + c ≤ 1. Chứng minh. Áp dụng bất đẳng thức AM-GM, ta có b4 + c4 = 3b4 + c4 4 + b4 + 3c4 4 ≥ 4 4 √ b4b4b4c4 4 + 4 4 √ b4c4c4c4 4 = bc(b2 + c2 ) ≥ b2 + c2 a . Do đó 1 a + b4 + c4 ≤ 1 a + b2 + c2 a = a a2 + b2 + c2 . Chứng minh tương tự, ta có 1 a4 + b + c4 ≤ b a2 + b2 + c2 48
  • 50. Chương 3. Bất đẳng thức có tích các biến không đổi 1 a4 + b4 + c ≤ c a2 + b2 + c2 . Cộng các bất đẳng thức cùng chiều, ta có 1 a + b4 + c4 + 1 a4 + b + c4 + 1 a4 + b4 + c ≤ a + b + c a2 + b2 + c2 ≤ a + b + c (a + b + c)2 3 = 3 a + b + c ≤ 3 3 3 √ abc ≤ 1. Đẳng thức xảy ra khi và chỉ khi a = b = c = 1. Bài toán 3.6 (Hongkong 2000). Với các số thực dương a, b, c thỏa mãn điều kiện abc = 1. Chứng minh rằng 1 + ab2 c3 + 1 + bc2 a3 + 1 + ca2 b3 ≥ 18 a3 + b3 + c3 . Chứng minh. Bất đẳng thức cần chứng minh tương đương với 1 a3 + 1 b3 + 1 c3 + bc2 a3 + ca2 b3 + ab2 c3 ≥ 18 a3 + b3 + c3 ⇔(a3 + b3 + c3 )( 1 a3 + 1 b3 + 1 c3 ) + (a3 + b3 + c3 ) bc2 a3 + ca2 b3 + ab2 c3 ≥ 18. Áp dụng bất đẳng thức AM-GM, ta có a3 + b3 + c3 1 a3 + 1 b3 + 1 c3 ≥ 3 3 √ a3b3c33 3 1 a3b3c3 = 9 a3 + b3 + c3 bc2 a3 + ca2 b3 + ab2 c3 ≥ 3 3 √ a3b3c3.3 3 a3b3c3 a3b3c3 = 9. Cộng vế với vế ta có điều phải chứng minh. Đẳng thức xảy ra khi và chỉ khi a = b = c = 1. 49
  • 51. Chương 3. Bất đẳng thức có tích các biến không đổi 3.1.2 Sử dụng bất đẳng thức Cauchy-Schwarz Bài toán 3.7. Với a, b, c là các số thực dương thỏa mãn abc = 1. Chứng minh rằng 1 1 + a + b2 + 1 1 + b + c2 + 1 1 + c + a2 ≤ 1. Chứng minh. Đặt a = x3, b = y3, c = x3, với x, y, z > 0 và xyz = 1. Bất đẳng thức cần chứng minh trở thành 1 1 + x3 + y6 + 1 1 + y3 + z6 + 1 1 + z3 + x6 ≤ 1 Sử dụng bất đẳng thức Cauchy- Schwarz kết hợp với xyz = 1, ta có 1 1 + x3 + y6 = z4 + x + 1 z2 (1 + x3 + y6)(z4 + x + 1 z2 ) ≤ z4 + x + 1 z2 (z2 + x2 + y2)2 = z4 + x2yz + x2y2 (x2 + y2 + z2)2 . Chứng minh tương tự, ta có 1 1 + y3 + z6 = x4 + y + 1 z2 (1 + y3 + z6)(x4 + y + 1 z2 ) ≤ x4 + y2xz + x2y2 (x2 + y2 + z2)2 1 1 + z3 + x6 = y4 + z + 1 x2 (1 + z3 + x6)(y4 + z + 1 x2 ) ≤ y4 + z2xy + y2z2 (x2 + y2 + z2)2 . Cộng các bất đẳng thức cùng chiều, ta có 1 1 + x3 + y6 + 1 1 + y3 + z6 + 1 1 + z3 + x6 ≤ x4 + y4 + z4 + xyz(x + y + z) + x2y2 + y2z2 + z2x2 (x2 + y2 + z2)2 . Bất đẳng thức được chứng minh nếu ta chứng minh được x4 + y4 + z4 + xyz(x + y + z) + x2y2 + y2z2 + z2x2 (x2 + y2 + z2)2 ≤ 1 ⇔xyz(x + y + z) ≤ x2 y2 + y2 z2 + z2 x2 (điều này luôn đúng). Đẳng thức xảy ra khi và chỉ khi a = b = c = 1. 50
  • 52. Chương 3. Bất đẳng thức có tích các biến không đổi Bài toán 3.8. Với các số thực dương x, y, z thỏa mãn xyz = 1. Chứng minh 1 x2 + x + 1 + 1 y2 + y + 1 + 1 z2 + z + 1 ≥ 1. Chứng minh. Đặt x = bc a2 , y = ca b2 , z = ab c2 , với a, b, c > 0. Khi đó bất đẳng thức trở thành a4 b2c2 + a2bc + a4 + b4 c2a2 + b2ca + b4 + c4 a2b2 + c2ab + c4 ≥ 1. Áp dụng bất đẳng thức Cauchy- Schwarz, ta có a4 b2c2 + a2bc + a4 + b4 c2a2 + b2ca + b4 + c4 a2b2 + c2ab + c4 ≥ (a2 + b2 + c2)2 b2c2 + a2bc + a4 + c2a2 + b2ca + b4 + a2b2 + c2ab + c4 . Bất đẳng thức được chứng minh nếu ta chỉ ra được (a2 + b2 + c2 )2 ≥ a4 + b4 + c4 + abc(a + b + c) + a2 b2 + b2 c2 + c2 a2 . Thật vậy (a2 + b2 + c2 )2 ≥ a4 + b4 + c4 + abc(a + b + c) + a2 b2 + b2 c2 + c2 a2 ⇔a2 b2 + b2 c2 + c2 a2 ≥ abc(a + b + c) ⇔(ab − ac)2 + (ab − bc)2 + (bc − ca)2 ≥ 0 ( luôn đúng). Đẳng thức xảy ra khi và chỉ khi x = y = z = 1. Bài toán 3.9 (IMO 2008). Với các số thực x, y, z khác 1 và thỏa mãn xyz = 1. Chứng minh x x − 1 2 + y y − 1 2 + z z − 1 2 ≥ 1. Chứng minh. Do x, y, z = 1 và xyz = 1 nên ∃a, b, c thỏa mãn x = a2 bc , y = b2 ca , z = c2 ab và (a2 − bc)(b2 − ca)(c2 − ab) = 0. Bất đẳng thức cần chứng minh trở thành a4 (b2 − bc)2 + b4 (b2 − ca)2 + c4 (c2 − ab)2 ≥ 1. 51
  • 53. Chương 3. Bất đẳng thức có tích các biến không đổi Áp dụng bất đẳng thức Cauchy-Schwarz, ta có a4 (b2 − bc)2 + b4 (b2 − ca)2 + c4 (c2 − ab)2 ≥ (a2 + b2 + c2)2 (a2 − bc)2 + (b2 − ca)2 + (c2 − ab)2 . Bất đẳng thức đã cho được chứng minh nếu ta chứng minh được (a2 + b2 + c2 )2 ≥ (a2 − bc)2 + (b2 − ac)2 + (c2 − ab)2 ⇔(ab + bc + ca)2 ≥ 0 (luôn đúng). Nhận xét 3.1. Bài toán trên có thể làm cách hai như sau Đặt a = x x − 1 , b = y y − 1 , c = z z − 1 suy ra x = a a − 1 , y = b b − 1 , z = c c − 1 . Khi đó bất đẳng thức cần chứng minh trở thành a2 + b2 + c2 ≥ 1 Theo giả thiết xyz = 1 ⇒ a a − 1 b b − 1 c c − 1 = 1 ⇔(a − 1)(b − 1)(c − 1) = abc ⇔(a + b + c) − 1 = ab + bc + ca ⇔2(a + b + c) − 2 = 2(ab + bc + ca) ⇔a2 + b2 + c2 − 1 = (a + b + c − 1)2 . Do vế trái luôn không âm nên ta có a2 + b2 + c2 ≥ 1 ( điều phải chứng minh). Bài toán 3.10 (IMO 1995). Với các số thực dương a, b, c và abc = 1. Chứng minh 1 a3(b + c) + 1 b3(c + a) + 1 c3(a + b) ≥ 3 2 . Chứng minh. Áp dụng bất đẳng thức Cauchy -Schwarz, ta có 1 a3(b + c) + 1 b3(c + a) + 1 c3(a + b) ≥ 1 a + 1 b + 1 c 2 a(b + c) + b(c + a) + c(a + b) = (ab + bc + ca)2 a(b + c) + b(c + a) + c(a + b) . 52
  • 54. Chương 3. Bất đẳng thức có tích các biến không đổi Bất đẳng thức được chứng minh nếu ta chứng minh được (ab + bc + ca)2 ≥ 3 2 [a(b + c) + b(c + a) + c(a + b)] ⇔ab + bc + ca ≥ 3 ( luôn đúng thao AM-GM). Dấu bằng xảy ra khi và chỉ khi a = b = c = 1. 3.1.3 Sử dụng các tính chất của hàm số Bài toán 3.11 (Romania MO 2003). Với các số thực a, b, c > 0 và thỏa mãn abc = 1. Chứng minh rằng 1 + 3 a + b + c ≥ 6 ab + bc + ca . Chứng minh. Bất đẳng thức cần chứng minh tương đương với 1 + 3 a + b + c − 6 ab + bc + ca ≥ 0. Đặt √ a + b + c = t với t ≥ 3 3 √ abc = √ 3. Ta có 1 + 3 a + b + c − 6 ab + bc + ca ≥1 + 3 a + b + c − 6 3abc(a + b + c) =1 + 3 a + b + c − 6 3(a + b + c) =1 + 3 t2 − 6 √ 3t . Xét hàm số f(t) = 1 + 3 t2 − 6 √ 3t với t ≥ √ 3 f (t) = −6 t3 + 2 √ 3 t2 = 2 t2 √ 3 − 3 t ≥ 0 với ∀t ≥ √ 3. Do đó f(t) ≥ f( √ 3) = 0. Vậy bất đẳng thức được chứng minh. 53
  • 55. Chương 3. Bất đẳng thức có tích các biến không đổi Bài toán 3.12. Với a, b, c là các số thực dương thỏa mãn abc = 1. Chứng minh rằng 1 a2 − a + 1 + 1 b2 − b + 1 + 1 c2 − c + 1 ≤ 3. Chứng minh. Bất đẳng thức được viết lại như sau 4 3 − 1 a2 − a + 1 + 4 3 − 1 b2 − b + 1 + 4 3 − 1 c2 − c + 1 ≥ 1 ⇔ (2a − 1)2 a2 − a + 1 + (2b − 1)2 b2 − b + 1 + (2c − 1)2 c2 − c + 1 ≥ 3. Áp dụng bất đẳng thức Cauchy-Schwarz, ta được (2a − 1)2 a2 − a + 1 + (2b − 1)2 b2 − b + 1 + (2c − 1)2 c2 − c + 1 ≥ (2a + 2b + 2c − 3)2 a2 + b2 + c2 − (a + b + c) + 3 . Do đó bất đẳng thức được chứng minh nếu ta chỉ ra được (2a + 2b + 2c − 3)2 ≥ 3 a2 + b2 + c2 − (a + b + c) + 3 ⇔4(a + b + c)2 − 12(a + b + c) + 9 ≥ 3(a2 + b2 + c2 ) − 3(a + b + c) + 9 ⇔(a2 + b2 + c2 ) − 9(a + b + c) + 8(ab + bc + ca) ≥ 0 ⇔(a + b + c)2 + 6(ab + bc + ca) ≥ 9(a + b + c) ( ) Ta có (ab + bc + ca)2 ≥ 3abc(a + b + c) = 3(a + b + c) suy ra ab + bc + ca ≥ 3(a + b + c). Đặt t = a + b + c 3 với t ≥ 1. Do đó muốn chứng minh được bất đẳng thức ( ) ta chỉ cần chứng minh 9t4 + 18t ≥ 9.3t2 ⇔t3 − 3t + 2 ≥ 0. ( ) Xét hàm f(t) = t3 − 3t + 2, với t ≥ 1 f (t) = 3t2 − 3 ≥ 0 với ∀t ≥ 1. Do đó f(t) ≥ f(1) = 0. Nên bất đẳng thức ( ) được chứng minh. Đẳng thức xảy ra khi và chỉ khi a = b = c = 1. 54
  • 56. Chương 3. Bất đẳng thức có tích các biến không đổi 3.1.4 Bài toán liên quan Bài toán 3.13. Với các số thực dương a, b, c thỏa mãn abc = 1. Chứng minh rằng a + b + c ≥ 1 + a 1 + b + 1 + b 1 + c + 1 + c 1 + a . Bài toán 3.14 (IMO -SL 1998). Với các số thực dương a, b, c thỏa mãn abc = 1. Chứng minh rằng a3 (1 + b)(1 + c) + b3 (1 + a)(1 + c) + c3 (1 + a)(1 + b) ≥ 3 4 . Bài toán 3.15 (Ukraine 1998 ). Với các số thực dương a, b, c > 0 và thỏa mãn abc = 1. Chứng minh rằng 1 + ab 1 + a + 1 + bc 1 + b + 1 + ca 1 + a ≥ 3. Bài toán 3.16. Với các số thực a, b, c > 0 và thỏa mãn abc = 1. Chứng minh rằng (a + b)(b + c)(c + a) ≥ 4(a + b + c − 1). Bài toán 3.17. Với a, b, c là các số thực dương thỏa mãn abc = 1. Chứng minh rằng 1 a2 − a + 1 + 1 b2 − b + 1 + 1 c2 − c + 1 ≤ 3. Bài toán 3.18. Với các số thực dương a, b, c thỏa mãn điều kiện abc = 1. Chứng minh rằng 1 (a + 1)(a + 2) + 1 (b + 1)(b + 2) + 1 (c + 1)(c + 2) ≥ 1 2 . Bài toán 3.19. Với ba số thực dương a, b, c thỏa mãn điều kiện abc = 1. Chứng minh bất đẳng thức sau 1 (1 + a)2 + 1 (1 + b)2 + 1 (1 + c)2 + 2 (1 + a)(1 + b)(1 + c) ≥ 1. 55
  • 57. Chương 3. Bất đẳng thức có tích các biến không đổi 3.2 Bất đẳng thức có tích các biến không đổi với hàm vô tỉ 3.2.1 Sử dụng bất đẳng thức AM-GM Bài toán 3.20 (Internationnal Compelition SRMC 2006). Với các số thực dương a, b, c thỏa mãn abc = 1. Chứng minh rằng 4 3 a b + 3 b c + 3 c a ≤ 3 2 + a + b + c + 1 a + 1 b + 1 c 2 3 . Chứng minh. Do abc = 1 nên tồn tại các số dương x, y, z sao cho a = y x , b = z y , c = x z . Thay vào bất đẳng thức đã cho, ta có 4 3 y2 xz + 3 z2 xy + 3 x2 yz ≤ 3 2 + y x + z y + x z + x y + y z + z x 2 3 ⇔4 3 y3 xyz + 3 z3 xyz + 3 x3 xyz ≤ 3 2xyz + y2z + xz2 + x2y + x2z + y2x + z2y xyz 2 3 ⇔4 x + y + z 3 √ xyz ≤ 3 (x + y)(y + z)(z + x) xyz 2 3 ⇔xyz(x + y + z)3 ≤ 27 64 (x + y)2 (y + z)2 (z + x)2 . Mà ta lại chứng minh được bất đẳng thức (x + y)(y + z)(z + x) ≥ 8 9 (x + y + z)(xy + yz + zx) Do đó bất đẳng thức được chứng minh nếu ta chứng minh được 27 64 64 81 (x + y + z)2 (xy + yz + zx)2 ≥ xyz(x + y + z)3 ⇔(xy + yz + zx)2 ≥ 3xyz(x + y + z) hiển nhiên đúng theo AM-GM . Đẳng thức xảy ra khi và chỉ khi a = b = c = 1. Bài toán 3.21. Với a, b, c là các số thực dương thỏa mãn điều kiện abc = 1. Chứng minh rằng 3 √ a + 3 √ b + 3 √ c ≤ 3 3(3 + a + b + c + ab + bc + ca). 56
  • 58. Chương 3. Bất đẳng thức có tích các biến không đổi Chứng minh. Vì abc = 1 nên đặt a = x2 yz , b = y2 zx , c = z2 xy . . Khi đó bất đẳng thức cần chứng minh trở thành 3 x2 yz + 3 y2 zx + 3 z2 xy ≤ 3 3 3 + x2 yz + y2 xz + z2 xy + xy z2 + yz x2 + xz y2 = 3 3 x y + y z + z x x z + y x + z y = 3 (1 + 1 + 1) x y + y z + z x x z + y x + z y . Áp dụng bất đẳng thức AM − GM, ta có 1 3 + x y x y + y z + z x + x z x z + y x + z y ≥ 3. 3 x2 yz 3 x y + y z + z x x z + y x + z y 1 3 + y z x y + y z + z x + y x x z + y x + z y ≥ 3 3 y2 xz 3 x y + y z + z x x z + y x + z y 1 3 + z x x y + y z + z x + z y x z + y x + z y ≥ 3 3 z2 xy 3 x y + y z + z x x z + y x + z y . Cộng các bất đẳng thức cùng chiều ta có điều phải chứng minh. Đẳng thức xảy ra khi và chỉ khi a = b = c = 1. Bài toán 3.22 (APMO 2005). Với a, b, c là các số thực dương thỏa mãn abc = 8. Chứng minh rằng a2 (a3 + 1)(b3 + 1) + b2 (b3 + 1)(c3 + 1) + c2 (c3 + 1)(a3 + 1) ≥ 4 3 . Chứng minh. 57
  • 59. Chương 3. Bất đẳng thức có tích các biến không đổi Với ∀x ≥ 0, áp dụng bất đẳng thức AM-GM, ta có x3 + 1 = (x + 1)(x2 − x + 1) ≤ x + 1 + x2 − x + 1 2 = x2 + 2 2 . Áp dụng bất đẳng thức này, ta có a2 (a3 + 1)(b3 + 1) + b2 (b3 + 1)(c3 + 1) + c2 (c3 + 1)(a3 + 1) ≥ 4a2 (a2 + 2)(b2 + 2) + 4b2 (b2 + 2)(c2 + 2) + 4c2 (c2 + 2)(a2 + 2) . Vậy ta chỉ cần chứng minh 4a2 (a2 + 2)(b2 + 2) + 4b2 (b2 + 2)(c2 + 2) + 4c2 (c2 + 2)(a2 + 2) ≥ 4 3 ⇔a2 (c2 + 2) + b2 (a2 + 2) + c2 (b2 + 2) ≥ 1 3 (a2 + 2)(b2 + 2)(c2 + 2) ⇔a2 b2 + b2 c2 + c2 a2 + 2(a2 + b2 + c2 ) ≥ a2 b2 c2 + 8 = 72. Điều này hiển nhiên đúng vì theo bất đẳng thức AM-GM, ta có a2 b2 + b2 c2 + c2 a2 ≥ 3 3 √ a4b4c4 = 48 a2 + b2 + c2 ≥ 3 3 √ a2b2c2 = 12. Đẳng thức xảy ra khi và chỉ khi a = b = c = 2. Bài toán 3.23 (Đề thi đại học khối D 2005). Với ba số thực dương x, y, z thỏa mãn điều kiện xyz = 1. Chứng minh rằng 1 + x3 + y3 xy + 1 + y3 + z3 yz + √ 1 + z3 + x3 zx ≥ 3 √ 3. Chứng minh. Áp dụng bất đẳng thức AM-GM, ta có 1 + x3 + y3 xy ≥ 3 3 x3y3 xy = 3 xy . Tương tự, ta có √ 1 + z3 + x3 yz ≥ 3 yz 58
  • 60. Chương 3. Bất đẳng thức có tích các biến không đổi √ 1 + z3 + x3 xz ≥ 3 xz . Cộng các bất đẳng thức cùng chiều ta có 1 + x3 + y3 xy + 1 + y3 + z3 yz + √ 1 + z3 + x3 zx ≥ 3 xy + 3 yz + 3 xz . Mặt khác theo bất đẳng thức AM-GM, ta có 3 xy + 3 yz + 3 zx ≥ 3 3 √ 27 x2y2z2 = 3 √ 3. Bất đẳng thức được chứng minh. Đẳng thức xảy ra khi và chỉ khi x = y = z = 1. 3.2.2 Sử dụng bất đẳng thức Cauchy-Schwarz Bài toán 3.24. Với a, b, c là các số thực dương thỏa mãn abc = 1. Chứng minh rằng √ a 2 + b √ a + √ b 2 + c √ b + √ c 2 + a √ c ≥ 1. Chứng minh. Đặt √ a = x y , √ b = y z , √ c = z x với x, y, z > 0. Khi đó bất đẳng thức cần chứng minh trở thành x y 2 + y2 z2 . x y + y z 2 + z2 x2 . y z + z x 2 + x2 y2 . z x ≥ 1 ⇔ x2z2 2xyz2 + x2y2 + y2x2 2x2yz + y2z2 + z2y2 2xy2z + x2z2 ≥ 1. Áp dụng bất đẳng thức Cauchy-Schwarz, ta có x2z2 2xyz2 + x2y2 + y2x2 2x2yz + y2z2 + z2y2 2xy2z + x2z2 ≥ (xz + xy + yz)2 x2y2 + y2z2 + z2x2 + 2xyz(x + y + z) = (xz + xy + yz)2 (xz + xy + yz)2 = 1. Bất đẳng thức được chứng minh. Dấu bằng xảy ra khi à chỉ khi a = b = c = 1. 59
  • 61. Chương 3. Bất đẳng thức có tích các biến không đổi Bài toán 3.25 (Balkan Shortlist 2002). Với a, b, c, là các số thực dương thỏa mãn abc = 2. Chứng minh rằng a3 + b3 + c3 ≥ a √ b + c + b √ c + a + c √ a + b. Chứng minh. Do 2(a3 + b3 + c3 ) − a2 (b + c) − b2 (c + a) − c2 (a + b) = a3 + b3 − ab(a + b) + b3 + c3 − bc(b + c) + c3 + a3 − ca(a + c) =(a + b)(a − b)2 + (b + c)(b − c)2 + (c + a)(c − a)2 ≥ 0 luôn đúng với ∀a, b, c . Do đó ta cần chứng minh bất đẳng thức a2 (b + c) + b2 (c + a) + c2 (a + b) ≥ 2(a √ b + c + b √ c + a + c √ a + b). Sử dụng Cauchy- Schwarz cho biểu thức ở vế trái, ta có a2 (b + c) + b2 (c + a) + c2 (a + b) ≥ 1 3 (a √ b + c + b √ c + a + c √ a + b)2 . Do đó ta chỉ cần chứng minh a √ b + c + b √ c + a + c √ a + b ≥ 6. Sử dụng Bất đẳng thức AM-GM, ta có a √ b + c + b √ c + a + c √ a + b ≥ 3 abc (a + b)(b + c)(c + a) ≥ 3 abc 2 √ ab.2 √ bc.2 √ ca = 6. Đẳng thức xảy ra khi và chỉ khi a = b = c = 3 √ 2. 3.2.3 Sử dụng các tính chất của hàm số Bài toán 3.26 (IMO 2002 ). Với a, b, c là các số thực dương thỏa mãn điều kiện abc = 1. Chứng minh rằng a − 1 + 1 b b − 1 + 1 c c − 1 + 1 a ≤ 1 Chứng minh. 60
  • 62. Chương 3. Bất đẳng thức có tích các biến không đổi Vì abc = 1 nên tồn tại các số dương x, y, z sao cho a = x y , b = y z , c = z x . Bất đẳng thức cần chứng minh trở thành (x − y + z)(y − z + x)(z − x + y) ≤ xyz. Ta để ý rằng, (x − y + z) + (y − z + x) = 2x > 0 (y − z + x) + (z − x + y) = 2y > 0 (z − x + y) + (x − y + z) = 2z > 0. Do đó, trong ba số x−y +z, y −z +x, z −x+y không thể có trường hợp hai số cùng âm hoặc cả ba số đều âm. Nếu trong ba số trên có một số âm, hiển nhiên ta có bất đẳng thức cần chứng minh. Trường hợp cả ba số đều dương, bằng các lấy logarit hai vế, ta có ln(x − y + z) + ln(y − z + x) + ln(z − x + y) ≤ lnx + lny + lnz. Không mất tính tổng quát, giả sử x ≥ y ≥ z. Khi đó,    y − z + x ≥ x (y − z + x) + (x − y + z) ≥ x + y (y − z + x) + (x − y + z) + (z − x + y) = x + y + z. Vì f(x) = lnx là hàm lõm trên (0; +∞), do đó, sử dụng bất đẳng thức Kara- mata, ta được ln(y − z + x) + ln(x − y + z) + ln(z − x + y) ≤ lnx + lny + lnz. Đẳng thức xảy ra khi và chỉ khi x = y = z hay a = b = c = 1. Bài toán 3.27. Với các số thực dương a, b, c thỏa mãn abc = 1. Chứng minh rằng a √ 1 + a + b √ 1 + b + c √ 1 + c ≥ 3 √ 2 2 . Chứng minh. Từ giả thiết abc = 1 ⇔ ln(abc) = 0 ⇔ ln a + ln b + ln c = 0. Đặt x = ln a, y = ln b, z = ln c. Khi đó x, y, z ∈ R và x + y + z = 0. Bất đẳng thức đã cho trở thành ex √ 1 + ex + ey √ 1 + ey + ez 1 + ez ≥ 3 √ 2 2 . 61
  • 63. Chương 3. Bất đẳng thức có tích các biến không đổi Xét hàm số f(t) = et √ 1 + et − 3 √ 2 8 t f (t) = e2t + 2et 2 (1 + et)3 − 3 √ 2 8 f (t) = 0 ⇔4(e2t + 2et ) = 3 √ 2 (1 + et)3 ⇔ (et − 1)(16e3t + 62e2t + 72et + 18) = 0 ⇔ et = 1 16e3t + 62e2t + 72et + 18 = 0 ( Vô nghiệm) ⇔ t = 0. Ta có bảng biến thiên t −∞ 0 +∞ f (t) − 0 + f(t) √ 2 2 Từ bảng biến thiên, ta có f(t) ≥ √ 2 2 ∀t ∈ R ⇔ et √ 1 + et ≥ 3 √ 2t 8 + √ 2 2 , ∀t ∈ R. Thay t bởi x, y, z rồi cộng các bất đẳng thức cùng chiều ta có ex √ 1 + ex + ey √ 1 + ey + ez 1 + ez ≥ 3 √ 2 8 (x + y + z) + 3 √ 2 2 = 3 √ 2 2 . Đẳng thức xảy ra khi và chỉ khi x = y = z = 0 hay a = b = c = 1. 3.2.4 Bài toán liên quan Bài toán 3.28. Với các số thực dương a, b, c thỏa mãn abc = 1. Chứng minh rằng 2(a2 + 1) + 2(b2 + 1) + 2(c2 + 1) ≤ 1 + 5 3 (a + b + c). Bài toán 3.29. Với các số thực dương a, b, c thỏa mãn abc = 1. Chứng minh rằng 8a2 + 1 + 8b2 + 1 + 8c2 + 1 ≤ 3 (a + b + c) . 62
  • 64. Chương 4 Một số lớp bài toán cực trị với đa thức đối xứng ba biến 4.1 Sử dụng bất đẳng thức AM-GM Bài toán 4.1. Với các số thực không âm a, b, c thỏa mãn a + b + c = 3. Tìm giá trị lớn nhất của biểu thức P = (a2 − ab + b2 )(b2 − bc + c2 )(c2 − ca + a2 ). Chứng minh. Không mất tính tổng quát ta có thể giả sử c = min{a, b, c}. Khi đó b2 − bc + c2 = c(c − b) + b2 ≤ b2 c2 − ca + a2 = c(c − a) + a2 ≤ a2 . Do đó P = (a2 − ab + b2 )(b2 − bc + c2 )(c2 − ca + a2 ) ≤ (a2 − ab + b2 )a2 b2 . Áp dụng bất đẳng thức AM-GM, ta có (a2 − ab + b2 )a2 b2 = 4 9 3ab 2 3ab 2 (a2 − ab + b2 ) ≤ 4 9    3ab 2 + 3ab 2 + (a2 − ab + b2) 3    3 63
  • 65. Chương 4. Một số lớp bài toán cực trị với đa thức đối xứng ba biến = 4 9 (a2 + 2ab + b2)3 27 = 4 9 (a + b)6 27 ≤ 4 9.27 (a + b + c)6 = 12. Đẳng thức xảy ra khi và chỉ khi a = 2, b = 1, c = 0 và các hoán vị của nó. Vậy giá trị lớn nhất của P là 12. Bài toán 4.2. Với a, b, c là các số thực dương thỏa mãn a + b + c = 2. Tìm giá trị lớn nhất của M = ab √ 2c + ab + bc √ 2a + bc + ca √ 2b + ca . Chứng minh. Sử dụng bất đẳng thức AM-GM, ta có M = ab √ 2c + ab + bc √ 2a + bc + ca √ 2b + ca = ab (a + b + c)c + ab + bc (a + b + c)a + bc + ca (a + b + c)b + ca = ab (b + c)(a + c) + bc (a + b)(a + c) + ca (b + c)(b + a) ≤ 1 2 ab c + a + ab c + b + bc a + b + bc a + c + ac b + a + ac b + c = 1 2 (a + b + c) = 1. Dấu bằng xảy ra khi và chỉ khi a = b = c = 2 3 . Vậy giá trị lớn nhất của biểu thức M là 1. Bài toán 4.3. Với a, b, c là các số thực dương thỏa mãn a + b + c = 1. Tìm giá trị lớn nhất của biểu thức P = ab + bc + ca + 5 2 (a + b) √ ab + (b + c) √ bc + (c + a) √ ca . Chứng minh. 64
  • 66. Chương 4. Một số lớp bài toán cực trị với đa thức đối xứng ba biến Áp dụng bất đẳng thức AM-GM, ta có 2(a + b)2 + 2ab = (a + b)2 2 + (a + b)2 2 + (a + b)2 2 + (a + b)2 2 + 2ab ≥5 5 ab(a + b)8 8 =5(a + b) 5 ab(a + b)3 8 ≥5(a + b) 5 ab(2 √ ab)3 8 =5(a + b) √ ab. Tương tự, ta có 2(b + c)2 + 2bc ≥ 5(b + c) √ bc 2(c + a)2 + 2ac ≥ 5(c + a) √ ca. Cộng các bất đẳng thức cùng chiều, ta có 5 (a + b) √ ab + (b + c) √ bc + (c + a) √ ca ≤ 4(a2 + b2 + c2 ) + 6(ab + bc + ca) ⇔2(ab + bc + ca) + 5 (a + b) √ ab + (b + c) √ bc + (c + a) √ ca ≤ 4(a2 + b2 + c2 ) + 8(ab + bc + ca) = 4(a + b + c)2 = 4. Đẳng thức xảy ra khi và chỉ khi a = b = c = 1 3 . Vậy giá trị lớn nhất của P là 2. Bài toán 4.4. Với các số thực dương a, b, c thỏa mãn a + b + c = 1. Tìm giá trị nhỏ nhất của biêu thức P = a2 + b b + c + b2 + c c + a + c2 + a a + b . Chứng minh. Ta có P + 1 = a2 + b b + c + a + b2 + c c + a + b + c2 + a a + b + c = a(a + b + c) + b b + c + b(a + b + c) + c c + a + c(a + b + c) + a a + b = a + b b + c + b + c c + a + c + a a + b . 65
  • 67. Chương 4. Một số lớp bài toán cực trị với đa thức đối xứng ba biến Áp dụng bất đẳng thức AM-GM, ta có P + 1 = a + b b + c + b + c c + a + c + a a + b ≥ 3 a + b b + c . b + c c + a . c + a a + b = 3. Đẳng thức xảy ra khi và chỉ khi a = b = c = 1 3 . Vậy giá trị nhỏ nhất của biểu thức P là 2. Bài toán 4.5. Với ba số thực dương a, b, c thỏa mãn abc = 1. Tìm giá trị lớn nhất của biểu thức P = ab a + b + ab + bc b + c + bc + ca c + a + ca . Chứng minh. Với giả thiết a, b, c > 0 và abc = 1, ta có P = ab a + b + ab + bc b + c + bc + ca c + a + ca = 1 1 a + 1 b + 1 + 1 1 b + 1 c + 1 + 1 1 c + 1 a + 1 . Với các số dương x, y, theo bất đẳng thức AM-GM, ta có x3 + x3 + y3 ≥ 3 3 x6y3 = 3x2 y y3 + y3 + x3 ≥ 3 3 y6x3 = 3y2 x. Do đó x3 + y3 ≥ xy(x + y) dấu ”=” xảy ra khi và chỉ khi x = y. Áp dụng bất đẳng thức trên ta có 1 a + 1 b + 1 ≥ 3 1 ab 1 3 √ a + 1 3 √ b + 1 = 3 √ c 1 3 √ a + 1 3 √ b + 1 = 3 √ cb + 3 √ ca + 3 √ ab 3 √ ab . 66
  • 68. Chương 4. Một số lớp bài toán cực trị với đa thức đối xứng ba biến Tương tự, ta có 1 b + 1 c + 1 ≥ 3 √ cb + 3 √ ca + 3 √ ab 3 √ bc 1 c + 1 a + 1 ≥ 3 √ cb + 3 √ ca + 3 √ ab 3 √ ac . Do đó T ≤ 3 √ ab 3 √ ab + 3 √ bc + 3 √ ca + 3 √ bc 3 √ ab + 3 √ bc + 3 √ ca + 3 √ ca 3 √ ab + 3 √ bc + 3 √ ca = 1. Đẳng thức xảy ra khi và chỉ khi a = b = c = 1. Vậy giá trị lớn nhất của P là 1. Bài toán 4.6. Với x, y, z là các số thực dương thay đổi thỏa mãn xyz = 1. Tìm giá trị nhỏ nhất của biểu thức P = x2(y + z) y √ y + 2z √ z + y2(z + x) z √ z + 2x √ x + z2(x + y) x √ x + 2y √ y . Chứng minh. Theo bất đẳng thức AM-GM, ta có x2 (y + z) ≥ 2x2√ yz = 2x √ x y2 (z + x) ≥ 2y2√ zx = 2y √ y z2 (x + y) ≥ 2z2√ xy = 2z √ z. Do đó P ≥ 2x √ x y √ y + 2z √ z + 2y √ y z √ z + 2x √ x + 2z √ z x √ x + 2y √ y . Đặt x √ x = a, y √ y = b, z √ z = c, suy ra P ≥ 2 a b + 2c + b c + 2a + c a + 2b = 2 a b + 2c + b c + 2a + c a + 2b Do đó áp dụng bất đẳng thức Cauchy-Schwarz và AM-GM, ta có P ≥ 2 (a + b + c)2 3(ab + bc + ca) ≥ 2(ab + bc + ca) ab + bc + ca = 2. 67
  • 69. Chương 4. Một số lớp bài toán cực trị với đa thức đối xứng ba biến Đẳng thức xảy ra khi và chỉ khi x = y = z = 1. Vậy giá trị nhỏ nhất của P là 2. Bài toán 4.7. Với a, b, c là các số thực dương thỏa mãn điều kiện abc = 1. Tìm giá trị lớn nhất của biểu thức P = a3 + b3 a2 + ab + b2 + b3 + c3 b2 + bc + c2 + c3 + a3 c2 + ca + a2 . Chứng minh. Ta có P = (a + b)(a2 − ab + b2) a2 + ab + b2 + (b + c)(b2 − bc2 c) b2 + bc + c2 + (c + a)(c2 − ca + a2) c2 + ca + a2 . Ta sẽ chứng minh với mọi x, y dương, ta có x2 − xy + y2 x2 + xy + y2 ≥ 1 3 . Thật vậy x2 − xy + y2 x2 + xy + y2 ≥ 1 3 ⇔3(x2 − xy + y2 ) ≥ x2 + xy + y2 ⇔2(x − y)2 ≥ 0 ( luôn đúng) . Áp dụng kết quả trên, ta có P ≥ 1 3 (a + b) + 1 3 (b + c) + 1 3 (c + a) = 2 3 (a + b + c). Lại theo bất đẳng thức AM-GM, ta có P ≥ 2 3 (a + b + c) ≥ 2.3 3 √ abc 3 = 2. Dấu "= " xảy ra khi và chỉ khi a = b = c = 1. Vậy giá trị nhỏ nhất của A bằng 2. 4.2 Sử dụng bất đẳng thức Cauchy-Schwarz Bài toán 4.8. Với các số thực dương a, b, c thỏa mãn a + b + c = 3. Tìm giá trị nhỏ nhất của biểu thức P = 1 (1 + √ ab)2 + 1 (1 + √ bc)2 + 1 (1 + √ ca)2 . 68
  • 70. Chương 4. Một số lớp bài toán cực trị với đa thức đối xứng ba biến Chứng minh. Áp dụng liên tiếp bất đẳng thức Cauchy- Schwarz, ta có 3P =3 1 (1 + √ ab)2 + 1 (1 + √ bc)2 + 1 (1 + √ ca)2 ≥ 1 1 + √ ab + 1 1 + √ bc + 1 1 + √ ca 2 ≥ (1 + 1 + 1)2 1 + √ ab + 1 + √ bc + 1 + √ ca 2 = 81 (3 + √ ab + √ bc + √ ca)2 . Mặt khác theo bất đẳng thức AM-GM, ta có √ ab ≤ a + b 2 √ bc ≤ b + c 2 √ ca ≤ c + a 2 . Do đó √ ab + √ bc + √ ca ≤ a + b + c Nên 3P ≥ 81 (3 + a + b + c)2 = 9 4 hay P ≥ 3 4 . Đẳng thức xảy ra khi và chỉ khi a = b = c = 1. Vậy giá trị nhỏ nhất của P = 3 4 . Bài toán 4.9. Cho các số thực dương x, y, z thỏa mãn điều kiện x + y + z = 3. Tìm giá trị nhỏ nhất của biểu thức P = x4 + 2y4 + 3z4 Chứng minh. 69
  • 71. Chương 4. Một số lớp bài toán cực trị với đa thức đối xứng ba biến Áp dụng bất đẳng thức Cauchy- Schwarz ta có x4 + 2y4 + 3z4 1 + 1 3 √ 2 + 1 3 √ 3 ≥ x2 + 3 √ 2y2 + 3 √ 3z2 2 và x2 + 3 √ 2y2 + 3 √ 3z2 1 + 1 3 √ 2 + 1 3 √ 3 ≥ (x + y + z)2 = 9 Do đó P = x4 + 2y4 + 3z4 ≥ 81 1 + 1 3 √ 2 + 1 3 √ 3 3 Dấu bằng xảy ra khi và chỉ khi    x = 3 3 √ 6 3 √ 6 + 3 √ 3 + 3 √ 2 y = 3 3 √ 3 3 √ 6 + 3 √ 3 + 3 √ 2 z = 3 3 √ 2 3 √ 6 + 3 √ 3 + 3 √ 2 Vậy giá trị nhỏ nhất của P là 81 1 + 1 3 √ 2 + 1 3 √ 3 3 . Bài toán 4.10. Với a, b, c là các số thực thỏa mãn a + b + c = 3. Tìm giá trị nhỏ nhất của biểu thức A = 1 a2 + b2 + c2 + 1 a2 + 1 b2 + 1 c2 . Chứng minh. Áp dụng bất đẳng thức AM-GM, ta có 1 a2 + 1 b2 + 1 c2 ≥ 1 ab + 1 bc + 1 ca . Do đó A ≥ 1 a2 + b2 + c2 + 1 ab + 1 bc + 1 ca . 70
  • 72. Chương 4. Một số lớp bài toán cực trị với đa thức đối xứng ba biến Áp dụng liên tiếp bất đẳng thức Cauchy - Schwarz, ta có A ≥ 1 a2 + b2 + c2 + 32 (ab + bc + ca) = 1 a2 + b2 + c2 + 92 9(ab + bc + ca) ≥ (1 + 9)2 a2 + b2 + c2 + 9(ab + bc + ca) = 102 (a + b + c)2 + 7(ab + bc + ca) . Áp dụng bất đẳng thức AM-GM ta có 7(ab + bc + ca) ≤ 7 3 (a + b + c)2 = 21. Do đó A ≥ 102 32 + 21 = 10 3 . Đẳng thức xảy ra khi và chỉ khi a = b = c = 1. Vậy giá trị nhỏ nhất của A bằng 10 3 . Bài toán 4.11. Với các số thực dương a, b, c thỏa mãn abc = 1. Tìm giá trị lớn nhất của biểu thức F = √ a2 + 1 + √ b2 + 1 + √ c2 + 1 a + b + c . Chứng minh. Áp dụng bất đẳng thức Cauchy-Schwarz, ta có 1 a2 + 1 + 1. √ 2a ≤ (1 + 1)(a2 + 1 + 2a) = √ 2(a + 1). Tương tự b2 + 1 + √ 2b ≤ √ 2(b + 1) c2 + 1 + √ 2c ≤ √ 2(c + 1). Cộng các bất đẳng thức cùng chiều ta có a2 + 1 + b2 + 1 + c2 + 1 + √ 2( √ a + √ b + √ c) ≤ √ 2(a + b + c + 3) ⇔ a2 + 1 + b2 + 1 + c2 + 1 ≤ √ 2(a + b + c + 3) − √ 2( √ a + √ b + √ c). 71
  • 73. Chương 4. Một số lớp bài toán cực trị với đa thức đối xứng ba biến Mặt khác theo bất đẳng thức AM-GM, ta có √ a + √ b + √ c ≥ 3 3 √ abc = 3 ⇔ − √ 2( √ a + √ b + √ c) ≤ −3 √ 2. Do đó a2 + 1 + b2 + 1 + c2 + 1 ≤ √ 2(a + b + c + 3) − 3 √ 2 = √ 2(a + b + c). Suy ra F ≤ √ 2 dấu "=" xảy ra khi và chỉ khi a = b = c = 1. Vậy giá trị lớn nhất của F là √ 2. Bài toán 4.12. Với các số thực dương x, y, z thỏa mãn x + y + z = 1. Tìm giá trị lớn nhất của biểu thức P = xy x2 + y2 + 2z2 + yz y2 + z2 + 2x2 + zx z2 + x2 + 2y2 . Chứng minh. Áp dụng bất đẳng thức Cauchy -Schwarz, ta có P = xy 3 2 1 9 + 1 9 + 2 9 (x2 + y2 + 2z2) + yz 3 2 1 9 + 1 9 + 2 9 (y2 + z2 + 2x2) + zx 3 2 1 9 + 1 9 + 2 9 (z2 + x2 + 2y2) ≤ xy 3 2 x 3 + y 3 + 2z 3 + yz 3 2 y 3 + z 3 + 2x 3 + zx 3 2 z 3 + x 3 + 2y 3 = 2xy (x + z) + (y + z) + 2yz (y + x) + (z + x) + 2zx (z + y) + (x + y) ≤ 1 4 2xy x + z + 2xy y + z + 1 4 2yz y + x + 2yz z + x + 1 4 2zx z + y + 2zx x + y = 1 4 (2x + 2y + 2z) = 1 2 . Dấu "=" xảy ra khi và chỉ khi x = y = z = 1 3 . Vậy giá trị lớn nhất của P là 1 2 . 72
  • 74. Chương 4. Một số lớp bài toán cực trị với đa thức đối xứng ba biến Bài toán 4.13. Với các số thực dương a, b, c thỏa mãn điều kiện a + b + c = 3 4 . Tìm giá trị nhỏ nhất của P = 1 3 √ a + 3b + 1 3 √ b + 3c + 1 3 √ c + 3a . Chứng minh. Áp dụng bất đẳng thức AM-GM, ta có P ≥ 1 a + 3b + 1 + 1 3 + 1 b + 3c + 1 + 1 3 + 1 c + 3a + 1 + 1 3 = 3 a + 3b + 2 + 3 b + 3c + 2 + 3 c + 3a + 1 . Theo bất đẳng thức Cauchy-Schwarz, ta có P ≥ 3 a + 3b + 2 + 3 b + 3c + 2 + 3 c + 3a + 1 ≥ 3 9 4(a + b + c) + 6 = 3. Dấu"=" xảy ra khi và chỉ khi a = b = c = 1 3 . Vậy giá trị nhỏ nhất của P là 3. 4.3 Sử dụng các tính chất của hàm số Bài toán 4.14. Với các số thực không âm a, b, c thỏa mãn a + b + c = 1. Tìm giá trị nhỏ nhất của P = 3(a2 b2 + b2 c2 + c2 a2 ) + 3(ab + bc + ca) + 2 a2 + b2 + c2. Chứng minh. Vì a2 + b2 + c2 = (a + b + c)2 − 2(ab + bc + ca) = 1 − 2(ab + bc + ca). Và theo bất đẳng thức Cauchy- Schwarz, ta có 3(a2 b2 + b2 c2 + c2 a2 ) ≥ (ab + bc + ca)2 . Do đó P ≥ (ab + bc + ca)2 + 3(ab + bc + ca) + 2 1 − 2(ab + bc + ca). Đặt ab + bc + ca = t. 73
  • 75. Chương 4. Một số lớp bài toán cực trị với đa thức đối xứng ba biến Có 0 ≤ t = ab + bc + ca ≤ (a + b + c)2 3 = 1 3 . Khi đó P ≥ t2 + 3t + 2 √ 1 − 2t, t ∈ 0; 1 3 . Xét hàm số f(t) = t2 + 3t + 2 √ 1 − 2t, t ∈ 0; 1 3 . f (t) = 2t + 3 − 2 √ 1 − 2t , f (t) = 2 − 2 (1 − 2t)3 ≤ 0, ∀t ∈ 0; 1 3 . Suy ra f (t) ≥ f ( 1 3 ) = 11 − 6 √ 3 3 > 0 ⇒ f(t) ≥ f(0) = 2, ∀t ∈ 0; 1 3 . Do đó P ≥ 2. Đẳng thức xảy ra khi và chỉ khi    ab + bc + ca = 0 ab = bc = ca a + b + c = 1 ⇔ a = 1; b = c = 0 và các hoán vị . Vậy giá trị nhỏ nhất của P là 2. Bài toán 4.15. Với các số thực dương a, b, c thỏa mãn điều kiện a + b + c = 3. Tìm giá trị nhỏ nhất của P = a b + b c + c a − 4abc. Chứng minh. Áp dụng bất đẳng thức AM- GM, ta có P = a √ ab + b √ bc + c √ ca − 4abc ≥ 2a a + b + 2b b + c + 2c c + a − 4abc = 2 a a + b + b b + c + c c + a − 4abc. 74
  • 76. Chương 4. Một số lớp bài toán cực trị với đa thức đối xứng ba biến Áp dụng bất đẳng thức Cauchy-Schwarz, ta có P ≥ 2 a a + b + b b + c + c c + a − 4abc ≥ 2 ( √ a + √ b + √ c)2 2(a + b + c) − 4abc = ( √ a + √ b + √ c)2 3 − 4abc ≥ 3 3 √ abc − 4abc. Đặt 3 √ abc = t với 0 < t = 3 √ abc ≤ a + b + c 3 = 1. Xét hàm f(t) = 3t − 4t3, t ∈ (0; 1], f (t) = 3 − 12t2 ⇒f (t) = 0 ⇔   t = 1 2 ∈ (0; 1] t = −1 2 /∈ (0; 1]. Bảng biến thiên t 0 1 2 1 f (t) | + 0 − | f(t) 0 1 −1 Từ bảng biến thiên ta có f(t) ≥ −1 ⇒ P ≥ f(t) ≥ −1. Đẳng thức xảy ra khi và chỉ khi a = b = c = 1. Vậy giá trị nhỏ nhất của P là -1. Bài toán 4.16. Với a, b, c là các số thực dương thỏa mãn a + b + c = 3. Tìm giá trị nhỏ nhất của biểu thức P = ab + bc + ca + 1 abc − abc. Chứng minh. Áp dụng bất đẳng thức AM-GM, ta có 3 = a + b + c ≥ 3 3 √ abc ⇒ abc ≤ 1. Theo giải thiết ta có 0 < abc ≤ 1. Khi đó P = ab + bc + ca + 1 abc − abc ≥ 3 3 √ a2b2c2 + 1 abc − abc. 75
  • 77. Chương 4. Một số lớp bài toán cực trị với đa thức đối xứng ba biến Đặt 3 √ abc = t với t ∈ (0; 1]. Ta có P ≥ 3t2 + 1 t3 − t3 . Xét hàm số f(t) = 3t2 + 1 t3 − t3 với t ∈ (0; 1] ta có f (t) =6t − 3 t4 − 3t2 = −3t6 + 6t5 − 3 t4 = 3(t − 1)(−t5 + t4 + t2 + t + 1) t4 ≤ 0 ∀t ∈ (0; 1] . Do đó P ≥ f(t) ≥ f(1) = 3. Đẳng thức xảy ra dấu bằng khi và chỉ khi t = 1 ⇒ a = b = c = 1. Vậy giá trị nhỏ nhất của P là 3. Bài toán 4.17. Với 3 số x, y, z thỏa mãn x, y, z ∈ [−1; 3] x + y + z = 3 . Tìm giá trị lớn nhất của P = x2 + y2 + z2 . Chứng minh. Giả sử 3 ≥ x ≥ y ≥ z ≥ −1 . Khi đó    3 ≥ x 3 + 1 ≥ x + y 3 + 1 + (−1) = x + y + z Xét hàm số f(t) = t2, với t ∈ [−1; 3] f (t) = 2t f (t) = 2 ≥ 0, với t ∈ [−1; 3]. Do đó với x, y, z ∈ [−1; 3], theo bất đẳng thức Karamata ta có f(3) + f(1) + f(−1) ≥ f(x) + f(y) + f(z) ⇔x2 + y2 + z2 ≤ 11. Đẳng thức xảy ra khi và chỉ khi x = 3; y = 1; z = −1 và các hoán vị của chúng. 76
  • 78. Chương 4. Một số lớp bài toán cực trị với đa thức đối xứng ba biến Bài toán 4.18. Với −1 ≤ a, b, c ≤ 1 và a + b + c = −1 2 . Hãy tìm giá trị lớn nhất của biểu thức F = a12 + b12 + c12 . Chứng minh. Không mất tính tổng quát, giả sử 1 ≥ a ≥ b ≥ c ≥ −1. Khi đó    a ≤ 1 a + b ≤ 1 − 1 2 a + b + c = 1 − 1 2 − 1 . Vì hàm số f(x) = x12 lồi trên [−1; 1], theo bất đẳng thức Karamata, ta có a12 + b12 + c12 = f(a) + f(b) + f(c) ≤ f(1) + f( −1 2 ) + f(−1) = 2 + 1 212 Đẳng thức xảy ra khi a = 1, b = −1 2 , c = −1 và các hoán vị của chúng. Vậy giá trị lớn nhất của F bằng 2 + 1 212 . 4.4 Bài toán liên quan Bài toán 4.19. Với ba số thực dương x, y, z thỏa mãn xyz = 1. Tìm giá trị nhỏ nhất của biểu thức P = x3 (1 + y)(1 + z) + y3 (1 + z)(1 + x) + z3 (1 + x)(1 + y) . Bài toán 4.20. Với x, y, z là các số thực dương thỏa mãn x + y + z = 6. Tìm giá trị lớn nhất của biểu thức P = x2 2 + y3 12 + z4 108 . Bài toán 4.21 (HSG Hà nội 2014-2015). Vơi ba số thực dương a, b, c thỏa mãn a + b + c = 1. Tìm giá trị nhỏ nhất của biểu thức P = 1 a2 + b2 + c2 + 1 abc . Bài toán 4.22 (Đại học khối B năm 2012). Với các số thực x, y, z thỏa mãn x + y + z = 0; x2 + y2 + z2 = 1. Tìm giá trị lớn nhất của P = x5 + y5 + z5 . 77
  • 79. KẾT LUẬN Các bài toán bất đẳng thức và cực trị với đa thức đối xứng vẫn là một chủ đề hay và khó với học sinh và ngay cả giáo viên. Tuy nhiên trong các kì thi Đại học, thi học sinh giỏi quốc gia, thi Olympic Toán học quốc tế lại thường có dạng toán này. Đặc biệt là các bài toán bất đẳng thức và cực trị với đa thức đối xứng ba ba biến. Luận văn hoàn thành và đạt được các kết quả sau: 1. Giới thiệu lại các kiến thức cơ bản của đa thức đối xứng ba biến, của bất đẳng thức và các bất đẳng thức AM-GM, bất đẳng thức Cauchy - Schwarz và bất đẳng thức Karamata cùng bổ đề cơ bản. 2. Đưa ra các cách tiếp cận đối với bất đẳng thức đối xứng ba biến có tổng - tích không đổi và hệ thống các ví dụ và đề thi quốc gia và quốc tế. 3. Đưa ra lớp các bài toán cực trị với đa thức đối xứng ba biến và các cách giải quyết bài toán cung hệ thống ví dụ đa dạng. Với những gì đạt được, tôi hi vọng luận văn sẽ là một tài liệu có ích cho những ai quan tâm tới bất đẳng thức và bài toán cực trị với đa thức đối xứng ba biến. Trong điều kiên về thời gian và khuôn khổ của luận văn, tôi chưa thể nghiên cứu thêm về bất đẳng thức và bài toán cực trị với đa thức đối xứng khác. Đó là hướng phát triển của luận văn. Tác giả rất mong nhận được sự góp ý của các thầy cô và các bạn để luận văn được hoàn thiện hơn. Xin chân thành cảm ơn! 78
  • 80. Chương 4. Một số lớp bài toán cực trị với đa thức đối xứng ba biến TÀI LIỆU THAM KHẢO Tiếng Việt [1] Võ Quốc Bá Cẩn - Trần Quốc Anh, Sử dụng AM-GM để chứng minh bất đẳng thức, NXBĐH Sư Phạm. [2] Phạm Kim Hùng, Sáng tạo bất đẳng thức, NXB Hà Nội. [3] Nguyễn Văn Mậu, 2002, Bất đẳng thức, định lý và áp dụng, NXBGD. [4] Nguyễn Văn Mậu, Các bài toán nội suy và áp dụng,NXB GD . [5] Nguyễn Văn Mậu, Đa thức đại số và phân thức hữu tỷ, NXB GD 2002. [6] N.V. Mậu, T.N. Dũng, N.Đ. Phất, N.T. Thanh, Số phức và áp dụng, NXB GD 2009. [7] Trần Phương, Võ Quốc Bá Cẩn, Trần Quốc Anh, Vẻ đẹp bất đẳng thức trong các kì thi Olympic toán học, NXBĐHQG Hà Nội. [8] Cao Minh Quang,Một số dạng toán về bất đẳng thức ba biến với tích các biến không đổi, Hội thảo khoa học Các chuyên đề toán học bồi dưỡng học sinh giỏi Đồng Tháp 2013. [9] Phạn Văn Thuận, Lê Vĩ, Bất đẳng thức suy luận và khám phá, NXBĐHQG Hà Nội. Tiếng Anh [10] D. Djukic, V. Jankovic, I. Matic and N. Petrovic, The IMO Compendium 1959-2004, Springer-Verlag 2004. [11] D, S. Mitrinovic, J. E. Pecaric, ” Recent Advances in Geometric Inequal- ities”, Kluwer Academic Publishers, 1989. 79